You are on page 1of 364

INTRODUCTION

PHYSIOLOGY
- Physiology is the study of normal functions of living organisms. It
includes many branches like viral physiology, bacterial physiology,
cellular physiology, plant physiology, animal physiology and human
physiology.
- Human physiology is the branch of physiology which is concerned
with the functions of the entire human body; from the sub-cellular
components to the organs and organ systems. It is also concerned
with how these functions are performed and how they are integrated.

HOMEOSTASIS
- All living organisms are composed of cells. Cells of the body do not
only contain water, but also surrounded by water (= intracellular and
extracellular fluid compartments). The extracellular fluid is the link
between the external world and the cells. It carries nutrients to the
cells and eliminates their waste products. It circulates between all
cells in the body and provides for them a homogenous environment.
In other words, it is essential for survival of cells. Disturbance of this
extracellular fluid impairs functions of cells and results in disease. For
this reason it is described as the "internal environment" (Claude
Bernard, 19th century). Later on, the term "Homeostasis" was
applied. It indicates that: "all systems in the body, however various,
they have one goal; to maintain the constancy of the internal
environment (which is the ECF)". Therefore each organ in the body
participates in homeostasis by maintaining constant ECF volume,
osmolarity, pH, pressure or temperature).
The core of medical physiology (1) – 3rd edition Page 1
CELL STRUCTURE
- Cells are the building blocks of the body (the basic units of each
tissue). The entire body contains about 75 to 100 trillion cells.
Although these cells are highly specialized in various organs to
perform specific functions, the structures inside their cytoplasm (i.e.
the organelles) are almost identical in all types of cells. These include
the following:
 The cell membrane:
- Phospholipid bilayer (about 25%) with proteins (about 50%); plus
some cholesterol (13%), carbohydrates (3%) and other lipids.
- The phospholipids have hydrophilic parts "phosphate" facing
outside and the hydrophobic parts "fatty acids" in the interior of the
membrane.
- Thickness = 7.5 nm (75 Angstrom)
- It is a semi-permeable membrane (allows passage of lipid soluble
substances and prevents passage of water and water soluble ones).
However, the protein channels and carriers in the membrane
facilitate passage of many substances- see below.
- It contains two types of protein:
o Peripheral proteins: attached to one surface of the cell
membrane (usually the inner surface)
o Integral proteins: extends throughout the cell membrane
- Functions of proteins in the cell membrane:
o Offer structural support to the membrane (cytoskeleton)
o Act as adhesion molecules (connect cells together)
o Act as enzymes (catalyze chemical reactions on the cell
membrane)

The core of medical physiology (1) – 3rd edition Page 2


o Act as antigens (usually glycoproteins; like the blood
group antigens on surface of red blood cells and the “HLA”
antigens (human leukocyte antigens) on surface of all
nucleated cells.
Notes about the HLA antigens:
 They are encoded by group of genes in the short arm (P
arm) of chromosome 6. These genes are known as the
genes of the major histocompatibility complex (MHC). They
include different classes (e.g. MHC class I, MHC class II &
MHC class III).
 HLA antigens (also known as MHC antigens) are unique
for every person; that’s why they are used by the immune
system to distinguish self cells or antigens from non-self
(they are considered in selection of a donor in organ
transplantation).
o Act as ion channels for movement of water and ions
across the cell membrane (osmosis and simple diffusion)
o Act as carriers for passive transport of certain substances
across the cell membrane (facilitated diffusion; see below)
o Act as pumps for active transport of certain substances
across the cell membrane (active transport; see below)
o Act as receptors for hormones and neurotransmitters
 Remember that peripheral proteins act as enzymes
whereas integral proteins carry out the other functions.
 Carbohydrates on the surface of a cell membrane (=
glycocalyx) are either attached to proteins (forming
glycoproteins) or to lipids (forming glycolipids).

The core of medical physiology (1) – 3rd edition Page 3


Fig 1: The cell membrane

 The nucleus
- Contains chromatin (DNA) which condenses to form chromosomes
before cell division. The DNA replicates during cell division to carry
genetic information from the mother cell to the daughter cells.
- The nucleus also contains one or more nucleoli rich in ribosomal
RNA (the RNA is synthesized from DNA by transcription). The
ribosomal RNA diffuses to the cytoplasm to be translated into
proteins (in the ribosomes of the rough endoplasmic reticulum).
- The proteins may act within the cell or may be packed within
vesicles (in the Golgi apparatus) for secretion to the outside.
 The endoplasmic reticulum
- Complex meshwork of canals and vesicles, extending from the
nucleus to the exterior of the cell.
- Two types:
The core of medical physiology (1) – 3rd edition Page 4
o Smooth endoplasmic reticulum:
- Has no ribosome on its surface
- For synthesis of lipids and steroids
- Contains enzymes for certain metabolic functions within
the cell (e.g. detoxification of foreign substances)
o Rough endoplasmic reticulum:
- Has ribosomes on its surface
- For protein synthesis
 The Golgi apparatus
- Closely associated with the rough endoplasmic reticulum.
- For processing and package of proteins synthesized in the rough
endoplasmic reticulum into secretory vesicles (most of the packed
proteins act as enzymes).
 The mitochondria
- The power houses of cells (provide the energy used by the cell to
perform its functions). They are abundant in certain cells like
endocrine cells, parietal cells and renal cells (because these cells
need energy for synthesis of hormones or active transport of ions).
- Each mitochondrion is surrounded by two phospholipid bilayer
membranes; the inner one is folded to produce cristae. The cristae
and the inner cavity of the mitochondrion (the matrix) contain the
respiratory enzymes needed for oxidative phosphorylation of glucose
to release large amount of energy in form of ATP (Adenosine
diphosphate).
- Each mitochondrion contains DNA that plays a role in formation of
few intra-mitochondrial proteins (using mitochondrial ribosomes) and
in its own replication.

The core of medical physiology (1) – 3rd edition Page 5


- Abnormalities of mitochondrial DNA may result in certain diseases,
usually affecting the high energy tissues (muscle, heart and brain).
These diseases are always inherited from mothers (this is because
the defective mitochondria are inherited through ova of mothers;
whereas sperms of fathers do not contain mitochondria).
 The lysosomes
- Vesicles formed from the endoplasmic reticulum and Golgi
apparatus. They contain hydrolytic enzymes (proteases, lipases,
carbohydrases & nucleases) that are used in hydrolysis or digestion
of engulfed material (e.g. digestion of bacteria within vacuoles of the
white blood cells).

Fig 2: The cell organelles

The core of medical physiology (1) – 3rd edition Page 6


TRANSPORT MECHANISMS ACROSS CELL MEMBRANES
- There are constant movements of O2, CO2, nutrients, electrolytes
and waste products across cell membranes. A variety of transport
mechanisms are involved, these are generally classified into passive
and active transport mechanisms.
 Passive transport mechanisms:
- Do not consume energy in transport.
- Transport substances from areas of higher concentration to
areas of lower concentration (i.e. down the concentration or
electrical gradient).
- Either do not use carrier (= simple diffusion) or use carrier (=
facilitated diffusion).
 Active transport mechanisms:
- Consume energy in transport.
- Transport substances from areas of lower concentration to
areas of higher concentration (i.e. against the concentration or
electrical gradient).
- Always need carrier for transport.
- Transport of a substance against its chemical or electrical gradient,
with consumption of energy and usage of a carrier is known as
primary active transport.
- Transport of a substance with an other one that’s transported
actively is known as secondary active transport. Here the substance
uses the same carrier that’s used by the other substance.
- Secondary active transport (also known as co-transport) may occur
in the same direction of the primary substance (= symport), or in the
opposite direction (= antiport).

The core of medical physiology (1) – 3rd edition Page 7


 Substances transported by simple diffusion
- Diffusion is the process by which a substance expands, because of
the random movement of its particles to fill the available volume.
- Non-polar substances transported by simple diffusion include:
o Fatty acids
o Steroid hormones (synthesized from cholesterol)
o Gases (O2 and CO2)
- Simple diffusion of polar substances (water soluble substances) like
ions is low. However, it can occur through certain "ion channels"
(integral proteins in the cell membrane).
- Passive diffusion of water through cell membranes is known as
osmosis. It occurs through certain water channels known as
aquaporins. Water moves from the side of lower concentration of a
solute to the side of higher concentration of the solute.

Fig 2: Passive transport

The core of medical physiology (1) – 3rd edition Page 8


 Substances transported by facilitated diffusion
- Do not consume energy in the process of transport.
- Have a maximum rate of transport that depends on the density of
carriers on the cell membrane. The maximum rate is reached when
all the carriers are saturated.
- Examples include transport of glucose through the basolateral
membranes of renal and intestinal cells and its absorption from the
ECF by most cells of the body.
Substances transported actively
- The well known example of primary active transport is the pump
that transports sodium and potassium against their concentrations (=
The Na+ / K+ pump). It transports three atoms of sodium from ICF to
ECF in exchange to two atoms of potassium from ECF to ICF.

Fig 3: Active transport

The core of medical physiology (1) – 3rd edition Page 9


- The well known example of secondary active transport is the
transport of glucose (coupled to sodium) through the luminal
membranes of intestinal and renal cells.
- Secretion of hydrogen by renal cells is another example of
secondary active transport. However, hydrogen moves in the
opposite direction to sodium (anti-port).

 Other transport mechanisms


o Endocytosis (active vesicular transport)
- Endocytosis is the uptake of molecules into cells. Here a molecule
fuses with the cell membrane, invaginates it and then the invagiation
is separated from the cell membrane to form of a vesicle.
- Special proteins may facilitate the process of endocytosis (clathrin
and dynamin).
- When the engulfed substance is dissolved in fluid, the process is
known as pinocytosis (cell drinking); and when it is a particulate
matter or bacteria, the process is known as phagocytosis (cell
eating).
o Exocytosis (active vesicular transport)
- Exocytosis is the release of substances from cells. (i.e. opposite to
endocytosis). Proteins synthesized within the cell are usually packed
into secretory vesicles and secreted by exocytosis.
- Notice that exocytosis requires calcium, energy and certain
proteins.
o Solvent drag (passive transport)
- During diffusion of a solvent, it tends to drag some solute with it.
This occurs in capillaries.

The core of medical physiology (1) – 3rd edition Page 10


Fig 3: Endocytosis & Exocytosis

 Transport proteins "carriers, pumps & ion channels"


- These are highly specialized transmembrane proteins that allow
passage of water, ions, glucose, urea and other substances through
cell membranes.
- The carriers change their shape (configuration) when they bind
their substances to move them from one side of the cell membrane to
the other side, usually down the chemical or electrical gradients (=
facilitated diffusion).
- The pumps act as ATPase enzymes to catalyze hydrolysis of ATP.
The released energy is used in active transport of substances,
against their chemical or electrical gradients. Examples include the
Na+/ K+ ATPase pump, the proton pump and the Ca2+ ATPase pump.

The core of medical physiology (1) – 3rd edition Page 11


(Remember that the active transport is either primary active or
secondary active, see above).
- The ion channels allow simple diffusion (down chemical or
electrical gradients). They include:
 Leak channels
- Always open
- Example: Potassium leak channels which are responsible
for the resting membrane potentials
- Resting membrane potentials are found in almost all cells
in the body (see chapter two)
 Voltage gated channels
- Have gates that open or close in response to changes in
voltage "or potential" in the cell membrane
- Examples: voltage gated sodium channels & voltage
gated potassium channels which are responsible for the
depolarization and the repolarization phases of action
potentials respectively
- Action potentials are only found in excitable tissues
(nerve and muscle)
 Ligand gated channels
- Have gates that open when certain membrane receptors
bind to specific neurotransmitters or hormones, and close
when these chemicals are released from the receptors
 Mechano-sensitive channels
- Have gates that open in response to direct mechanical
stimulation of the cell membrane
- They are involved in movement of some cells

The core of medical physiology (1) – 3rd edition Page 12


QUESTIONS FOR SELF ASSESSMENT-1 (BEST OF FIVE)
1- Concerning the cell organelles:
a. mitochondria synthesize proteins
b. endoplasmic reticulum is needed for cell division
c. nuclei are the power houses of cells
d. lysosomes contain hydrolytic enzymes
e. Golgi apparatus translates RNA into proteins
2- Water passes through cell membranes by:
a. facilitated diffusion
b. primary active transport
c. osmosis
d. co transport
e. none of the above
3- Maintenance of constant internal environment is known as:
a. endocytosis
b. feedback mechanism
c. physiology
d. homeostasis
e. haemostasis
4- Facilitated diffusion:
a. occurs against the electrical gradient
b. requires energy
c. not influenced by the concentration gradient
d. has no transport maximum capacity
e. mediated by a protein carrier
5- Which of the following can cross the cell membrane passively?
a. Proteins
b. carbon dioxide
c. potassium ions
d. calcium ions
e. glucose
6- Facilitated diffusion differs from simple diffusion in that it:
a. does not require energy
b. occurs against the electrical gradient
c. is not influenced by the concentration gradient
d. is mediated by a protein carrier
e. has no transport maximum capacity
7- Which of the following is a passive type of transport:
a. solvent drag
b. endocytosis
c. exocytosis
d. co-transport
e. antiport

The core of medical physiology (1) – 3rd edition Page 13


8- Concerning transport of ions across cell membranes, which of
the following is not true:
a. secondary active transport requires energy
b. simple diffusion occurs down the concentration gradient
c. facilitated diffusion occurs against the electrical gradient
d. active transport is mediated by a protein carrier
e. secondary active transport has a transport maximum capacity
9- Concerning transport across cell membranes, all the following
require integral proteins to cross cell membranes except:
a. water
b. glucose
c. oxygen
d. potassium ions
e. calcium ions
10- Leak channels:
a. are present in almost all cells in the body
b. have gates that open or close in response to changes in potential
c. open when a hormone binds to a nearby membrane receptor
d. are responsible for the depolarization phase of action potentials
e. are activated by mechanical stimulation of cell membranes
11- All of the following substances cross the cell membrane
through channels or transporters except:
a. sodium
b. bicarbonate
c. oxygen
d. water
e. potassium
12- Homeostasis is:
a- arrest of bleeding
b- maintenance of constancy of the external environment
c- formation of a blood clot
d- normal pH
e- represented by control of body temperature

Question 1. 2. 3. 4. 5. 6. 7. 8. 9. 10. 11. 12.


Answer d c d e b d a c c a c e

The core of medical physiology (1) – 3rd edition Page 14


CHAPTER 1
BODY FLUIDS
TOTAL BODY WATER (TBW)
-Body composition in a 70 Kg young adult male:
– Water = 60% (of the total weight which is 70 kg)
– Proteins = 18%
– Fats = 15%
– Minerals = 7%
– Carbohydrates < 1%
-Therefore: water is the most important constituent in the body.
-With total deprivation of water, survival is limited to a few days,
whereas total food deprivation is tolerated for at least a month.
-Total body water (in a 70 kg, young adult male):
= 60 % of the total body weight
= 42 Kg (60/100 x 70) = 42 L (because density of water = 1)
 Variation in the % of TBW among different subjects
-The % of TBW varies according to:
1) Age
- TBW decreases with age (e.g. in an embryo it is near to
100%, in a neonate = 80%, in an adult male = 60% and above
the age of 60 years = 52% in males).
2) Sex
- The % of TBW is higher in males when compared to
equivalent females. This is because the female has higher
percentage of fat in her body, compared to an equivalent male,
with the same age and weight.

The core of medical physiology (1) – 3rd edition Page 15


- The higher percentage of fat is associated with less
percentage of water because fat cells contain less water than
other types of cells. For example, water in a fat cell is about
13% whereas in a muscle cell is about 75%.
3) Body size
- The % of TBW is higher in thin-tall subjects compared to
obese-short subjects.
- This is also explained by the higher percentage of fat in the
obese subject, and therefore less percentage of water.

Table1.1: % of TBW in males and females of different ages


Age % of water in a male % of water in a female
Embryo Almost 100% Almost 100%
Neonate 80% 80%
Adult 60% 51%
Elderly 52% 46%

 Body Fluid Compartments


- As mentioned in the introduction of this book, cells of the body do
not only contain water, but also surrounded by water.
- Therefore, total body water is divided into two compartments:
– 1) Intracellular fluid (ICF): = 2/3 of the total body water or 40%
of the total body weight (TBwt).
– 2) Extracellular fluid (ECF): = 1/3 of the total body water or 20%
of the total body weight (TBwt).
- ECF is further divided into:
– a) Interstitial fluid: = 75% of ECF or 15% of TBwt.
The core of medical physiology (1) – 3rd edition Page 16
– b) Intravascular fluid (plasma): = 25% of ECF or 5% of TBwt.
– c) Trans-cellular fluid: Negligible.
Notes about trans-cellular fluid:
 It includes synovial fluid, pleural fluid, pericardial fluid,
peritoneal fluid, cerebrospinal fluid...
 Its volume is very low, that’s why it is not included in
calculations of ECF.
 In abnormal conditions (like pleural effusion, pericardial
effusion and ascites) its volume becomes very high.

- In a 70 Kg adult male:
– Total body water = 42 L (60% of the total body weight)
• ICF = 28 L (= 40% of total body weight)
• ECF = 14 L (= 20% of total body weight)
– ISF = 10.5 L (= 15% of total body weight)
– IVF (Plasma) = 3.5 L (= 5% of total body weight)
Question: Calculate the expected body fluid compartments in an
average 60 kg adult male
Answer: Total body weight= 60Kg, therefore: Total body water =
60/100 x 60 = 36L, ICF = 40/100 x 60 = 24 L, ECF = 20/100 x 60 =
12 L, ISF = 15/100 x 60 = 9 L and IVF = 5/100 x 60 = 3 L.
Notes about body fluid compartments in neonates:
 Total body water constitutes a very high proportion of the total
body weight (about 80%)
 ECF exceeds 30% and ICF volume is less than 40% of total
body weight. Therefore, ECF/ICF volume ratio is very high

The core of medical physiology (1) – 3rd edition Page 17


 Differences between ECF & ICF:
 ECF has: lower volume, higher pH and higher concentrations of
sodium, calcium, chloride & bicarbonate. Sodium is the main
cation and chloride is the main anion.
 ICF has: higher volume, lower pH and higher concentrations of
potassium, magnesium, phosphate, sulphate and protein.
Potassium is the main cation and non-diffusible anions (like
organic phosphate and protein) are the main anions.

Table 1.2: Differences between ECF and ICF in a 70 kg adult male

Difference Extracellular fluid Intracellular fluid

Volume 15L 25L


Conc. Of Cations:
Sodium (Na+) 142.0 mmol/L 10 mmol/L
Potassium (K+) 4.0 mmol/L 140 mmol/L
+2
Calcium (Ca ) 2.5 mmol/L Negligible (0)
Magnesium (Mg+2) 2.0 mmEq/L 26 mmEq/L
Conc. Of Anions:
Chloride (Cl-) 105.0 mmol/L 4 mmol/L
Bicarbonate (HCO3-) 25.0 mmol/L 10 mmol/L
Phosphate (PO4-3) 2.0 mEq/L 100 mEq/L
Protein 17.0 mEq/L 65 mEq/L
pH 7.4 7.2

Temperature 37 c 37 c

Osmolarity 280-300 mOsm/L 280-300 mOsm/L

The core of medical physiology (1) – 3rd edition Page 18


 Measurement of body Fluid Compartments
- Volumes of body fluid compartments can be measured using the
indicator (dye) dilution method.
- In this method, a known quantity of a substance (e.g. a dye) is
injected and allowed to distribute in the compartment of interest. After
distribution, a sample of fluid is taken from the same compartment to
measure the final concentration of the dye. Then the volume of the
compartment (known as the volume of distribution of the dye) is
calculated using the formula:
Volume of distribution = Q-e /C
Where:
- Q is the quantity of the dye injected
- e is the amount of the dye excreted or metabolized by cells
- C is the concentration of the dye after equilibration
- Substances used for measurement of body fluid compartments
should have the following characteristics:
- Non toxic
- Easily measured
- Their amounts are not altered by the body (not stored,
metabolized, excreted or produced by the body) or the
altered amount can be calculated easily
- Distribute only in the compartment being measured
- Do not affect water distribution in other compartments
Remember that:
 Leakage of a substance into other compartment decreases its
final concentration and therefore increases the calculated
volume of distribution.

The core of medical physiology (1) – 3rd edition Page 19


Examples for substances used in measurement of body fluids:
Substances used in measurement of total body water:
-Deuterium oxide (heavy water)
-Tritium oxide (3H2O = an isotope of water)
-Antipyrine
-Aminopyridine
Substances used in measurement of extracellular fluid:
-Saccharides (inulin, manitol & sucrose)
These fail to penetrate the trans-cellular fluid
Therefore they underestimate the ECF
-Radioactive electrolytes (sodium, chloride & bromide)
These easily penetrate the entire ECF and may
escape into cells
Therefore they overestimate the ECF
-Thiosulphate & thiocynate
Substances used in measurement of plasma:
-Radioactive iodine used to label serum albumin (RISA)
-Evan’s blue dye (which binds to plasma protein and stay in
plasma)
-Labeled macroglobulin
Substances used in measurement of Blood:
-Blood volume can be calculated from plasma and packed cell
volume (PCV) by the formula: Blood = Plasma X 100/100-PCV
-The PCV is measured by centrifugation of a sample of blood
in a capillary tube, and then the percentage of the packed
cells at the bottom of the tube is calculated (read about the
PCV in chapter 5).

The core of medical physiology (1) – 3rd edition Page 20


-It is also calculated from red blood cell volume and plasma
volume by the formula: Blood = plasma + red blood cell
volume.
-The red blood cell volume is obtained from the volume of
distribution of re-injected red blood cells labeled with
radioactive chromium (51Cr).
Intracellular fluid and interstitial fluid:
- Are measured indirectly by two substances, for two
compartments. Then ICF or ISF can be subtracted as follows:
ICF = TBW - ECF
ISF = ECF - plasma

Factors Affecting Body Fluid Compartments


Body fluid compartments are affected by:
1. Osmosis
2. Diffusion
3. Gibbs Donnan equilibrium
4. Sodium-potassium pump
5. Starling's forces
6. Abnormalities of water balance

1- Osmosis
- Osmosis is the movement of water molecules across a semi-
permeable membrane, from a region of lower concentration of a
solute to a region of higher concentration of the solute (see the
introduction).

The core of medical physiology (1) – 3rd edition Page 21


- All cell membranes and capillaries are semi-permeable membranes
(permeable to water and generally not permeable to solute).
- For osmosis to occur there should be a difference in solute
concentration between the two sides of the membrane, i.e. difference
in osmolarity.
Fig 1.1: Osmosis
1
Solution A has the same
concentration as solution B
= No osmosis

2
Solution A has higher
concentration than solution B
= Osmosis from B to A

_______________________________________________________

Osmolarity, Osmolality & Tonicity


Osmolarity
- Osmolarity is the number of osmoles of solute per one liter of
solution.
- It is used to describe concentrations of osmotically active particles in
a solution.
- If a solute dissociates into ions to form an ideal solution, each
liberated ion is an osmotically active particle.

The core of medical physiology (1) – 3rd edition Page 22


- For example: dissociation of one mole of (NaCl) gives one osmole of
sodium and one osmole of chloride (i.e. 2 osmoles).
- However, one mole of glucose (C6H12O6) in a solution gives one
osmole; because organic substances like glucose are non-ionizable.
- Remember that one mole of a substance contains the same number
of molecules that are found in one mole of any other substance (=
Avogadro's number = 6.061 x 1023).
- The osmotically active particles can exert osmotic pressure if they
are in contact with another solution, but separated from it by a semi-
permeable membrane (permeable to the solvent but not to the
solute).
- Osmotic pressure is defined as the pressure necessary to prevent
solvent migration (i.e. prevent osmosis).

Fig 1.2: The osmotic pressure

Osmotic pressure prevents


osmosis from solution B to
solution A

The core of medical physiology (1) – 3rd edition Page 23


Osmolality
- Osmolality is the number of osmoles per one kilogram of solvent.
- It is more accurate than osmolarity since it depends on mass (which
is constant) rather than volume (which is affected by changes in
temperature and pressure).
- In body fluids, where the solvent is water, the concentration of
solutes is very low (highly diluted); therefore one liter and one
kilogram are equal. More over, temperature and pressure are
constant under normal physiological conditions; that’s why osmolality
and osmolarity are equal in body fluids.
- Because of this similarity between osmolality and osmolarity, you
may find osmolality expressed in (mosm/L) rather than (mosm/Kg).
- Osmolality of plasma = (280-300) mosm/L. Na+ and its anions are
responsible for most of this value (Na+ determines ECF osmolality).
- Osmolality of intracellular fluid = (280-300) mosm/L. K+ and its anions
are responsible for most of this value (K+ determines ICF osmolality).

Tonicity
- This term is used when describing osmolality of a solution relative to
osmolality of the plasma.
- Accordingly, solutions may be:
o Isotonic (with osmolality similar to plasma)
o Hypotonic (with osmolality lower than plasma)
o Hypertonic (with osmolality higher than plasma)
- Intravenous (I.V.) infusion of each type of these solutions affects
volumes and osmolarities of body fluid compartments. These effects
can be studied from the following figure and table:

The core of medical physiology (1) – 3rd edition Page 24


Fig 1.3: The effects of different types of solutions on cells:

Table 1.3: Effects of I.V. solutions on volumes and osmolarities


ECF ICF
Solution Osmosis
Vol. Osmol. Vol. Osmol.

Isotonic The same The same The same

Hypotonic

Hypertonic

- From the above table:


- I.V. infusion of an isotonic solution increases volume of ECF
with no effect on its osmolarity, and no effect on volume or
osmolarity of ICF (i.e. no effect on cells).
- I.V. infusion of a hypotonic solution increases volumes of ECF
and ICF and decreases osmolarities of ECF and ICF.
- I.V. infusion of a hypertonic solution increases volume and
osmolarity of ECF, and decreases volume of ICF while its
osmolarity is increased.

The core of medical physiology (1) – 3rd edition Page 25


Calculation of osmolality:
A. From the Freezing Point Depression
- One osmole depresses the freezing point of a solution by 1.86 οc
- One milliosmole depresses the freezing point by 0.00186 οc
- Number of milliosmoles per liter in a solution =
The freezing point depression/0.00186
Q: Calculate the osmolality of normal human plasma if the freezing
point = - 0.55 οc
Answer: Plasma osmolality = 0.55/0.00186 = 295 mosm/L

B. From the Molarity


Number of osmoles = Number of moles x number of particles
(liberated by a single molecule)
Q: Calculate the osmolarity of 0.9% NaCl solution and mention the
effect of this solution on volume and osmolarity of ICF after its
infusion in the plasma? (Molecular weight of NaCl = 58.5).
Answer: 0.9% NaCl = 0.9 g/dL, (x 10) = 9 g/L
Molarity = [conc. g/L] / MWt of NaCl = 9 / 58.5 mol/L
= 0.154 mol/L, (x 1000) = 154 mmol/L
Osmolarity = 154 x 2 = 308 mosm/L
(Isotonic, has no effect on ICF)
(Remember that: This solution is regarded as isotonic solution
although its osmolarity is higher than plasma Osmolarity (> 300
mosm/L). This is because the dissociation of NaCl in plasma is not
as complete as in true ideal solutions (dissociation is about 93%).
Therefore the osmolarity of 0.9% NaCl solution in plasma is actually
less than 308 mosm/L; that’s why it is isotonic).

The core of medical physiology (1) – 3rd edition Page 26


Q: Calculate the osmolarity of 5% glucose solution. If one liter of this
solution is infused intravenously, mention the immediate and the later
effects on the cells? (MWt glucose = 180).
Answer: 5% glucose = 5g/dL = 50 g/L
Molarity = [Conc. g/L]/MWt
= 50/180 = 0.278 mol/L
(x 1000) = 278 mmol/L
Osmolarity = 278 x 1 = 278 mosm/L (Isotonic)
Immediate effect on cells: no effect because it is isotonic
Later effect on cells: After uptake of glucose by cells the solution
becomes hypotonic; water enters cells, it increases volume &
decreases osmolarity of ICF.

Q: Calculate the osmolality of a solution containing 110 mmol NaCl,


25 mmol NaHCO3, 2.5 mmol CaCl2, 5 mmol urea & 5 mmol glucose
Answer: Plasma osmolality = (110x2) + (25x2) + (2.5x3) + (5x1) +
(5x1) = 287.5 mosm/L.
(Remember that glucose and urea are non-ionizable).

C. Using a formula
Osmolarity of the plasma can be calculated using the following
formula: Plasma Osmolarity = 2([Na] + [K]) + [glucose] + [urea]
(All concentrations are in mmol/L)
Q: Calculate the osmolarity of the plasma if [Na] = 140 mmol/L, [K] =
4 mmol/L, [Glucose]= 5 mmol/L and [Urea]= 7 mmol/L.
Answer: Osmolarity = 2(140 + 4) + 5 + 7
= 300 mosm/L (isotonic).

The core of medical physiology (1) – 3rd edition Page 27


2- Diffusion
- It is expansion or passage of a substance through a cell membrane
down its chemical or electrical gradient, due to continuous random
movement of its molecules.
- Water follows osmotically active particles to inside or to outside the
cell, this affects volumes of body fluid compartments.
Fig 1.4: Diffusion

3- Gibbs Donnan equilibrium


- The presence of non-diffusible anions (protein and organic
phosphate) within the cell affects distribution of diffusible ions (both
anions and cations); it allows entry of diffusible cations (e.g. Na+) into
the cell and prevents entry of diffusible anions (e.g. Cl-).
Fig 1.5: Gibbs Donnan equilibrium

The core of medical physiology (1) – 3rd edition Page 28


- The concept was shown theoretically by Gibbs and confirmed
experimentally by Donnan (= known as Gibbs Donnan equilibrium).
- At equilibrium:
1- Total cations = total anions (on either side of the membrane)
2- The product of diffusible ions on one side equals the product of
diffusible ions on the other side of the membrane (This holds for
cations and anions of the same valence).
- Take this example of two solutions “a” and “b”; in which Na+ and Cl-
are diffusible cations and anions respectively; X- indicates non
diffusible anions.
Solution (a) Solution (b)
+
Na Na+
Cl- Cl-
X-
At equilibrium:
1- [Na+]a = [Cl-]a + [X-]a (i.e. cations in a = anions in a)
+ -
[Na ]b = [Cl ]b (i.e. cations in b = anions in b)
2- [Na+]a x [Cl-]a = [Na+]b x [Cl-]b
[Na+]a/ [Na+]b = [Cl-]b / [Cl-]a

From the above relationships:


 [Na+]a > [Cl-]a
(Cations on the side of X- are greater than anions on the same side)
 [Cl-]a < [Cl-]b
(Diffusible anions on the side of X- are less than on the other side)
 [Na+]a > [Na+]b
(Cations on the side of X- are greater than cations on the other side)

The core of medical physiology (1) – 3rd edition Page 29


 [Na+]a + [Cl-]a + [X-]a> [Na+]b + [Cl-]b
(There is greater number of ions on the side of X- than on the other
side).
Remember that:
 The greater number of particles in compartment “a” exerts an
osmotic effect resulting in swelling of this compartment.
 A similar effect occurs in body fluids, i.e. cells tend to undergo
swelling but this is prevented by the Na+/K+ pump.

Effects of Donnan equilibrium in the body:


1- Swelling of cells
The ICF contains higher concentration of non diffusible anions than
the ECF and therefore more particles. This may result in swelling of
the cells and eventually their rupture. However, swelling of cells is
prevented by the action of the Na+/K+ pump and other ion channels.
The Na+/K+ pump transports 3 sodium ions to outside of the cell and
2 potassium ions to inside; this decreases the total number of ions
inside the cell and prevents its swelling (see below).
2- Electrical difference across the cell membrane
The asymmetrical distribution of diffusible ions across the cell
membrane generates an electrical difference that can be determined
for each ion by the Nernst equation. In addition the asymmetrical
distribution of non diffusible anions participates in genesis of the
resting membrane potential (see chapter two).
3- Slight difference in concentration of ions between plasma and ISF
Since plasma contains higher concentration of protein than the
interstitial fluid (ISF), it contains slightly higher concentration of

The core of medical physiology (1) – 3rd edition Page 30


cations (like sodium and potassium) and lower concentration of
anions (like chloride and bicarbonate).
4- Slight difference in concentration of ions between the plasma and
the glomerular filtrate
At the kidney, proteins are not filtered out of the glomerular
capillaries. The filtrate is free of plasma protein. Therefore the
plasma contains slightly higher concentration of cations (like sodium
and potassium) and lower concentration of anions (like chloride and
bicarbonate) than in the filtrate.

Table 1.2: Differences between plasma and ISF in an adult male

Difference Plasma Interstitial fluid

Volume 5% of body weight 15% of body weight


Conc. Of Cations:
Sodium (Na+) 145 mmol/L 140 mmol/L
+
Potassium (K ) 4.1 mmol/L 4 mmol/L
Calcium (Ca+2) 2.5 mmol/L 2.1 mmol/L
Magnesium (Mg+2) 1 mmol/L 1 mmol/L
Total cations 152.6 mmol/L 147.1 mmol/L
Conc. Of Anions:
Chloride (Cl-) 105 mmol/L 110 mmol/L
Bicarbonate (HCO3-) 25 mmol/L 30 mmol/L
Phosphate (PO4-3) 1 mmol/L 1 mmol/L
Organic acids 5 mmol/L 5 mmol/L
Protein 16.6 mmol/L 1.1 mmol/L
Total anions 152.6 mmol/L 147.1 mmol/L

The core of medical physiology (1) – 3rd edition Page 31


4- The Sodium-Potassium Pump
- Found in all cells of the body.
- Transports sodium and potassium actively against their chemical
gradients (3 Na+ ions to outside and 2 K+ ions to inside the cell).
Fig 1.6: The sodium potassium pump

- It accounts for 20-45% of the total metabolic energy expended by


the cell.
- Consists of alpha and beta subunits extending through the cell
membrane. The beta subunit is a glycoprotein, whereas the alpha
subunit is a protein with extracellular binding sites for K + and
intracellular binding sites for Na+ and ATP.
Fig 1.7: Subunits of the sodium potassium pump

The core of medical physiology (1) – 3rd edition Page 32


- It is the alpha subunit that transports Na+ and K+; however,
separation of the two subunits inactivates the pump
- The Na+/K+ ATPase enzyme releases energy from ATP that’s used
in transport of 3 sodium ions to outside the cell and 2 potassium ions
to inside. This results in accumulation of negative charges inside the
cell (i.e. it is an electrogenic pump).
Functions of the Na+/K+ pump
- Participates in genesis of the resting membrane potential (i.e.
generates negative charges towards the inner side of the cell
membrane (see chapter two)).
- Prevents swelling and rupture of cells by removing excess sodium
ions to outside (see Donnan effect).
Regulation of the pump
- The pump is activated by accumulation of Na+ ions intracellularly.
- Activity is increased by:
o Insulin, Aldosterone and thyroid hormones
- Activity is inhibited by:
o Dopamine and digitalis

5- Starling's forces
- As mentioned above, movement of water across cell membranes
depends on osmosis. However, movement of water across the walls
of capillaries depends, in addition to that, on 4 primary forces (known
as Starling’s forces) that control fluid exchange between plasma and
interstitium.
- Although Starling’s forces act in all blood vessels, they cause fluid
exchange only in capillaries.

The core of medical physiology (1) – 3rd edition Page 33


- This is because the walls of capillaries, unlike arteries and veins,
are characterized by pores between the endothelial cells that allow
movement of water (for this reason capillaries are called “the
exchange vessels”).
- Starling forces include:
1- Capillary hydrostatic pressure (HPc)
- It is the pressure of plasma acting on the lateral wall of the
blood vessel
- For filtration (from plasma to ISF)
= 35 mmHg at the arteriolar end of capillaries
= 15 mmHg at the veniolar end of capillaries
2- Capillary oncotic pressure (OPc):
- The osmotic pressure of plasma proteins (also known as
colloid osmotic pressure or oncotic pressure)
- It is exerted mainly by albumin
- For absorption (from ISF to plasma)
= 25 mmHg throughout the capillaries (proteins are not filtered
and therefore their oncotic pressure is not changed)
3- Interstitial fluid hydrostatic pressure HPISF:
- Acts in the opposite side to HPC;(i.e. against filtration)
4- Interstitial fluid oncotic pressure OPISF:
- Acts in the opposite side to OPC; (i.e. against absorption)
Notes about osmotic pressure of proteins (Oncotic pressure)
 Oncotic pressure is mainly exerted by albumin because it is
the smallest plasma protein and therefore it has the highest
number of particles than other types of plasma proteins.
 Osmolarity depends on “number” not “size” of particles.

The core of medical physiology (1) – 3rd edition Page 34


- Calculation of filtration pressure:
- Both hydrostatic pressure of ISF and oncotic pressure of ISF are of
low magnitude, they act against each other and therefore they cancel
each other. That’s why they are not considered in calculation of the
filtration pressure.
- The filtration pressure is calculated by subtracting the capillary
oncotic pressure from the capillary hydrostatic pressure as follows:
o At the arteriolar end= (35-25)= +10 mmHg (i.e. net filtration)
o At the veniolar end= (15-25)= -10 mmHg (i.e. net absorption)

Fig 1.8: Starling's forces

- A filtration pressure of + 10 mmHg indicates that Starling’s forces


cause filtration of plasma at the arteriolar end of capillaries to the
interstitium. The filtered plasma carries nutrients to the surrounding
cells & then the fluid is absorbed back at the veniolar end with the
waste products.

The core of medical physiology (1) – 3rd edition Page 35


 About 90% of the filtered fluid at the arteriolar end of capillaries is
absorbed back to the capillaries at their veniolar end; the remaining
10% of the filtered fluid is also absorbed but by the lymphatics.
 The lymphatic vessels also absorb small amount of protein that
may escape out of the plasma to the interstitium.
 The lymphatics return this protein together with the 10% of the
filtered fluid back to the circulation at the neck where the main
lymphatic duct drains into the jugular vein. This keeps balance
between filtration and absorption of fluid at the capillaries.
 Disturbance of this balance between filtration and absorption may
result in accumulation of fluid in the interstitium causing edema.

Edema
- Edema is defined as abnormal accumulation of fluid in the
interstitial space. It is caused by many diseases through one or more
of the following mechanisms:
1- Increased capillary hydrostatic pressure (HPC)
- Some diseases may cause accumulation of blood in veins; thus
increasing the HPC at the veniolar end of capillaries.
- When the HPC becomes higher than the OPC , the return of the
filtered fluid to the capillaries is prevented causing edema.
- Examples include:
o Heart failure
 Left sided heart failure results in accumulation of blood in
the lung veins causing pulmonary edema whereas right
sided heart failure causes accumulation of blood in
systemic veins causing generalized edema.

The core of medical physiology (1) – 3rd edition Page 36


o Venous obstruction
 Results in accumulation of blood proximal to the site of
obstruction causing localized edema
Remember that:
 Oedema is not caused by hypertension because the
hydrostatic pressure of capillaries is not increased at the
veniolar end of capillaries, which is the site of absorption
 Oedema is not caused by arterial obstruction because this
decreases blood flow to the capillaries and therefore
decreases HPC

2- Decreased oncotic pressure (OPC)


- Some diseases may lower the level of proteins in the plasma. This
decreases OPC.
- The low OPC prevents absorption of the filtered fluid back to the
intravascular space causing edema.
- Examples include:
o Malnutrition
 Decreased protein intake (e.g. Kwashiorkor)
 Results in generalized edema
o Malabsorption
 Decreased absorption of protein (e.g. due to chronic
pancreatitis)
 Results in generalized edema
o Liver disease (chronic disease like liver cirrhosis)
 Decreased synthesis of plasma proteins
 Results in generalized edema

The core of medical physiology (1) – 3rd edition Page 37


o Renal disease (nephrotic or nephritic syndromes or renal failure)
 Loss of plasma proteins in urine
 Results in generalized edema

3- Lymphatic obstruction
- Obstruction of lymphatics results in accumulation of the fluid that’s
supposed to be absorbed by the lymphatics to be removed from
the interstitium. Therefore, it accumulates causing edema.
- The obstruction is caused by:
o Filaria (worms that live within the lymphatic vessels)
 Filariasis results in localized edema (very large swelling
proximal to the site of obstruction known as elephantiasis)
o Surgical removal of lymph nodes (which drain a site of cancer)
 This is done to prevent spread of secondaries from the
cancer
o It interrupts the lymph flow resulting in localized edema

4- Increased permeability of capillaries:


- Some diseases may increase the permeability of capillaries and
allow filtration of plasma proteins to the interstitium thus increasing
the OPISF which absorbs fluid to the outside causing edema.
- The capillary permeability is increased by:
o Inflammation
 The increased permeability is due to mediators of
inflammation released by white blood cells and the nearby
tissues
 Results in localized edema

The core of medical physiology (1) – 3rd edition Page 38


o Burn
 The increased permeability is due to the high temperature
 Results in localized edema
o Allergy
 The increased permeability is due to histamine which is
released by mast cells and basophils
 Results in localized or generalized edema

Types of edema:
- Edema can be classified into 2 types (by applying pressure on it in
one site using one finger “the thumb” against bone, for a minute):
1- Pitting edema
- The finger leaves a mark (a pit) on the skin
- The mark appears because the fluid escapes away from the
site of pressure and returns slowly
- Causes of pitting edema include:
 All causes of high capillary hydrostatic pressure
 All causes of low capillary oncotic pressure
2- Non pitting edema
- The finger does not leave a mark on the skin because the
escaped fluid returns rapidly.
- This is because it is attracted by proteins that are filtered to
the interstitium
- Causes of non pitting edema include:
 All causes of increased permeability
 All causes of lymphatic obstruction

The core of medical physiology (1) – 3rd edition Page 39


6- Abnormalities of water balance
 Water balance
- In normal physiological conditions, the body loses water in urine to
excrete waste products of metabolism and in addition to that, there is
insensible loss of water through the skin and in expired air.
- These water losses should continuously be replaced by water
intake to maintain normal water content of the body.
- On the other hand, if water intake is higher than the daily
requirement of the body, the excess water should be excreted.
- In summary: water loss should equal water intake (= water balance)

Fig 1.9: Water balance

- Abnormalities of this balance between water intake and output


cause disturbances in volumes and osmolarity of body fluid
compartments.

The core of medical physiology (1) – 3rd edition Page 40


Normal water intake occurs through:
o drinking 1.3L/day
o solid food 0.9L/day
o metabolism 0.3L/day
Net = 2.5L/day
Normal water output occurs through:
o urine 1.5 L/day
o stool 0.1 L/day
o sweating & insensible loss 0.9 L/day
Net= 2.5 L/day
Note
- The average water intake = the average water loss = 2.5 L/day
- The above values vary greatly in different physiological and
pathological conditions:

Examples of the physiological conditions:


1- Type of work
E.g. heavy work increases sweating= increased water loss
2- Exercise
E.g. strenuous exercise increases sweating and causes
hyperventilation (increases insensible water loss in expired
air) = increased water loss
3- Degree of water intake
Affects urine volume as follows:
- High water intake increases urine volume
- Low water intake decreases urine volume
 Remember that the minimum volume of urine required for
excretion of waste products of metabolism is 500 ml/day

The core of medical physiology (1) – 3rd edition Page 41


4- Variation in body temperature and environmental temperature
Both affect the amount of sweating, the rate of metabolism
and the rate of respiration (see chapter 4)

Examples of the pathological conditions:


1- Abnormal water intake through:
- increased metabolism (fever, hyperthyroidism)
- increased drinking (psychogenic polydypsia)
- excess intravenous fluids (fluid overload)
- complete water deprivation
2- Abnormal water loss through:
- vomiting
- diarrhea
- polyuria (diabetes mellitus, diabetes insipidus)
- excessive sweating (heat exhaustion)
- hyperventilation (metabolic acidosis)

Regulation of water balance


- As mentioned earlier, maintenance of constancy of ECF is the goal
of all systems in the body.
- Variation in water intake or water loss produces minor changes in
extracellular fluid volume and osmolarity. These changes stimulate
certain receptors (e.g. volume receptors & osmoreceptors) that
activate multiple regulatory mechanisms to restore back the
constancy of the ECF.

The core of medical physiology (1) – 3rd edition Page 42


- The regulatory mechanisms include:
1- Mechanisms for control of water intake
- act principally through control of thirst
2- Mechanisms for control of water loss
- act principally through control of urine volume

Control of water intake


- Under normal environmental and physiological conditions, the
amount of water gained by metabolism or solid food is almost
constant whereas the amount gained by drinking may be variable.
That’s why water intake is generally regulated through regulation of
drinking (= regulation of thirst). On the other hand, water intake
through solid food and metabolism is a non-regulatory component.
Thirst
- Defined as the subjective perception that provides the urge for
humans and animals to drink fluids.
- Also defined as the conscious desire for water.
- Regulated by thirst center located in the hypothalamus.
- There are 4 major stimuli to thirst:
 Angiotensin II: This is an octa-peptide hormone produced in the
plasma following release of renin enzyme from the "Juxta-
glomerular apparatus" in the kidney (see below).
- It acts directly on specific receptors located in
circumventricular organs in the brain (neural organs that lie
outside the blood-brain barrier) to stimulate thirst. The
neuronal pathway from the circumventricular organs to the
hypothalamus also uses angiotensin II as a neurotransmitter.

The core of medical physiology (1) – 3rd edition Page 43


 Hypertonicity: Small increases of 1-2% of the effective osmotic
pressure of plasma causes shrinkage of osmoreceptors in the
hypothalamus (due to osmosis from the osmoreceptors to the
ECF).
- Shrinkage of the osmoreceptors results in direct mechanical
stimulation of the thirst center because its dendrites which are
attached to the osmoreceptors are stretched by the shrinkage.
 Hypovolemia: The volume of ECF is sensed via volume
receptors located at the low pressure side of the circulation (i.e.
the venous side, at the junction of the right atrium and the vena
cava and at the entry of the pulmonary vein into the left atrium).
- Hypervolemia stretches these receptors which send inhibitory
impulses through the vagus nerve to inhibit thirst. On the other
hand, hypovolemia stimulates thirst through reduction of the
inhibitory discharge from the volume receptors to the thirst center.
 Hypotension: The blood pressure is sensed via baroreceptors
located at the high pressure side of the circulation (i.e. the arterial
side, at the carotid sinus and the aortic sinus which are found at
the bifurcation of the common carotid artery and the aortic arch
respectively).
- Hypertension stretches these receptors which send inhibitory
impulses through the vagus and the glossopharyngeal nerves to
inhibit the thirst center. On the other hand, hypotension stimulates
thirst through reduction of the inhibitory discharge from these
receptors to the thirst center.
Remember that:
 Thirst is one of the symptoms of dehydration and shock

The core of medical physiology (1) – 3rd edition Page 44


The hormone atrial natriuretic peptide (ANP) inhibits thirst!
ANP is a peptide hormone released by atria in response to
hypervolemia. It inhibits the effect of angiotensin II on thirst to
decrease ECF volume. It also decreases ECF volume through
inhibition of aldosterone action on the kidney, increasing the rate of
glomerular filtration and increasing the rate of sodium excretion in
urine (notice that loss of sodium is followed by loss of water).
Important note: Drinking stimulates mechanoreceptors in the mouth
and pharynx which provide input to the hypothalamus to attenuate
the sensation of thirst. This occurs before any reduction in plasma
tonicity; thus acting as a safeguard against over-ingestion of water.

Control of water loss


- Under normal environmental and physiological conditions, the
amount of water lost by sweating, respiration and through the skin is
almost constant whereas the amount lost by urine is variable. The
minimal volume of urine that can be excreted to eliminate the
metabolic waste products = 0.5 L/day. The maximal volume depends
on water intake.
- For this reason water loss is controlled through regulation of urine
volume. This involves the kidney and the hormones acting on it.
Renal function
- The functional unit of the kidney is the "nephron" which consists of:
glomerulus, for filtration & tubules: for reabsorption and secretion.
- About 180 L of fluid pass through the glomeruli of the kidney each
day. However, only 1.5 L is excreted in urine indicating that the renal
tubules reabsorb more than 99% of the filtered fluid.

The core of medical physiology (1) – 3rd edition Page 45


Fig 1.10: The nephron

- Reabsorption of water in the renal tubules occurs as follows:


- The PCT: reabsorbs 70% of the filtrate following sodium
reabsorption. The filtrate remains isotonic
- The loop of Henle: water reabsorption occurs in the thin
descending limb without sodium reabsorption. The filtrate becomes
hypertonic; however it becomes hypotonic at the end of the thick
ascending limb because it reabsorbs solutes without water.
- The DCT: relatively impermeable to water. About 5% of the
filtrate may be reabsorbed. The filtrate remains hypotonic.
- The CDs: Completely impermeable to water except at the
presence of ADH (see ADH below).

The core of medical physiology (1) – 3rd edition Page 46


Hormonal activity
 ADH
- Antidiuretic hormone, also known as vasopressin
- It is a nona-peptide hormone synthesized in the hypothalamus and
stored in the posterior pituitary gland
- It is released in response to the same major stimuli of thirst, through
similar mechanisms:
o Hyperosmolarity (detected by osmoreceptors located at the
hypothalamus. They cause mechanical stimulation of ADH)
o Hypovolemia (results in less stretch of the volume receptors at
the low pressure side of the circulation and therefore less
inhibition of ADH release)
o Hypotension (results in less stretch of the baroreceptors at the
high pressure side of the circulation and therefore less
inhibition of ADH release)
o Angiotensin II (stimulates ADH release, see below)
o Drugs (drugs that stimulate ADH release include barbiturates,
clofibrate, nicotine, acetylcholine and others)
Functions of ADH
- ADH acts on the collecting ducts in the kidney causing water
retention. It facilitates reabsorption of 7-13% of the filtrate
- When ADH level in plasma is high, it also causes vasoconstriction
resulting in elevation of the blood pressure
Abnormalities of ADH:
- Deficiency of ADH causes polyuria and excessive thirst due to
hypovolemia. Urine volume may reach up to 23 L/day. The condition
is known as diabetes insipidus (DI). It may result from a problem in

The core of medical physiology (1) – 3rd edition Page 47


the hypothalamus (neurogenic DI) or a problem in the renal receptors
for ADH (nephrogenic DI)
- Excessive ADH secretion causes reduction in urine volume,
hypertension and edema due to water retention. The condition is
known as syndrome of inappropriate ADH secretion (SIADH). It is
caused by many problems; these include head trauma, lung tumors,
pneumonia and pancreatitis.

 Aldosterone
- Steroid hormone synthesized in the adrenal cortex
- It is released in response to the following stimuli:
o Hyperkalemia
Directly stimulates aldosterone release from the adrenal cortex.
o High level of ACTH
The adrenocorticotrophic hormone (ACTH) is released by the
anterior pituitary gland to stimulate secretion of cortisol (not
aldosterone) by the adrenal cortex. But, in high levels (due to
endocrine abnormalities) it also stimulates release of aldosterone.
o The renin-angiotensin-aldosterone system
In this system, renin enzyme which is produced by the Juxta-
glomerular apparatus in the kidney results in formation of
angiotensin II. The later stimulates aldosterone secretion from the
adrenal cortex.
The Juxta-glomerular apparatus (JGA) is formed by:
- Cells of the afferent arteriole (juxta-glomerular cells)
- Cells of the DCT (macula densa cells)
- Lacis cells (= extra-glomerular mesangial cells)

The core of medical physiology (1) – 3rd edition Page 48


Fig 1.11: The juxta-glomerular apparatus (JGA)

- This apparatus (specifically the juxta-glomerular cells) secretes


renin in response to one of the following stimuli;
o Hyponatremia (low [Na])
o Renal ischemia (e.g. due to hypotension or hypovolemia)
o Sympathetic stimulation
- Renin acts on a plasma peptide known as angiotensinogen (14
amino acids) produced by the liver to form angiotensin I (10 aa).
- Angiotensin I is converted to angiotensin II (8 aa) by the action of
angiotensin converting enzyme (ACE).
- This enzyme is produced by the pulmonary endothelial cells. It is
also released to the circulation by lung macrophages.

The core of medical physiology (1) – 3rd edition Page 49


- Angiotensin II increases ECF volume & pressure because it causes:
1- Vasoconstriction
2- Stimulation of the sympathetic (this stimulates release of
renin and renin forms angiotensin I and then angiotensin II
and the cycle repeats itself in a positive feedback mechanism)
3- Stimulation of thirst
4-Stimulation of ADH
5- Stimulation of aldosterone
- Aldosterone acts on the DCT and CDs in the kidney to stimulate
retention of sodium and secretion of potassium. Water follows
sodium resulting in increased ECF volume and pressure.
- Unlike excessive ADH secretion, excessive aldosterone secretion
due to adrenal tumors results in hypervolemia and hypertension but it
does not cause edema. The explanation involves ANP as follows:
 Hypervolemia caused by sodium and water retention
stimulates release of ANP from the atria. ANP acts in the
kidney to increase sodium and water excretion. This prevents
development of edema. This phenomenon is known as
“aldosterone escape phenomenon”.
 Atrial natriuretic peptide (ANP)
- A peptide hormone released by atria in response to hypervolemia.
- It reduces ECF volume through the following effects:
o Inhibition of the effect of angiotensin II on thirst
o Inhibition of the effect of aldosterone on the kidney
o Increasing the rate of glomerular filtration
o Increasing excretion of sodium in urine (= natriuresis; notice
that loss of sodium is followed by loss of water).)

The core of medical physiology (1) – 3rd edition Page 50


QUESTIONS FOR SELF ASSESSMENT-2 (BEST OF FIVE)

1. The percentage of total body water is higher in:


a. females compared to equivalent males
b. old subjects compared to children
c. infants compared to neonates
d. thin subjects compared to obese ones
e. male children compared to female children
2. Using the indicator dilution method, ECF volume is measured by:
a. heavy water
b. RISA
c. inulin
d. heavy water and RISA
e. Evan’s blue dye
3. The volume of distribution of substance A in an average adult
male was found to be 3.5 litres. This substance could be:
a. distributed to all body fluid compartments
b. highly exchangeable between plasma and interstitium
c. tightly bound to plasma proteins
d. rapidly metabolised by cells
e. all the above are correct
4. The ECF differs from the ICF because it contains:
a. less concentration of sodium
b. lower pH
c. higher temperature
d. osmolarity of about 300 mosm/L
e. higher concentration of chloride
5. The interstitial fluid:
a. constitutes about 5% of total body weight in adults
b. temperature is less than that of the plasma
c. osmolarity is determined by Cl- concentration
d. volume can be measured by using inulin and manitol
e. contains slightly higher concentration of Cl- than plasma
6. In a twenty years old man, the following volumes of distribution
are obtained using the indicator dilution method: inulin = 14L and
Evan’s blue dye = 3L; which of the following is not true:
a- intravascular fluid volume is about 3L
b- ECF volume is about 14L
c- ICF volume cannot be calculated from this data
d- ISF volume cannot be calculated from this data
e- blood volume cannot be calculated from this data

The core of medical physiology (1) – 3rd edition Page 51


7. The osmolarity of plasma is a function of:
a. calcium concentration
b. protein concentration
c. sodium concentration
d. urea concentration
e. glucose concentration
8. The concentration of calcium in the extracellular fluid is about:
a. 10 mg/dL
b. 5 mmol/L
c. 1.5 meq/L
d. 1.0 %
e. 0
9. Filtration of fluid from plasma to interstitium is increased by:
a. increased capillary oncotic pressure
b. increased capillary hydrostatic pressure
c. increased interstitial hydrostatic pressure
d. decreased interstitial oncotic pressure
e. lymphatic obstruction
10. Edema due to high capillary permeability results from:
a. filariasis
b. renal failure
c. burn
d. hepatic failure
e. deep vein thrombosis
11. Gibbs Donnan equilibrium is responsible for:
a. higher concentration of Na+ in ICF than in ECF
b. lower concentration of K+ in plasma than in ISF
c. slightly higher concentration of HCO3- in ISF than in plasma
d. lower concentration of Cl- in intracellular fluid than in plasma
e. higher volume of interstitial fluid than plasma
12. The ECF/ICF volume ratio is highest in:
a- lean males
b- obese males
c- male children
d- female children
e- newborns

Question 1. 2. 3. 4. 5. 6. 7. 8. 9. 10. 11. 12.


Answer d c c e e d c a b c c e

The core of medical physiology (1) – 3rd edition Page 52


CHAPTER 2
EXCITABLE TISSUES
- Nerves and muscles are said to be excitable because they are
capable of generating electrical signals known as action potentials.
- In addition to action potentials, excitable tissues are also
characterized by resting membrane potentials. However, unlike
action potentials, resting membrane potentials are found in almost all
cells of the body.

THE RESTING MEMBRANE POTENTIAL (RMP)


- It is the difference in electrical potential between the interior and the
exterior of cell membranes at rest, with the interior being more
negative relative to the exterior.
- The cell membranes are said to be “polarized”; i.e. negative inside
and positive outside.

Fig 2.1: The polarized cell membrane

- The difference in potential across the cell membrane can be


measured by special devices (e.g. the Cathode Ray Oscilloscope),
using electrodes placed in the two sides of the cell membrane.
Fig 2.2: Measurement of the resting membrane potential

The core of medical physiology (1) – 3rd edition Page 53


- The magnitude of the RMP differs in different types of cells, for
example:
In a skeletal muscle cell = - 90 mv
In a cardiac muscle cell = - 90 mv
In a nerve cell = - 70 mv
Causes of the RMP:
a- Potassium efflux
b- The Sodium- potassium pump
c- ICF non diffusible anions

a- Potassium efflux
- This is the major cause of RMP.
- The cell membrane is more permeable to potassium than sodium
(because the hydrated atom of potassium is smaller than the hydrated
atom of sodium).
- Therefore potassium diffuses to the outside down its concentration
gradient through potassium leak channels.
Fig 2.3: Potassium efflux

- For each atom of potassium that diffuses to the outside, a negative


charge is generated inside and a positive charge is generated outside
the cell membrane.

The core of medical physiology (1) – 3rd edition Page 54


- The positive charges outside the cell membrane rebel the potassium
ions, forcing them back to the inside through the leak channels (down
their electrical gradient from the positive side to the negative side).
However, net potassium efflux continues until the chemical gradient
which drives potassium to the outside is equal to the electrical
gradient that drives potassium to the inside (= stage of equilibrium).
b- The Sodium- potassium pump
- An electrogenic pump that pumps 3 sodium ions to outside and 2
potassium ions to inside, thus generating negative charges within the
cell.
Fig 2.4:The sodium potassium pump

- Sodium and potassium ions are transported actively against their


concentration gradients, using energy released by hydrolysis of ATP
(see chapter 1).

c- ICF non diffusible anions

- Presence of non diffusible anions inside the cell (protein and organic
phosphate) contributes to the genesis of RMP by increasing the
number of negative charges inside the cell (= very low contribution).

The core of medical physiology (1) – 3rd edition Page 55


The equilibrium potential:

- The equilibrium potential (E) for an ion gives an idea about its role in
genesis of the resting membrane potential. Here the ion is placed in a
medium similar to ECF and allowed to pass into or out of the cell to
reach equilibrium, without contribution of other ions. The membrane
potential generated due to difference in concentrations of that ion in
ECF and ICF is called the equilibrium potential. However, it does not
occur normally because of the contribution of other ions.

- The equilibrium potential can be measured by the Nernst equation


as follows:

 for cations: E = 61 log [cation]ECF/[cation]ICF


 for anions E = 61 log [anion]ICF/[anion]ECF
- Examples:

 E for K+ = 61 log 4/140 = - 90 mv (close to the RMP, indicating


a major role of K+ in the genesis of RMP).
 E for Na+ = 61 log 150/15 = + 61 mv (very far from the RMP,
indicating a minor role of Na+ in genesis of the RMP).
 E for Cl- = 61 log 110/8 = - 70 mv (However, there is no
evidence that Cl- has an active transport similar to Na+ and K+;
so it does not contribute to the RMP).
- The Goldman equation can be used for direct calculation of the
RMP. It includes the effects of the major ions and their permeabilities
(P) across the cell membrane; as follows:
RMP (mv) = 61 log PK [K]ECF + P Na [Na]ECF + PCl [Cl]ICF
PK [K]ICF + P Na [Na]ICF + PCl [Cl]ECF

The core of medical physiology (1) – 3rd edition Page 56


THE ACTION POTENTIAL (AP)
- Occurs only in excitable tissues (nerve & muscle).
- Involves alterations in the membrane potential (depolarization and
repolarization) following sufficient mechanical, electrical or chemical
stimulation.
- The action potential can be recorded by special devices (e.g. the
cathode ray oscilloscope).

- The recorded action potential can be: Monophasic in which the


recording electrodes are placed on the surface of cell membrane; or
Biphasic in which they are placed on the two sides of cell membrane.

Components of the action potential

1- Stimulus artifact

- Occurs at the time of stimulation (marks the point of stimulation)

- Due to current leakage from the stimulus electrode

2- Latent period

- The isopotential interval from the point of stimulation to the start of


action potential.

- Can be used for measurement of the conduction velocity (CV) in an


axon. Conduction velocity = (the distance between the stimulating
and recording electrodes) divided by (the latent period).

3- Threshold

- The voltage at which the "fast voltage gated sodium channels"


open. If not reached, action potential never occurs.

The core of medical physiology (1) – 3rd edition Page 57


- It is usually about 15 mv less than the resting membrane potential
(e.g. -75 mv when the RMP is -90 mv).

4- Depolarization

- Due to sodium influx through the "fast voltage gated sodium


channels".

- The membrane potential moves towards the equilibrium potential


for sodium (+60 mv) but it does not reach it because opening of the
sodium channels is short lived (the channels rapidly become
inactivated and will never open again unless the membrane potential
returns back to the resting level); (see the refractory period below).

5- Repolarization

- Due to potassium efflux through "voltage gated potassium


channels".

- Opening of the "voltage gated potassium channels" is slower than


sodium channels; that’s why potassium efflux occurs after sodium
influx. Opening is also more prolonged than sodium channels and
this explains the occurrence of after-hyperpolarization (see below).

- Additional factors that facilitate repolarization are the inactivation of


the "fast voltage gated sodium channels" and the reversal of the
membrane potential (becomes positive inside); thus limiting further
sodium influx.

6- After- depolarization

- The slower fall in the rate of repolarization.

- occurs when repolarization is 70% completed.

The core of medical physiology (1) – 3rd edition Page 58


7- After-hyperpolarization

- Slight overshooting in the hyperpolarizing direction, due to excess


potassium efflux through the "voltage gated potassium channels",
and then restoration of the membrane potential to the resting level
(due to the activity of the Na/K pump).

Fig 2.5: The action potential

Characteristics of action potentials


1- All or none law
- Sub threshold stimuli can never generate an action potential
whereas threshold and supra threshold stimuli can generate action
potentials of the same magnitude (i.e. full action potential).
- This indicates that action potentials either occur in a full form or do
not occur at all (i.e. all or none).

The core of medical physiology (1) – 3rd edition Page 59


2- Refractory period
- Period of time during which a cell membrane is refractory to
excitation, because it is already in a state of excitation. During this
period the "fast voltage gated sodium channels" are inactivated
- The refractory period is divided into:
 Absolute refractory period:
o Action potential can never be generated even with supra-
threshold stimulation.
o Its duration from the firing level to the end of one third of
repolarization.
 Relative refractory period:
o Action potential can not be generated by ordinary stimulation
but it can be generated by supra-threshold stimulation.
o Its duration from the end of absolute refractory period to the
end of repolarization (i.e. to the start of after-depolarization).
Fig 2.6: The refractory period

The core of medical physiology (1) – 3rd edition Page 60


3- Conduction
- Action potential can be propagated (conducted) through axons of
neurons.
- Conduction is an active, self propagating process that occurs with
no change in amplitude or velocity of the action potential.
- There are two types of conduction:
 Conduction in unmylinated nerves (continuous conduction)
 Conduction in myelinated nerves (saltatory conduction)
(jumping of action potential from a node of Ranvier to
another).
Fig 2.7: Types of electrical conduction

_______________________________________________________

The core of medical physiology (1) – 3rd edition Page 61


- The velocity of conduction depends on:
 Degree of myelination:
- Faster in thick myelinated > thin myelinated > un-myelinated
 Distance between nodes of Ranvier:
- Faster when the distance is increased
 Diameter of the axon:
- Faster when the diameter is larger

The compound action potential


- The above description of action potential refers to action potential
that’s recorded from a single axon. It differs from an action potential
recorded from a peripheral nerve (which contains many axons).
- The action potential that’s recorded from a peripheral nerve is
called the compound action potential and it is characterized by
multiple peaks.
Fig 2.8: The compound action potential

- It represents algebraic summation of action potentials of many


axons in the nerve. The multiple peaks appear because a peripheral
nerve contains different types of fibers with different sizes, thresholds
and peaks.

The core of medical physiology (1) – 3rd edition Page 62


Effects of ECF electrolyte disturbances on excitability
Disturbances in sodium ions
- Have little effect on the resting membrane potential (RMP).
- Hyponatremia decreases the amplitude of the action potential.
Disturbances in potassium ions
- Affect the resting membrane potential because potassium efflux is
the main cause of the resting membrane potential.
- Hypokalemia causes hyperpolarization (because it allows more K
efflux and therefore the resting membrane potential gets away from
the threshold; this decreases excitability.

Fig 2.9: Hyperpolarization caused by excessive K efflux

- Mild to moderate hyperkalemia limits potassium efflux and therefore


the RMP is elevated to become closer to the threshold, this
increases excitability. However, severe hyperkalemia elevates the
RMP to the level of the threshold and therefore it inhibits excitability
(the cell can not respond to stimulation since its RMP is not below
the threshold; that’s why hyperkalemia prevents excitation of the
heart and stops it in diastole).
The core of medical physiology (1) – 3rd edition Page 63
Fig 2.10: Effect of hyperkalemia on the RMP

Disturbances in calcium ions


- Calcium ions guard the sodium leak channels, thus preventing
sodium influx through these channels during the resting state.
- In hypocalcemia sodium ions enter the cells because the leak
channels are not guarded, this elevates the RMP and makes the
cells more excitable (for this reason hypocalcaemia is characterized
by high excitability in neuromuscular junctions resulting in
involuntary contraction of some muscles (= this is known as tetany).
- Hypercalcemia has an opposite effect; it prevents influx of sodium
ions through their leak channels; therefore, it stabilizes cell
membranes and decreases excitability.
- Severe hypercalcemia may interfere with the normal function of
nerves and may result in coma.
- Hypercalcemia increases contractility of cardiac muscle but not
skeletal muscle; that’s because calcium ions in ECF enter cardiac
muscle cells and increase contraction (they do not enter skeletal
muscle cells). Severe hypercalcemia stops the heart in systole.

The core of medical physiology (1) – 3rd edition Page 64


Action potential of the cardiac muscle
Fig 2.11:

- There are two types of muscle in the heart:


 The cardiac muscle proper: whose AP is characterized by the
plateau phase (due to calcium influx). The plateau prolongs the
refractory period.
 The conductive system: whose AP is characterized by the
prepotential (unstable resting membrane potential, mainly due to
slow potassium efflux).

NERVE
- Nerves are distributed throughout the body to form the nervous
system. The nervous system is subdivided into:
 Central nervous system (CNS = the brain and the spinal cord).
 Peripheral nervous system (PNS = the spinal and cranial nerves).
- Each nerve consists of many nerve cells (neurons).

The core of medical physiology (1) – 3rd edition Page 65


- A neuron is the main functional cell in the nervous system.
- It consists of:
o Cell body (or soma)
o Dendrites
o Axon
Fig 2.12: The neuron

- There are about 100 billion neurons in the CNS and about 10-50
times this number glial cell (neuroglia).
- - Neuroglia (= non-excitable cells) are 3 types in the CNS:
o Microglia
- Phagocytic cells in the brain (resemble tissue
macrophages)
o Astrocytes (= macrglia)
- Provide a supportive matrix around the neurons
- Form part of the blood brain barrier (BBB)
- Maintain stable ECF concentration of ions (by taking
up K+)
- Two types: fibrous astrocytes (in the white matter) and
protoplasmic astrocytes (in the gray matter)
o Oligodendrocytes (= macroglia)
- Form myelin sheath around axons in the CNS

The core of medical physiology (1) – 3rd edition Page 66


- There is one type of glia in the peripheral nervous system (PNS):
o Schwann cells
- The only glial cells in the PNS
- Form myelin sheath around axons in the PNS
- Each cell wraps its membrane around axons up to
100 times to form a myelin sheath
Fig 2.13: Myelination

Types of nerve fibers


- Nerve fibers are divided into 3 groups: (A, B and C).
- These groups differ in the axon size and degree of myelination.
- The A group is subdivided into: (A, A, A and A).
Group A
 Thick myelinated = The fastest (especially the A type)
 Has the largest diameter
 Carries touch, proprioception and motor impulses
 Most susceptible to pressure
 Least susceptible to local anesthetics, that’s why touch
sensation, which is carried by type A fibers, is not completely
depressed by local anesthetics

The core of medical physiology (1) – 3rd edition Page 67


 Conduction velocity of type A fibers:
- A = 70-120 m/s
- A = 30-70 m/s
- A = 15-30 m/s
- A= 12-30 m/s
Group B
 Thin myelinated
 Found in the preganglionic autonomic neurons
 Most susceptible to hypoxia than to pressure or local
anesthetics
 Conduction velocity= 3-15 m/s
Group C
 Un-myelinated
 Mainly carries pain and cold sensations
 Also found in the postganglionic sympathetic neurons
 Most susceptible to local anesthetics (that’s why local
anesthetics depress pain sensation which is carried through
type C fibers)
 Least susceptible to hypoxia and pressure
 Conduction velocity= 0.5-2 m/s
Important note
- There is another method of nerve fiber classification (Ia, Ib, II, III
and IV) However, it is rather confusing.
- Here Ia, Ib, II and III are equivalent to the subdivisions of group A
fibers whereas IV is equivalent to type C fibers.
- This classification is used to describe afferents of the muscle
spindle & the Golgi tendon organ (see the motor system).

The core of medical physiology (1) – 3rd edition Page 68


SYNAPTIC TRANSMISSION
THE SYNAPSE
- It is a junction between two neurons. It allows transmission of
electrical impulses from one neuron to another.
- The transmission is called "chemical transmission" because a
chemical substance "a neurotransmitter" is released by the first
neuron (the pre-synaptic neuron) to bind receptors in the second one
(the post-synaptic neuron). However, direct electrical transmission
can occur through special synapses.
- Unlike electrical transmission through the axons, chemical
transmission is uni-directional (i.e. allows conduction in one way only;
from the pre-synaptic neuron to the post-synaptic neuron).
- The total number of synapses in the CNS is very large; a single
axon may divide to form over 2000 synaptic endings.
- Most synapses occur between the axon terminals of the pre-
synaptic neuron and the cell body or dendrites of the post-synaptic
neuron (axo-somatic and axo-dendritic connections). However, other
types of connection may occur (e.g. axo-axonal).
Fig 2.14: Synapses

The core of medical physiology (1) – 3rd edition Page 69


General structure of the synapse
Fig 2.15:

The presynaptic neuron:


 Releases a neurotransmitter (NT) through the presynaptic
membrane
 The NT is synthesized in the cell body, packed into vesicles in
the Golgi apparatus and then transported through the axon to
be stored at the terminal end of the axon.
 The terminal end of the axon is slightly dilated for storage of
the NT. It is called the synaptic knob or button.
The postsynaptic neuron:
- Contains receptors for the NT.
- The receptors are located on the postsynaptic membrane.

The core of medical physiology (1) – 3rd edition Page 70


The synaptic cleft:
- The area that separates the pre and post synaptic neurons
- It is about 20-40 nm wide
- The NT has to cross this area to bind its receptors on the
postsynaptic membrane
- It may contain an enzyme for hydrolysis of the NT and
therefore terminates its action (e.g. acetylcholine esterase
enzyme for hydrolysis of acetylcholine).

Steps of neurotransmitter release to the synaptic cleft


1- Arrival of the action potential to the terminal end of the axon
2- Opening of voltage-gated Ca++ channels, allowing Ca++ influx
3- Ca++ stimulates release of the neurotransmitter (NT) from the
vesicles by exocytosis (it increases movement of the vesicles
to fuse with the pre-synaptic membrane and facilitates the
process of exocytosis)
4- The NT within the vesicles is released to the synaptic cleft.
- Note: Ca++ is then removed from the knob by "Ca2+/Na+ antiport "

Fate of the neurotransmitter at the synaptic cleft


 Neurotransmitters at the synaptic cleft may:
1- Combine with receptors on the post-synaptic membrane (to
exert their effects)
2- Be hydrolyzed by an enzyme in the synaptic cleft
3- Return back to the synaptic knob by endocytosis
4- Diffuse to the plasma (where it is catabolized there or in other
tissues)

The core of medical physiology (1) – 3rd edition Page 71


Fig 2.16: Fate of neurotransmitters at the synapse

Notes to remember:
 Not all of these options are applicable for all types of
neurotransmitters.
 For example: acetylcholine is hydrolyzed in the cleft by
acetylcholine-esterase into acetate and choline. Therefore it
does not return back to the synaptic knob by endocytosis;
however, its metabolite choline returns back.
 Another example: noradrenaline is not hydrolyzed in the cleft.
Therefore it returns back to the synaptic knob by endocytosis
and packed again into vesicles to be released later (recycling).
 Because it is not hydrolyzed in the cleft, the amount of
noradrenaline that diffuses to plasma is higher than that of
acetylcholine.

The core of medical physiology (1) – 3rd edition Page 72


ELECTRICAL EVENTS IN THE POSTSYNAPTIC NEURON
- When a neurotransmitter combines with its specific receptor, it
results in opening or closure of some "ligand gated channels" on the
postsynaptic membrane.
- This changes the permeability of the postsynaptic membrane to
specific ions and results in a postsynaptic potential.
- The postsynaptic potential is a local potential that can be excitatory
or inhibitory (depending on which type of ion channels is opened).
Excitatory post synaptic potential (EPSP):
 This is a depolarizing potential that increases excitability of the
post-synaptic neuron
 It does not cause a response because it does not reach the
threshold
 It is caused by an excitatory neurotransmitter that opens:
- sodium channels causing sodium influx
- or calcium channels causing calcium influx
Inhibitory post synaptic potential (IPSP):
 This is a hyperpolarizing potential that decreases excitability of
the post-synaptic neuron
 It is caused by an inhibitory neurotransmitter that opens
potassium or chloride channels causing potassium efflux or
chloride influx respectively or closes: sodium or calcium
channels preventing their influx.
Summation of local potentials:
- For an EPSP to reach a threshold and cause a response, it should
summate with other EPS potentials.
- There are two types of summation: spatial and temporal.

The core of medical physiology (1) – 3rd edition Page 73


Spatial summation: Occurs when multiple EPSPs arrive
simultaneously, at the same time, on the post-synaptic neuron. The
potentials when added together reach the threshold and produce an
action potential.
Temporal summation: Occurs when a single synaptic knob is
stimulated repeatedly to produce successive EPSPs. These when
added together reach the threshold and produce an action potential.

Fig 2.17: Summation of local potentials

Action potential in the post-synaptic neuron


- Summation of several EPSPs generates an action potential in the
post-synaptic neuron.
- The action potential starts in the generator zone which is situated
at the origin of the axon (axon hillock).
- It is more excitable than the rest of the axon because:
 It contains higher number of voltage-gated Na+ channels
 Its threshold is more near to the RMP (-59 mV)

The core of medical physiology (1) – 3rd edition Page 74


Inhibition and facilitation at synapses
Direct inhibition
 Occurs directly by releasing an inhibitory NT from a
presynaptic neuron into a postsynaptic neuron.
 This direct inhibition on the postsynaptic neuron is called
postsynaptic inhibition.

Fig 2.18: Direct inhibition

Indirect inhibition
 Occurs indirectly on the neuron
 Has many forms; for example:
o it follows a previous discharge on the postsynaptic neuron (=
here the already excited postsynaptic cell is in a refractory
period or in after-hyperpolarization (Periods of low excitability)
o it occurs in a postsynaptic neuron if the release of the
excitatory NT coming from its presynaptic neuron is prevented
by direct inhibition from another neuron (this is known as
presynaptic inhibition)

The core of medical physiology (1) – 3rd edition Page 75


Fig 2.19: Indirect inhibition

Properties of synapses
a- Transmission is uni-directional
- From the presynaptic neuron to the postsynaptic neuron.
b- Synaptic delay
- The minimum time required for chemical conduction from a synaptic
knob to its postsynaptic neuron is a bout 0.5 ms.
- Measurement of synaptic delay within the CNS gives information
about the number of synapses in a pathway. For example if the delay
is about 1.2 ms, the number of synapses is probably two.
c- Synaptic fatigue
- Failure or decrease in frequency of conduction in a synapse
following repetitive stimulation.
- Due to exhaustion of the NT (release of all vesicles at the synaptic
knob) or inactivation of the receptors at the postsynaptic membrane.
d- Post-tetanic facilitation
- Increased frequency of conduction in a synapse following repeated
stimulation.
- Caused by increased availability of calcium in the synaptic knob
due to repeated stimulation.

The core of medical physiology (1) – 3rd edition Page 76


e- Convergence and divergence
- Most of the inputs to the postsynaptic neurons are multiple
(convergence).
- Most of the outputs from the presynaptic neurons are multiple
(divergence)
Fig 2.20: Convergence and divergence

The Neuromuscular junction (NMJ)


- Special type of synapse (between a neuron and a muscle cell).
- The muscle cell membrane that receives the terminal end of the
neuron is known as the motor end plate. It is characterized by
multiple invaginations (junctional folds) that increase its surface area.
- Transmission through the NMJ is similar to chemical transmission
through other synapses.
- The neurotransmitter is acetylcholine and the receptors are
nicotinic.
- The local potential produced by binding of acetylcholine to its
nicotinic receptors at the NMJ is called the end plate potential.
- The end plate potential is always excitatory (caused by sodium
influx) and it is regularly capable of generating an action potential
(the high amount of acetylcholine released in the cleft is 10 times
sufficient for generation of an action potential).

The core of medical physiology (1) – 3rd edition Page 77


Fig 2.21: The neuromuscular junction

Factors affecting neuromuscular transmission


 Autoimmune destruction of acetylcholine receptors at the NMJ (=
Myasthenia gravis): characterized by weak skeletal muscles.
 Autoimmune destruction of calcium channels at the synaptic knob
of the NMJ (= Lambert-Eaton syndrome): similar to Myasthenia
but unlike it, the weakness of skeletal muscles is improved by
repeated contractions, due to increased Ca++ level in the knob.
 Blockers of acetylcholine receptors (e.g. curare and
succinylcholine): cause paralysis of skeletal muscles.
 Snake venoms: block acetylcholine receptors causing paralysis.
 Organophosphorus compounds (OPC e.g. parathion) & nerve
agents (e.g. Sarin, Tabun and VX): inhibit acetylcholinesterase to
allow acetylcholine to act for prolonged periods in both nicotinic
receptors (causing fasciculations, tachycardia & mydriasis) and
muscarinic receptors (causing bradycardia, bronchospasm,
salivation, lacrimation, diarrhea and vomiting). OPC are used as
insecticides whereas nerve agents are military weapons.
 Botulinum toxin: released by bacteria (Clostridium botulinum):
Prevents release of acetylcholine from neurons causing paralysis.

The core of medical physiology (1) – 3rd edition Page 78


MUSCLE
Types of muscle
- There are 3 types of muscle:
 Skeletal muscle
- Striated
- Under voluntary control (somatic nervous system)
- Has no anatomical connection between fibers
- Does not contract in the absence of external stimulation
Fig 2.22:

 Cardiac muscle
- Striated
- Under involuntary control (autonomic nervous system)
- There are connections between fibers (gap junctions and
intercalated discs)
- Contracts rhythmically in the absence of external stimulation
Fig 2.23:

The core of medical physiology (1) – 3rd edition Page 79


 Smooth muscle
- Non striated
- Under involuntary control (autonomic nervous system)
- Connections between fibers may be present (e.g. smooth
muscle in viscera) or absent (e.g. smooth muscle in the eye).
Fig 2.24:

Structure of skeletal muscle


- Each muscle consists of bundles of fibers
Fig 2.25:

- Each fiber is a single cell, composed of myofibrils


Fig 2.26:

The core of medical physiology (1) – 3rd edition Page 80


- Each myofibril consists of filaments, the filaments are:
 Thick filaments: myosin II
 Thin filaments: actin, tropomyosin and troponins (I, T & C)
 Others: actinin and titin
Fig 2.27:

Striations
- Appearance of alternate light and dark bands in skeletal or cardiac
muscle fibers when examined under direct polarized light, using the
microscope.
- Are due to differences in refractive indexes of thick & thin filaments
- The light area (I band):
 Is isotropic to polarized light
 contains actin only
 divided into two halves by the Z line
- The dark area (A band):
 is anisotropic to polarized light
 contains myosin + ends of actin
 contains lighter area in the middle (H band) that contains
myosin only and divided by the M line

The core of medical physiology (1) – 3rd edition Page 81


- The area between two Z lines is called sarcomere
- The sarcomere is the functional unit of muscles. It contains an A
band + two halves of I band.
Fig 2.28:

The thick filaments:


Myosin II
- Each has 2 heavy chains and 4 light chains forming a tail and 2
globular heads.
- Each head of myosin has:
 Actin binding site
 ATPase activity
Fig 2.29:

The core of medical physiology (1) – 3rd edition Page 82


The thin filaments :
a- Actin:
- Consists of two chains that form double helix
- Has active sites that act as targets for myosin heads
Fig 2.30:

b- Tropomyosin:
- Lie in the groove between the two filaments of actin
- Hides the actin active sites during relaxation of muscle
- This prevents binding of myosin heads to actin
Fig 2.31:

The core of medical physiology (1) – 3rd edition Page 83


c- Troponins:
- Troponin T: binds other troponin units to tropomyosin
- Troponin I: inhibits interaction between actin and myosin
- Troponin C: binds calcium which initiates contraction
Fig 2.32:

The sarcotubular system


- Membranous structures in the muscle fiber
- Made up of: T tubules & sarcoplasmic reticulum
Fig 2.33:

The core of medical physiology (1) – 3rd edition Page 84


The T tubules:
- Are transverse tubules that arise as invaginations from the cell
membrane.
- Located at the A-I junctions in skeletal muscles (= two per cell); and
at the Z lines in cardiac muscles (one per cell).
- Inside the cell they are perforated by the myofibrils.
- They transmit action potentials from the sarcolemma to the interior
of the cell.
The sarcoplasmic reticulum:
- Forms irregular curtain around the myofibrils.
- Has an enlarged terminal on each side of the T tubule "the
sarcoplasmic cisternae". These form with the T tubule a triad (T
tubule + 2 cisternae on each side).
- It stores calcium to be released to the sarcoplasm when an action
potential arrives through the T tubule.

Molecular basis of contraction (Sliding theory)


- Contraction occurs by sliding of the thin filaments over the thick
filaments, thus shortening the sarcomers.
Steps of muscle contraction:
 Depolarization of the sarcolemma (by an action potential
coming from the neuromuscular junction).
 Transmission of the action potential to interior of the muscle
cell via the T tubules.
 The passage of the depolarization wave through the T tubules
induces opening of dihydropyridine receptors in the membrane
of the T tubules.

The core of medical physiology (1) – 3rd edition Page 85


 The dihydropyridine receptors:
o in the cardiac muscle they allow influx of calcium & this
triggers release of stored calcium in the terminal cisterns.
o in skeletal muscle fibers they act as sensors that detect
arrival of the depolarizing wave & trigger release of stored
calcium.
 Release of stored calcium from the terminal cisterns to the
sarcoplasm occurs via the ryanodine receptors
 Calcium in the sarcoplasm binds to troponin C
 This results in:
o Weakness of the binding of troponin I to actin
o Conformational changes in tropomyosin
 Tropomyosin moves laterally to uncover the actin active sites
 Myosin heads bind the uncovered actin active sites
 ATP is hydrolyzed and the energy released by hydrolysis
causes flexion of myosin heads inwards.
 This causes sliding of actin on myosin resulting in shortening
of the sarcomers (= contraction).
Fig 2.34:

The core of medical physiology (1) – 3rd edition Page 86


Notes to remember:
- The process by which depolarization of the muscle cell results in
contraction is known as excitation-contraction coupling.
- Seven active sites in actin are uncovered for each molecule of
troponin C that binds a calcium ion.
- Contraction of skeletal muscle depends on release of stored
calcium from the sarcoplasmic reticulum whereas contraction of
cardiac and smooth muscles requires influx of calcium from the
ECF.
- The calcium binding protein in smooth muscle is known as
calmodulin. Unlike troponin, calmodulin initiates contraction
immediately. First it activates a kinase enzyme that’s needed for
phosphorylation of myosin head. This phosphorylation triggers the
ATPase activity of the head and this is followed by contraction.
Steps of muscle relaxation:
 Shortly after releasing calcium to the sarcoplasm, the
sarcoplasmic reticulum begins to transport it back actively by
the Ca-Mg ATPase pump
 When the level of calcium in the sarcoplasm is lowered
sufficiently, calcium is released from troponin C
 This causes actin & myosin interaction to stop, tropomyosin
returns to its position & the muscle relaxes.

Remember that:
 ATP is consumed for both contraction & relaxation.
 If calcium transport back to the sarcoplasmic cisterns is inhibited,
relaxation will never occur (= contracture)

The core of medical physiology (1) – 3rd edition Page 87


 If no energy is available for relaxation (no ATP), the muscle
becomes rigid (= rigor). A common example is (rigor mortis) which
occurs after death.
- Relaxation in smooth muscles follows de-phosphorylation of myosin
heads by a phosphatase enzyme. However, contraction may
continue in spite of that for some time; this is due to the latch bridge
mechanism.
- In the latch bridge mechanism, myosin remains attached to actin
resulting in a sustained contraction with minimal energy expenditure;
as in the vascular smooth muscle.

Types of contraction
Isotonic contraction:
- Isotonic contraction (the same tension) is contraction against a
constant load that results in shortening of a muscle.
- The muscle develops a constant tension throughout the range of
movement (e.g. when lifting a light object).
Isometric contraction:
- Isometric contraction (the same length) is contraction without
appreciable shortening of a muscle.
- The tension developed by the muscle is not constant, it is
increasing (e.g. when trying to lift a very heavy object).
- Since there is no distance of movement (the same length of
muscle); no work is done during the isometric contraction (remember
that work = force times distance, which is zero).
- Standing involves isometric contraction of muscles whereas walking
involves both isometric & isotonic contractions.

The core of medical physiology (1) – 3rd edition Page 88


Single muscle twitch and summation of contractions
- A single sufficient stimulation of a muscle fiber results in a single
action potential that results in a single contraction followed by
relaxation (= single muscle twitch).
- The duration of the single muscle twitch is longer than the duration
of the action potential.
- Repeated stimulation of a muscle fiber results in repeated action
potentials that are followed by successive contractions.
- Since there is no refractory period for contractions, each new
contraction starts before completion of the previous one. This results
in summation of these contractions.
- The tension developed by summation of contractions is greater than
that’s developed by a single contraction.
- Summation of contractions into one continuous contraction is called
tetanus.
Tetanus
- Tetanus may be complete (with no any relaxation between
contractions) or incomplete (with some incomplete relaxations
between contractions).
- The frequency of stimulation that results in tetanus is determined by
the duration of the single muscle twitch.
- With a frequency of stimulation just below the frequency of
summation, the tension developed is increased by each new stimulus
until a uniform tension per contraction is reached. This is called
treppe or staircase phenomenon.
- Here the increase in tension is not due to summation, it is due to
increased availability of calcium to troponin C.

The core of medical physiology (1) – 3rd edition Page 89


Types of muscle fibers
There are two types of muscle fibers:
Type one fibers:
 Called red muscles (darker than other types)
 Respond slowly to stimulation (i.e. have long latency)
 Have high oxidative capacity
 Contract to maintain posture
 E.g. muscles of the back
Type two fibers:
 Called white muscles
 Have short twitch duration
 Respond rapidly (i.e. have short latency)
 Have high glycolytic capacity
 Specialized for fine skilled movement
 E.g. muscles of the hand

The oxygen debt mechanism


- During severe exercise, the amount of oxygen delivered to skeletal
muscle by the circulation is not enough for aerobic metabolism; for
this reason skeletal muscle has to find extra oxygen or additional
sources of energy; this is achieved as follows:
 It takes oxygen from myoglobin
 It metabolizes glucose anaerobically to lactic acid to synthesize
ATP
 It uses phosphorylcreatine to synthesize ATP
- Hyperventilation after exercise provides extra amount of oxygen
that can be used to repay oxygen taken from myoglobin, to

The core of medical physiology (1) – 3rd edition Page 90


catabolize lactic acid to carbon dioxide and water, and to replenish
phosphorylcreatine.
- This extra amount of oxygen consumed after exercise is described
as the oxygen debt.
- The mechanism of oxygen debt does not occur in the cardiac
muscle because it depends exclusively on aerobic metabolism for
energy production.

QUESTIONS FOR SELF ASSESSMENT-3 (BEST OF FIVE)


1- The resting membrane potential is generated by:
a. opening and closure of ligand gated channels
b. passage of ions through voltage gated channels
c. selective permeability of the cell membrane to potassium
d. closure of sodium leak channels by calcium
e. the high concentration of phosphates in the cell
2- The following is true about the action potential:
a. an external stimulus is not always needed for its initiation
b. the depolarization phase is always due to Na+ influx
c. its duration is equal in all types of excitable cells
d. after-depolarization phase is due to activity of the Na+/K+ pump
e. it can be summated in skeletal but not in cardiac muscle
3- Concerning myelination, which of the following is not true:
a. synthesized from phospholipids and protein
b. found in preganglionic sympathetic neurons
c. found in postganglionic sympathetic neurons
d. found in the central nervous system
e. formed by oligodendroglial cells
4- If ECF K+ concentration is increased from 3.5 to 5.0 mmol/L:
a. the resting membrane potential becomes less negative
b. the resting membrane potential will not change
c. fast voltage gated sodium channels will open
d. an action potential will occur spontaneously
e. the cell membrane becomes hyperpolarized
5- Leak channels:
a. are present in almost all cells in the body
b. have gates that open or close during action potentials
c. have gates that open when a hormone binds to its receptor
d. are responsible for the depolarization phase of action potentials
e. gates are closed by the end of the action potential

The core of medical physiology (1) – 3rd edition Page 91


6- Calcium channels in synaptic knob:
a. Allow release of neurotransmitter through them
b. Are voltage gated
c. Are ligand gated
d. Are opened when acetylecholine is released
e. Cause termination of the action potential
7- These proteins are involved in skeletal muscle contraction except:
a. calmodulin
b. actin
c. tropomyosin
d. myosin
e. troponin C
8- The A band in a striated muscle
a. is a light area
b. contains thick filaments only
c. is divided by the M line
d. is longer during relaxation than during contraction
e. appears due to overlapping between actin and myosin
9- Concerning chemical conduction, which of the following is not
true:
a. it is unidirectional (one way direction)
b. excitatory neurotransmitters open Na+ channels
c. inhibitory neurotransmitters open Ca++ channels
d. GABA opens CI- channels in the postsynaptic neurons
e. the post-synaptic neuron is influenced by many pre-synaptic
neurons
10- Ryanodine receptors:
a. allow influx of calcium from ECF to ICF in cardiac muscle
b. allow release of calcium from the sarcoplasmic cisternae to ICF
in skeletal muscle
c. are found in skeletal but not cardiac muscle
d. act as sensors for arrival of excitation through the T tubule
e. allow pumping back of calcium to sarcoplasmic cisternae
11- Isometric contraction differs from isotonic contraction in that:
a- it does not occur in cardiac muscle
b- it occurs during walking
c- it does not involve interaction between actin and myosin
d- the muscle tension applied is increased during the contraction
e- the force applied equals zero

Question 1. 2. 3. 4. 5. 6. 7. 8. 9. 10. 11.


Answer c a c a a b a c c b d

The core of medical physiology (1) – 3rd edition Page 92


CHAPTER 3
THE AUTONOMIC NERVOUS SYSTEM
INTRODUCTION
- The nervous system can be classified into two physiological
divisions: The somatic nervous system (SNS) and the autonomic
nervous system (ANS)
Table 3.1: Comparison between the SNS and the ANS:

The somatic nervous system The autonomic nervous system


- Controls voluntary functions - Controls involuntary functions
e.g. limb movements e.g. cardiac beats
- Supplies skeletal muscles - Supplies smooth and cardiac
muscles
- One neuron connects the - Two neurons connect the central
central part with the target part with the target muscle
muscle - The two neurons are:
preganglionic & postganglionic
neurons connected by a ganglion

Fig 3.1 Peripheral neurons of the SNS and the ANS

The core of medical physiology (1) – 3rd edition Page 93


Divisions of the autonomic nervous system
- The autonomic nervous system (ANS) is divided into:
 Sympathetic division
 Parasympathetic division
Differences between the sympathetic and the parasympathetic
divisions:
* Activators
- Stressful stimuli (e.g. fear, pain, exercise…) activates the
sympathetic while complete physical and mental rest (e.g. sleep)
activates the parasympathetic.
* Metabolism
- The sympathetic nervous system causes catabolism for immediate
release of energy that’s used for fight or flight; whereas the
parasympathetic is associated with anabolic reactions that store
energy until needed by the body.
* Anatomical points
- The peripheral parts of the ANS are made up of preganglionic
(myelinated, type B) and postganglionic (unmyelinated, type C)
neurons.
- The cell bodies of the preganglionic neurons are located in the
lateral horn of the spinal cord or the motor nuclei of the cranial
nerves 10, 9, 7 and 3 (= 1973). Their axons pass through the ventral
horns to the spinal nerves and then they follow the blood vessels to
the target organs.
- The preganglionic neurons of the sympathetic are short. They leave
the spinal nerve through white rami communicantes to synapse in
sympathetic ganglia with postganglionic neurons that return back to

The core of medical physiology (1) – 3rd edition Page 94


the spinal nerve via gray rami communicantes to be distributed to
their targets. They are longer than the preganglionic neurons.
Fig 3.2: Origin and course of a preganglionic neuron

- The sympathetic ganglia, the site of cell bodies of the


postganglionic neurons, are connected together to form the
sympathetic trunk that extends parallel to the thoracic and upper
lumbar spinal segments. Additional sympathetic ganglia are also
found in the neck (superior, middle and stellate ganglia) and in the
abdomen (collateral ganglia).
- On the other hand, the preganglionic parasympathetic neurons are
longer than the postganglionic. The ganglia are found near or within
the wall of the target organs.
Fig 3.3: The output of autonomic neurons

The core of medical physiology (1) – 3rd edition Page 95


Output from the CNS
- The central parts of the sympathetic and parasympathetic nervous
systems originate from certain autonomic centers at the brain (e.g.
the hypothalamus or the brain stem) and then descend through the
spinal cord.
- The peripheral part of the sympathetic passes out through the
spinal nerves that originate from the thoracic and some lumbar
segments (from T1 to L2); whereas the peripheral part of the
parasympathetic passes out through some cranial nerves (number
10, 9, 7 and 3) and some sacral spinal nerves (S2, S3 and S4).
- For this reason the outflow of the sympathetic is described as
thoraco-lumbar and that of the parasympathetic as cranio-sacral.
* Effects
- The ratio of a preganglionic neuron to postganglionic neurons is
about 1:20 for the sympathetic neurons and 1:1 for the
parasympathetic neurons. This indicates that the effects of the
sympathetic are generalized (because of divergence of its
preganglionic neurons) while those of the parasympathetic are
localized.
* The neurotransmitters released by the neurons
- All preganglionic neurons whether sympathetic or parasympathetic,
and all postganglionic parasympathetic neurons release
acetylcholine whereas most postganglionic sympathetic neurons
release noradrenaline. Others release acetylcholine.
- Postganglionic sympathetic neurons that release acetylcholine
supply the sweat glands, arterioles of skeletal muscles and pilo-
erectror muscles.

The core of medical physiology (1) – 3rd edition Page 96


Table 3.2: Comparison between the symp. and the parasymp.
Difference Sympathetic Parasympathetic
Activator* Stress Rest
Metabolism* Catabolic to release Anabolic to store
energy for fight or flight energy until needed
Pregan. neurons Short Long
Postga. neurons Long Short
Ganglia Near the spinal cord Near or within the
(paravertebral) wall of the organ
Output from the Thoracolumbar Craniosacral
CNS*
Effects* Generalized Localized
Neurotransmitter Pregananglionic: Preganglionic
released by the = Acetylcholine = Acetylcholine
neurons Postganglionic Postganglionic
= Noradrenaline = Acetylcholine

THE NEUROTRANSMTTERS
- The principal neurotransmitters (NT) in the autonomic nervous
system are acetylcholine and noradrenaline.
Acetylcholine:
- Synthesized from acetyl co A and choline in the cell body of
cholinergic neurons (i.e. neurons that release acetylcholine).
- Synthesis is catalyzed by the enzyme choline acetyltransferase.

The core of medical physiology (1) – 3rd edition Page 97


- Acetylcholine is released by:
o All preganglionic parasympathetic neurons
o All postganglionic parasympathetic neurons
o All preganglionic sympathetic neurons
o Some postganglionic sympathetic neurons (those
supplying sweat glands, arterioles of skeletal muscles and
pilo-erector muscles)
- When released at the synaptic cleft, acetylcholine acts on its
receptors. However, its action is terminated by the
acetylcholinesterase enzyme which converts it to acetate and
choline. Therefore generally no acetylcholine diffuses to plasma.
Noradrenaline (= norepinephrine):
- Synthesized from the amino acid tyrosine in noradrenergic
neurons (i.e. neurons that release noradrenaline)
- Steps of synthesis:
 Tyrosine (from diet or from phenylalanine) is hydroxylated to
dopa (by tyrosine hydroxylase in the cytoplasm).
 Dopa is decarboxylated to dopamine (by dopa decarboxylase
in the cytoplasm).
 Dopamine is converted to norepinephrine (by dopamine beta
hydroxylase in the vesicles).
- Noradrenaline is also synthesized in the adrenal medulla (an
endocrine gland regarded as a modified sympathetic ganglion
that lost its postganglionic axons). However, in this gland most of
the synthesized noradrenaline (= norepinephrine) is converted to
adrenaline (= epinephrine) by the enzyme phenylethanolamine-N-
methyltransferase (PNMT).

The core of medical physiology (1) – 3rd edition Page 98


- Adrenaline, noradrenaline and dopamine are called
catecholamines.
- In summary, noradrenaline is released by:
o Most postganglionic sympathetic neurons (excluding
those which release acetylcholine).
o The adrenal medulla (releases adrenaline plus some
noradrenaline).
- When released at the synaptic cleft, noradrenaline acts on its
receptors. There is no enzyme for its hydrolysis in the cleft.
However, after its diffusion into the plasma, it is hydrolyzed by two
enzymes:
 Monoamine oxidase (MAO)
 Catechol-O-methyl-transferase (COMT)
- The most important metabolite is vanillylmandelic acid (VMA),
which is excreted in urine.
- Excretion of high amount of this metabolite in urine indicates
hyper-production of catecholamines (e.g. by a tumor in the
adrenal medulla (= pheochromocytoma)).
Fig 3.4: Cholinergic and noradrenergic neurons

The core of medical physiology (1) – 3rd edition Page 99


Other neurotransmitters:
Dopamine: Released by some interneurons in symp. ganglia
GnRH: Released by some preganglionic neurons
Co-transmitters:
(VIP): May be found with acetylcholine
(ATP, Neuropeptide Y): may be found with noradrenaline

AUTONOMIC RECEPTORS
- These are the receptors for the neurotransmitters in the autonomic
nervous system (i.e. receptors for acetylcholine and noradrenaline)
Acetylcholine receptors
1- Nicotinic receptors (N)
- Stimulated by small amount of nicotine (a chemical substance
found in tobacco).
- Found at the following sites:
 Sympathetic ganglia
 Parasympathetic ganglia
 Neuromuscular junction
 Adrenal medulla
 The brain
2- Muscarinic receptors (M)
- Stimulated by small amount of muscarine (a chemical substance
excreted in urine of tadpoles).
- Found at the following sites:
 All organs supplied by postganglionic cholinergic nerves
(these include all organs in the body supplied by autonomic
neurons except ventricles of the heart and blood vessels of

The core of medical physiology (1) – 3rd edition Page 100


the skin, abdominal viscera and the kidney which are supplied
only by sympathetic, without parasympathetic supply).
 The brain

Noradrenaline receptors
1- Alpha receptors
Sites of alpha receptors:
Alpha 1:
Blood vessels
Dilator pupillae muscle
Sphincters
Alpha 2:
Pancreas
Presynaptic membranes
2- Beta receptors
Sites of beta receptors:
Beta 1:
The heart
Renin secreting cells
Adipose tissue
Beta 2:
Bronchi
Uterus
Pancreas
Beta 3:
Adipose tissue
-

The core of medical physiology (1) – 3rd edition Page 101


Remember that:
 Both noradrenaline and adrenaline act on α and β receptors.
 Noradrenaline acts better on α receptors than on β receptors
while adrenaline acts better on β receptors than on α
receptors.

Fig 2.5: Autonomic receptors

 N = Nicotinic receptors M= Muscarinic receptors

EFFECTS OF SYMP. AND PARASYMP. STIMULATION


- All postganglionic parasympathetic neurons are cholinergic and
most postganglionic sympathetic neurons are noradrenergic.
- Therefore sympathetic effects are generally mediated by
noradrenaline and parasympathetic effects are generally mediated by
acetylcholine.
The core of medical physiology (1) – 3rd edition Page 102
- Examples of these effects on various organs in the body:
Organ Sympathetic effect Parasympathetic effect
Eye pupil Dilation (it contracts the Constriction (it contracts
dilator pupillae muscle/ the constrictor pupillae
alpha receptors) muscle/ muscarinic Rs)
Salivary Stimulates secretion Stimulates secretion
glands (small amount, mucus in (large amount, watery in
consistency) consistency)
Bronchioles Bronchodilation/ Bronchoconstriction/
through beta 2 Rs through muscarinic Rs
Heart rate Increased (Positive Decreased (Negative
chronotropic)/ beta 1 chronotropic/ Muscarin.
Contractility Increased No direct effect on
of the heart Positive inotropic contractility (it does not
supply the ventricles)
GIT motility Decreased (inhibitory, Increased (excitatory, it
& contracts the sphincters relaxes the sphincters
Secretions and relaxes the walls) and contracts the walls)
Blood - Vasoconstriction - Does not supply and
vessels (mediated through alpha therefore has no effect
receptors) on blood vessels of the
- Vasodilatation of some skin, abdominal viscera
blood vessels like the and the kidney.
coronary artery - Causes vasodilatation
(mediated through beta 2 in some blood vessels
receptors). like the coronary artery)

The core of medical physiology (1) – 3rd edition Page 103


Organ Sympathetic effect Parasympathetic effect

Micturition & Not involved in the Excitatory (contracts the


Defecation reflex; but, its walls of the bladder &
reflexes stimulation inhibits rectum & relaxes their
these reflexes sphincters)
Sex organs Mediates the Mediates the erection reflex
ejaculation reflex
Sweat glands Causes sweating No effect (does not
supply sweat glands)

BLOCKERS
- Block the actions of neurotransmitters at their receptors
- Two types:
 Competitive blockers
- Compete with the neurotransmitter (NT) for binding with its
receptors.
- The affinity of the receptor for the blocker is higher than for the
NT.
- The blocker occupies the receptor without producing any
response.
 Depolarizing blockers
- Causes prolonged depolarization of the receptor.
- The receptor becomes in state of refractory period, therefore it
does not respond to the neurotransmitter.

The core of medical physiology (1) – 3rd edition Page 104


Blockers of Acetylcholine
 Nicotinic blockers:
- Blockers of acetylcholine at nicotinic receptors
- Competitive blockers:
o Curare (at the neuromuscular junction)
o Hexamethonium (at the ganglia)
- Depolarizing blocker:
o Large amount of nicotine
 Muscarinic blockers:
 Competitive blockers:
o Atropine
o Scopolamine (hyoscine)
 Depolarizing blocker:
o Large amount of muscarine

Blockers of noradrenaline
 Alpha blockers:
o Phentolamine
o Prazosin (alpha 1)
o Yohimbine (Alpha 2)
 Beta blockers:
o Propranolol
- (non selective blocker; blocks beta 1 & beta 2 receptors)
o Atenolol
- (selective blocker; blocks beta 1)
o Butoxamine
- (selective blocker; blocks beta 2)

The core of medical physiology (1) – 3rd edition Page 105


ABNORMALITIES
 Horner’s Syndrome
- Results from damage to sympathetic neurons that supply the face
- The damage may occur at any site along the course of sympathetic
neurons which originate from the hypothalamus, descend through the
brain stem, emerge from the upper thoracic segments, synapse at
the superior, middle or inferior cervical ganglia and then pass with
blood vessels to supply the face. However, the damage usually
occurs at the neck (cervical sympathectomy).
- Examples of the causes:
o Apical lung tumor
o Surgical trauma at the neck
o Hypothalamic lesions
- The syndrome is characterized by the following signs (on the
affected side of the face):
o Ptosis (drooping of the upper eye lid)
o Meiosis (constriction of the eye pupil)
o Anhydrosis (dryness, due to loss of sweating)
o Enophthalmos (abnormal recession of the eyeball in the
orbit)
o Rubor (redness, due to vasodilatation)
o Calor (hotness, due to vasodilatation)
 Myasthenia Gravis
- Results from damage to acetylcholine receptors at the
neuromuscular junctions.
- The damage is caused by auto-antibodies induced by unknown
mechanism

The core of medical physiology (1) – 3rd edition Page 106


- Develops in females more than males.
- It is characterized by:
o Muscle weakness (especially after repeated movement)
o Ptosis (drooping of the upper eye lid)
o Weakness of respiratory muscles (the patient may die
due to respiratory failure).
- Treatment:
- By anti- acetylcholine esterase (e.g. neostigmine); to decrease
break down of acetylcholine at the neuromuscular junction and allow
it to act on the remaining receptors.
- Other modalities of treatment include:
o Use of steroids to inhibit production of the auto-antibodies by
lymphocytes
o Decrease of further breakdown of acetylcholine receptors by
plasmapheresis (a machine that eliminates the autoantibodies
from plasma).
Table 3.3: Effects of drugs acting on autonomic receptors
ATROPINE PROPRANOLOL SALBUTAMOL
Mechanism of Muscarinic β1 & β2 blocker β2 agonist
action blocker (antagonist) (stimulant)
Effects Heart rate Heart rate & - Bronchodilation
GIT motility & contractility Heart rate (side-
secretion Cardiac output effect, because
- Bronchodilation Blood pressure although it is β2
- Dilatation of the - Vasoconstriction agonist, it may
eye pupil - Broncho- react with β1
- Dries mouth constriction receptors resulting
( saliva) in tachycardia)

The core of medical physiology (1) – 3rd edition Page 107


QUESTIONS FOR SELF ASSESSMENT-4 (BEST OF FIVE)

1. The somatic nervous system differs from the autonomic nervous


system because:
a- it has sensory and motor neurons
b- its neurons release acetylcholine
c- some of its neurons arise from the spinal cord
d- it has nicotinic receptors
e- it causes skeletal muscle contraction
2. Effects of the parasympathetic nervous system include all the
following except:
a- meiosis
b- penile erection
c- increased gastrointestinal motility
d- decreased airway resistance
e- bradycardia
3. Stimulation of the sympathetic nervous system results in:
a- bradycardia
b- recruitment in the cardiac muscle
c- dilatation of skeletal muscle arterioles
d- excessive salivary secretion
e- negative inotropic effect
4. Acetylcholine:
a- is released by all postganglionic sympathetic neurons
b- is released from the adrenal medulla
c- causes contraction of skeletal muscles
d- is blocked by propranolol
e- has alpha and beta receptors
5. The drug that blocks muscarinic receptors results in:
a- tachycardia
b- excessive salivation
c- elevation of blood pressure
d- micturition
e- sweating
6. Nicotinic receptors are found in:
a- arterioles of skeletal muscle
b- motor end plates of skeletal muscles
c- sweat glands in the skin
d- small intestine
e- ventricles of the heart

The core of medical physiology (1) – 3rd edition Page 108


7. The nicotinic cholinergic receptors at the autonomic ganglia are:
a- blocked by curare
b- stimulated by salbutamol
c- stimulated with noradrenaline
d- blocked by hexamethonium
e- identical to those in skeletal muscles
8. Stimulation of beta 1 receptors results in all the following except:
a- breakdown of adipose tissue
b- increased cardiac output
c- release of renin from the juxta-glomerular cells
d- tachycardia
e- vasoconstriction
9. Adrenal medullary cells are stimulated by:
a- dopamine
b- GABA
c- adrenaline
d- acetylcholine
e- nor-adrenaline
10. Dopamine is:
a- released by all autonomic ganglia
b- an excitatory neurotransmitter
c- an acetylcholine derivative
d- a noradrenaline precursor
e- the main neurotransmitter released by the adrenal medulla
11. Sympathetic nerves
a- release noradrenalin from pre-ganglionic fibres
b- release adrenalin from post- ganglionic fibres
c- are stimulated during sleep
d- release of acetylcholine by some postganglionic fibres
e- are mostly myelinated
12. Stimulation of the sympathetic nervous system results in:
a- tachycardia through β2 receptors
b- constriction of skin blood vessels through β1
c- dilatation of the eye pupil through β1 receptors
d- contraction of intestinal wall through alpha receptors
e- sweating through muscarinic receptors

Question 1. 2. 3. 4. 5. 6. 7. 8. 9. 10. 11. 12.


Answer e d C c a b d e d d d e

The core of medical physiology (1) – 3rd edition Page 109


CHAPTER 4
TEMPERATURE AND METABOLIC RATE
TEMPERATURE
Normal body temperature
- The temperature of the internal structures of the body is called the
core body temperature.
- It equals 37oC (± 0.5oC) or 98.6oF (± 1.3oF)
- The core body temperature differs from the temperature of the skin
because:
 The temperature of the skin is affected directly by the temperature
of the external environment.
 The subcutaneous fat layer beneath the skin acts as insulator. It
preserves the core body temperature and prevents the effect of
the external environment.
Remember that:
 Skin vasodilatation (as occurs when the core body temperature
rises) allows blood to pass through the subcutaneous fat layer
and comes under the skin to facilitate heat loss to the outside.
The opposite occurs when the core body temperature falls.

Measurement of the core body temperature


- The device used is the clinical thermometer
- The sites used for measurement are:
The rectum
 The most accurate site
 Not socially acceptable
 Used in children and unconscious patients
The core of medical physiology (1) – 3rd edition Page 110
The mouth (under the tongue)
 The most frequently used site (Socially acceptable)
 Differs slightly from the core body temperature (less by 0.5oC )
 Affected by hyperventilation, smoking and recently ingested food
or drink.
Other less frequently used sites:
o The axilla (less accurate; affected by the external environment)
o The ear (less accurate; affected by the external environment)
o Freshly passed urine (more accurate “but used for research
purposes only”)

Regulation of body temperature


- Maintenance of normal body temperature is an important goal in
homeostasis.
- This is because disturbance in body temperature affects:
 Activity of enzymes
 Speed of metabolic reactions within the body
 Activity of the various systems in the body. For example:
o The nervous system: higher body temperature increases
conduction in neurons whereas lower body temperature
decreases conduction.
o The cardiovascular system: higher body temperature causes
tachycardia and increases contractility whereas lower body
temperature does the reverse.
o The respiratory system: higher body temperature causes
hyperventilation whereas lower body temperature causes
hypoventilation.

The core of medical physiology (1) – 3rd edition Page 111


o The endocrine system: higher body temperature decreases
release of thyroid hormones whereas lower body temperature
increases release of thyroid hormones.
- Regulation of body temperature within narrow limits is not only a
characteristic of humans; it is also a characteristic of birds and
mammals. For this reason they are described as "warm blooded" or
homeothermic.
- Other vertebrates like reptiles, amphibia and fish are "cold blooded"
or poikilothermic because the range of their body temperature is
rather wide.
- Invertebrates, on the other hand, can not regulate their body
temperature.
The temperature regulatory center:
- There is a center specialized for control of body temperature located
in the hypothalamus, known as the temperature regulatory center.
- This center controls heat loss & heat gain (the anterior
hypothalamus controls heat loss & the posterior hypothalamus
controls heat gain.
- Notice that: to maintain constant body temperature, the degree of
heat loss should always be balanced by an equivalent degree of heat
gain.
Heat loss:
- Heat is lost from the body by:
1. Conduction
- Transfer of heat between objects in contact with each other
(From an object with higher temperature to another with lower
temperature).

The core of medical physiology (1) – 3rd edition Page 112


- Conduction occurs for either heat loss or heat gain,
depending on the level of body temperature.
2- Convection
- Transfer of heat away from a surface by successive currents
of air or water (from the object to the air or water currents, if
they have temperature lower than that of the object).
- Convection occurs for either heat loss or heat gain, depending
on the level of body temperature.
3- Radiation
- Transfer of heat between objects not in contact with each
other (from that of higher temperature to another with lower
temperature).
- Radiation occurs for either heat loss or heat gain, depending
on the level of body temperature, compared to the
temperature of the water or air currents.
4- Evaporation
- Vaporization of water molecules from a surface, taking heat.
- For example, evaporation of sweat from the surface of the
body or evaporation of water from the mucus membranes
during respiration.
- Notice that vaporization of 1g of water removes about 0.6 kcal
of heat.
- During exercise, sweat secretion may reach 1.6 L/h.
Evaporation of this volume results in loss of more than 900
kcal of heat per hour.
- Evaporation occurs for heat loss only (heat is always lost, not
gained by evaporation).

The core of medical physiology (1) – 3rd edition Page 113


Heat gain
- The body gains heat by:
1- Metabolism
- Metabolism of all types of energy substrate, especially brown
fat, produces considerable heat.
- Brown fat is more abundant in infants than adults. It is found
mainly between scapulae and at the nape of the neck. Unlike the
white fat, it has rich sympathetic innervation and high metabolic
activity.
2- Muscular activity
- Contraction of muscles is a major source of heat. This may be
induced voluntarily (e.g. exercise) or involuntarily (e.g. shivering)
- Shivering is stimulated when the body temperature reaches
35.5oC.
3- Food intake (Specific dynamic action of food)
- Obligatory energy release that occurs during assimilation of food
within the digestive system (i.e. before absorption).
- The energy release of specific dynamic action of food is highest
for proteins than for carbohydrates or fats.
4- From the environment
- By conduction, convection or radiation (notice that these
mechanisms are suitable for both heat loss and heat gain).
- Remember that: In a hot weather, the mechanisms of conduction,
convection and radiation are all for heat gain; the only way for heat
loss is through evaporation of sweating. However, in a very humid
environment sweating occurs without evaporation. Therefore the
temperature of the body is not decreased.

The core of medical physiology (1) – 3rd edition Page 114


Body responses to a hot environment
- Due to the direct effect of a hot environment, the temperature of
the body starts to rise.
- This rise is detected by thermoreceptors located peripherally in
the skin and centrally in the spinal cord and hypothalamus.
- The peripheral and central chemoreceptors send their impulses to
the temperature regulatory center in the hypothalamus.
- The center, to correct the body temperature, increases heat loss
(by the anterior hypothalamus) & decreases heat gain (by the
posterior hypothalamus).
- Heat loss is increased by:
o Sweating (to increase evaporation from the skin)
o Vasodilatation (to increase blood supply to the skin, for
radiation)
o Increased respiration (to increase evaporation from the mucus
membranes).
o Behavioral changes to increase heat loss, these include:
 Stretching at sleep (to increase surface area for radiation)
 Light clothes and cold drinks
- Heat gain is decreased by:
o Decreased metabolism, decreased food intake and decreased
muscular activity.

 Body responses to a cold environment


- Due to the direct effect of the cold environment, the temperature
of the body starts to drop.
- This drop is detected by peripheral and central thermoreceptors.
The core of medical physiology (1) – 3rd edition Page 115
- The thermoreceptors send impulses to activate the temperature
regulatory center in the hypothalamus.
- The center increases heat gain (by the posterior hypothalamus) &
decreases heat loss (by the anterior hypothalamus).
- Heat gain is increased by:
o Increased metabolism
o Increased food intake (the body temperature rises due to the
specific dynamic action of food).
o Increased muscular activity (shivering and exercise)
- Heat loss is decreased by:
o Decreased sweating (to decrease evaporation from the skin)
o Vasoconstriction (to decrease blood flow to the skin)
o Decreased respiration (to decrease evaporation through the
mucus membranes)
o Behavioral changes that include:
 Curling up at sleep (to reduce the surface area). Other
behavior changes like heavy, dark-colored clothes and
hot drinks increase heat gain.

Physiological variation in body temperature


- Body temperature varies physiologically according to:
Age
- Higher in neonates> children > adults
- The higher temperature in neonates and children is due to heat
released by metabolic reactions of the growing tissues
Gender
- Higher in males than females of the same age and weight

The core of medical physiology (1) – 3rd edition Page 116


- Due to presence of higher percentage of inactive tissues (fats) in
females
At and following ovulation
- Due to release of progesterone which has thermogenic effect
Pregnancy
- Due to higher rate of metabolism (fetal tissues) & higher level of
progesterone
Lactation
- Due to higher rate of metabolism (breast tissue)
Circulating hormones
- Normal variation in levels of thyroid hormones and catecholamines
results in equivalent variation in body temperature
Circadian fluctuation
- Body temperature fluctuates normally throughout the day of 0.5- 0.7
o
C. It is lowest early in the morning and highest at the evening.
Emotions
- Due to unconscious tensing of muscles.
Sleep and exercise
- Body temperature rises with muscular activity and drops at sleep.

Abnormalities of temperature regulation


Fever (Pyrexia)
 An important sign of disease (e.g. tonsillitis, pneumonia, typhoid,
malaria, malignancies …)
 Caused by endogenous pyrogens (e.g. cytokines released by
phagocytic cells, such as interleukin-1 (IL-1), IL-6 and tumor
necrosis factor (TNF ), which act to increase body temperature).

The core of medical physiology (1) – 3rd edition Page 117


 Release of the endogenous pyrogens is induced, in most times, by
exogenous pyrogens (substances not synthesized by the body
such as bacterial toxins).
 Interleukins, which can not cross the blood brain barrier to reach
the preoptic area in the hypothalamus, induce synthesis of a
substance (most probably prostaglandin E2) to reach that area
which is the site of temperature setting in the body (= Thermostat).
 Prostaglandin E2 resets the thermostat from 37oC to a higher
level (e.g. 40oC). The subject feels cold, and therefore he
develops changes that elevate the body temperature such as
shivering, non-shivering thermogenesis (breakdown of brown
fat) and skin vasoconstriction; these result in fever.
Remember that:
 Immediately following loss of the prostaglandin effect, the
temperature setting point returns back to 37oC. Since the body
temperature is still higher than 37oC, the subject develops
changes to decrease the body temperature such as sweating and
vasodilatation of skin blood vessels.
 Drugs that decrease synthesis of prostaglandins can treat fever
(e.g. Aspirin).

Heat stroke
 In a hot dry environment, the only way to lose heat is through
evaporation of sweating.
 Therefore, failure of sweating results in abnormal elevation of
body temperature to a degree that affects the activity of the
nervous system, resulting in convulsions, coma and even death.

The core of medical physiology (1) – 3rd edition Page 118


 In heat stroke, there is damage to the temperature regulatory
center caused by the high environmental temperature. This was
confirmed at autopsy. However, the exact mechanism of damage
is not well understood.
 Sometimes, as in severe exercise, heat stroke may result from
failure of the temperature regulatory mechanisms to maintain
normal body temperature (i.e. inadequate sweating); due to high
rate of heat production by the exercising muscles.
 Failure to reduce body temperature is followed by complications of
high temperature (convulsions, coma and death).
 Treatment should start immediately (e.g. electrical beds to reduce
the body temperature gradually or if not available: ice water;
However, body temperature should not be lowered quickly).

Heat exhaustion
 In a hot humid environment, even sweating can not reduce body
temperature because it fails to evaporate. For this reason subjects
should keep themselves in a cold and well ventilated area to lose
heat.
 In heat exhaustion, there is excessive sweating but with no
evaporation.
 The body temperature becomes elevated, the activity of the
nervous system is affected and the patient develops dehydration
(due to the excessive sweating).
 If not treated, the temperature regulatory mechanisms eventually
fail and the heat exhaustion becomes complicated by heat stroke
(permanent damage to temperature center).

The core of medical physiology (1) – 3rd edition Page 119


 Therefore it is important to treat the condition immediately by
taking the patient to a well ventilated, cold room and giving him IV
fluids (in form of normal saline).

Table 4.1: Differences between heat stroke and heat exhaustion


Difference Heat stroke Heat exhaustion
Classical environment Hot dry Hot humid
Temperature center Abnormal Normal
Sweating Absent Excessive
Skin Hot & dry Hot & wet
Dehydration Absent Present
Effects of the high temperature
- Tachycardia Present Present
- Hyperventilation
- Convulsions & coma

Hypothermia
 Diagnosed when rectal temperature is < 35oC.
 Characterized by loss of consciousness, bradycardia and
decreased respiration.
 Occurs due to the direct effect of cold environments on slim
subjects with thin subcutaneous fat layer (e.g. malnourished
children and old people); or subjects with immature temperature
regulatory mechanisms (e.g. pre-term babies).
 Treatment by stepwise elevation of body temperature using heavy
clothes or electric blankets (because sudden elevation of body
temperature is dangerous).

The core of medical physiology (1) – 3rd edition Page 120


Lesions of the hypothalamus
- The anterior hypothalamus controls heat loss (e.g. vasodilatation
and sweating) whereas the posterior hypothalamus controls heat
gain (e.g. shivering).
- Damage involving the anterior hypothalamus causes hyperthermia
in hot environments (due to failure of heat loss) whereas damage
involving the posterior hypothalamus causes hypothermia in cold
environments (due to failure of heat gain).

Malignant hyperthermia
- This is a life threatening condition that occurs due to a mutation in
the gene coding for the ryanodine receptors in skeletal muscles.
- Anesthetic drugs such as halothane and suxamethonium trigger
excessive release of calcium through these receptors from the
sarcoplasmic cisternae into the sarcoplasm, causing prolonged
contraction. Calcium causes contraction of muscles, this results in
sudden rise in body temperature and eventually death.

Heat cramps
- These are brief, painful muscle spasms occurring in the abdomen,
thigh or calf muscles.
- They occur during or after exercise, especially in the un-acclimatized
subjects, due to loss of large amounts of water and salts in sweat.
- Acclimatization involves elevation of aldosterone level which
reabsorbs sodium from sweat glands to prevent its loss in sweat.
- Treatment measures include resting of the muscles, rehydration and
correction of sodium and potassium loss.

The core of medical physiology (1) – 3rd edition Page 121


THE METABOLIC RATE
- The word metabolism refers to all chemical transformations that
occur within the body.

- It includes:
 Anabolic reactions:
o Synthesis of complex molecules from simpler ones
o Example: glycogen from glucose, protein from amino acids…
o Involves consumption of energy for synthesis
 Catabolic reactions:
o Degradation of complex molecules to simpler ones
o Example: glycogen to glucose, protein to amino acids…
o Involves release of energy during degradation

THE BASAL METABOLIC RATE


 The metabolic rate is defined as the rate of energy release per unit
time.
 If it is measured at certain standard conditions; it is called the
basal metabolic rate (BMR).
 The standard conditions for its measurement are:
o Complete physical and mental rest
o Fasting for at least 12 hours
o Comfortable temperature
 The energy released by the body at these standard conditions is
used to maintain basal functions in the body such as:
o Beating of the heart
o Respiration

The core of medical physiology (1) – 3rd edition Page 122


o Activity of the nervous system
o Sodium-Potassium ATPase pump

Measurement of the BMR


 Normal BMR = 2000 kcal/day or 40 kcal/m2/h
 Measurement of the BMR was formerly used to diagnose thyroid
problems:
o If increased by 15% = it indicates hyperthyroidism
o If decreased by 15% = it indicates hypothyroidism
BMR is measured by 2 methods:
 Direct Calorimetry
(Not preferred because it is difficult and needs special laboratories)
 Indirect Calorimetry
(Preferred)

Direct Calorimetry
 Involves incubation of a subject in a vessel container for a certain
period of time.
 The vessel container is surrounded by a known volume of water
and covered from outside by insulator.
 The energy released during this period of time, in the form of heat,
will raise the temperature of the water around the container.
 This change in temperature is used to calculate the BMR.

Indirect Calorimetry
 Involves indirect measurement of the BMR by measuring certain
substances consumed or produced during the catabolic reactions.
 For example, take the reaction:

The core of medical physiology (1) – 3rd edition Page 123


C6 H12 O6 + (6) O2 = (6) CO2 + (6) H2 O + energy (heat)
 Here the amount of energy released is known
 Using the above information, the metabolic rate can be calculated
indirectly by measuring any of the following during a certain period
of time:
o the amount of oxygen consumption
o the amount of glucose consumption
o the amount of carbon dioxide production
o the amount of water production

Using the oxygen consumption:


 The amount of energy produced when one liter of oxygen is
consumed is called the joule equivalent (JE)
 It differs according to difference in the foodstuff being oxidized.
This is because the ratio of carbon to oxygen atoms differs in
different types of foodstuff.
 The difference in the ratio of carbon to oxygen atoms also results
in difference in the respiratory quotients (RQ) of foodstuffs.
 The RQ of each foodstuff is defined as the ratio of carbon dioxide
produced to oxygen consumed in the steady state, when
equilibrium is reached at rest.
 It differs from the respiratory exchange ratio (R) which is the
ratio of CO2 to O2 at any given time (see below).
 The RQ for;
o CHO = 1, Protein = 0.82, Fat = 0.7, Mixed diet = 0.85
 The RQ can be used to calculate the joule equivalent (JE) for each
foodstuff as follows: JE = 17.15 + (RQ x 3.5)

The core of medical physiology (1) – 3rd edition Page 124


 Now the BMR can be calculated as follows:
 BMR = oxygen consumption x joule equivalent
(in joule/unit time)
 BMR = oxygen consumption x joule equivalent/surface area x 4.2
(in kcal/m2 /h).

Note: The volume of oxygen consumption should be corrected for


standard temperature & pressure (STP) by multiplying it by a factor,
obtained from nomograms (e.g. the correction factor at STP when air
temperature = 21◦c & ambient pressure = 741 mmHg equals 0.883)
Notes to remember about the respiratory exchange ratio (R)
 It is affected by hyperventilation (e.g. during exercise or metabolic
acidosis); here R is increased (due to expiration of more CO2 by
the hyperventilation). Then following exercise, R is decreased
(due to the oxygen debt mechanism).
 The respiratory quotient (RQ) and the respiratory exchange ratio
(R) of individual organs give information about their activities.
 For example the RQ for the brain (measured during the steady
state) = 0.99; indicating that its main source of energy is glucose.
 The R for the stomach (measured during active secretion of HCl)
is negative, indicating that more CO2 is consumed than produced;
this is used in synthesis of HCL (see the GIT in volume 2).

Measurement of oxygen consumption


 Requirement
 Bell-type spirometer (Benedict’s Roth)
 Kymograph

The core of medical physiology (1) – 3rd edition Page 125


 Method
 The subject puts the mouth piece of the spirometer in his mouth
 Firstly, the spirometer is connected to the atmosphere to familiarize
the subject to the procedure before recording
 Then the atmospheric opening is closed to allow the subject to
consume the oxygen inside the cylinder of the spirometer
 Oxygen consumption is obtained from a recording on a kymograph
paper of a spirometer by measuring the slope of the line A-B as in
this example:
Fig 4.1: Measurement of oxygen consumption

 Calculation
 BMR = oxygen consumption x correction factor for STP x Joule
equivalent/ Surface area x 4.2

The core of medical physiology (1) – 3rd edition Page 126


Remember that:
 If the standard conditions mentioned above are not fulfilled; the
value obtained is a metabolic rate (not BMR)
 To calculate the joule equivalent using the formula (JE = 71.15 +
(3.5 x RQ)), the type of foodstuff consumed by the subject over
the past few days should be known; this gives the RQ
 The surface area can be obtained, using the height and weight of
a subject, from nomograms or certain equations

Factors affecting the metabolic rate


 Age
- Higher in neonates> children > adults> old subjects
- Due to increased metabolism of the growing tissues. Here the
metabolic rate should be expressed per surface area for comparison.
 Gender
- Higher in males than females of the same age and weight.
- Due to presence of higher percentage of fats in females.
 At and following ovulation
- Due to the higher body temperature caused by progesterone.
 Pregnancy and lactation
- Due to presence of more metabolically active tissues (fetal and
breast tissues); & higher level of progesterone.
 Circulating hormones
- Normal variation in levels of thyroid hormones and catecholamines
results in equivalent variation in the metabolic rate.
 Emotions
- Due to unconscious tensing of muscles.

The core of medical physiology (1) – 3rd edition Page 127


 Sleep and exercise
- The metabolic rate rises with activity.
 Recent ingestion of food (SDA)
- Due to the specific dynamic action of food (SDA). As mentioned
earlier it is the obligatory energy release following ingestion of food,
due to its assimilation within the GIT. However, its exact cause is
unknown. It lasts for 6 hours. The rise in metabolic rate is highest
following protein ingestion and lowest following fat ingestion.
 Body temperature and environmental temperature
- The relation between environmental temperature and the rate of
metabolism is generally an inverse relationship (increased
environmental temperature = decreased metabolic rate). However,
the relation between body temperature and the rate of metabolism is
a direct relationship (increased body temperature = increased
metabolic rate).
- The relation between the environmental temperature and the rate of
metabolism is explained by the body responses to hot or cold
environment. - For example, the rate of metabolism rises in a cold
environment and drops in a hot one (see above).
- However, when the environmental temperature rises enough to
elevate the body temperature or drops enough to decrease the body
temperature, the metabolic rate increases or decreases respectively
(i.e. the relation becomes a direct relationship).
- It is calculated that for each one Celsius degree rise in body
temperature, the metabolic rate increases by 14%.

The core of medical physiology (1) – 3rd edition Page 128


QUESTIONS FOR SELF ASSESSMENT-5 (BEST OF FIVE)
1. The most accurate site for measurement of body temperature is:
a. The mouth
b. The axilla
c. The rectum
d. The nose
e. The skin
2. Whenever the body temperature is lower than the set point of the
thermostat in the hypothalamus:
a. Sweating is increased
b. Skin blood vessels dilate
c. Rate of metabolism is increased
d. Subjects feel hot sensation
e. Appetite to food is inhibited
3. Features of heat stroke include all the following except:
a. Hypotension
b. Dehydration
c. Low urine output
d. Absence of sweating
e. Convulsions
4. Heat exhaustion:
a. Occurs in hot dry environments
b. Is due to failure of sweating
c. Is best treated with ice water immersion
d. Is associated with low plasma osmolarity
e. Is associated with dehydration
5. At the onset of fever, patients develop:
a. Hot sensation
b. Sweating
c. Convulsions
d. Skin vasoconstriction
e. Nausea and vomiting
6. Exposure to a cold environment is not excepted to cause:
a. Release of thyroid hormones
b. Peripheral cyanosis
c. Acceleration of metabolism
d. Reduction in blood pressure
e. Reduction in heart rate
7. The main way for heat loss from a body in a hot dry environment is
through:
a. Sweating
b. Hyperventilation
c. Evaporation

The core of medical physiology (1) – 3rd edition Page 129


d. Radiation
e. Conduction
8. Concerning regulation of body temperature:
a. Birds and mammals are poikilothermic
b. The temperature regulatory centre is located in the medulla
c. Convection never causes heat gain
d. Shivering occurs when body temperature falls below 36.5 ◦C
e. Damage to the posterior hypothalamus is expected to cause
hypothermia
9. Heat stroke differs from heat exhaustion because it:
a. Usually occurs in both dry and humid environments
b. Is associated with elevated core body temperature
c. Causes tachycardia and impaired level of consciousness
d. Is not characterized by dehydration
e. Is not associated with sweating
10. Measurement of the basal metabolic rate requires:
a. fasting for two hours
b. ingestion of a heavy meal
c. moderate exercise during measurement
d. normal body temperature
e. suitable environmental temperature
11. Basal metabolism is mainly accounted for by:
a. Energy spent by cardio-respiratory functions
b. Secretions in the endocrine system
c. The sodium-potassium pump
d. Maintenance of body temperature
e. Renal tubular mechanisms
12. In the body the metabolism of 10 grams of protein would produce
approximately:
a. 10 kcal
b. 53 kcal
c. 41 kcal
d. 90 kcal
e. 20 cal
13. For measurement of the basal metabolic rate all the following
conditions are required except:
a. mental rest
b. physic rest
c. normal body temperature
d. 12 hours fasting prior to measurement
e. comfortable room temperature
14. The specific dynamic action of food is due to:
a. an increase in core body temperature
b. amino acid metabolism in the liver

The core of medical physiology (1) – 3rd edition Page 130


c. energy expenditure during digestion
d. hypothalamic sympathetic stimulation
e. secretion of insulin
15. The basal metabolic rate:
a. is about 500 kcal/ kg body weight / day
b. is measured during exercise
c. is measured during exposure to cold environment
d. rises during menstruation and lactation
e. is usually measured two hour following a meal
16. All the following conditions are expected to increase the rate of
metabolism in a young woman EXCEPT:
a. Fasting
b. Fever
c. Menstruation
d. Second trimester of pregnancy
e. Cold environment
17. The respiratory exchange ratio is:
a. The ratio of CO2 production to O2 consumption during steady
states
b. Used to calculate the joule equivalent
c. Used to calculate oxygen debt of organs
d. Negative for the stomach during active secretion of HCL
e. Negative for the brain at rest
18. The metabolic rate is increased by all of the following except:
a. Rise in body temperature
b. Rise in environmental temperature
c. Increased sympathetic outflow
d. Stress
e. Hyperfunction of the thyroid gland
19. Which of the following is the main consumer of energy under
basal conditions?
a. Heart activity
b. Respiratory muscle activity
c. Sodium-potassium pump
d. Skeletal muscle activity
e. Liver metabolism

Question 1. 2. 3. 4. 5. 6. 7. 8. 9. 10.
Answer c c b e d d c e e e
Question 11. 12. 13. 14. 15. 16. 17. 18. 19. 20.
Answer c c c c d a d b c

The core of medical physiology (1) – 3rd edition Page 131


CHAPTER 5
HEMATOLOGY
BLOOD
- Blood is a type of connective tissue.
- Like other types of connective tissue it consists of:
– Cells and intercellular substance
- The cells are:
– Red blood cells (RBCs) or erythrocytes
– White blood cells (WBCs) or leukocytes
– Platelets or thrombocytes
- The intercellular substance is called plasma
- Volume of blood is about 5L in an average adult male (= 8% of total
body weight).
- Volume of plasma is about 3.5L (= 5% of total body weight).
- The cells in blood can be separated from plasma by centrifugation.
- The percentage of cells in blood is called the packed cell volume
(PCV) or the hematocrit.

The PCV

• PCV is the percentage of blood occupied by cells.

• Most of these cells are RBCs (more than 90%).

• Normal values = 40-50% in males [average = 45%]


= 37-47% in females [average = 42%]

• Low PCV indicates anemia

• High PCV indicates polycythemia.

The core of medical physiology (1) – 3rd edition Page 132


Fig 5.1: The packed cell volume

Functions of blood
1. Transport of:
– Gases (e.g. oxygen and carbon dioxide)
– Nutrients (e.g. glucose, amino acids and free fatty acids)
– Waste products (e.g. urea and uric acid)
– Hormones (e.g. catecholamines, insulin, cortisol and thyroid
hormones)
2. Defense by WBCs
3- Homeostasis (maintenance of the constancy of the internal
environment, which is the ECF):
– Control of temperature (by distribution of heat by the blood)
– Control of pH (by buffers in the blood, e.g. HCO3-, proteins and
hemoglobin)
4- Hemostasis
– Prevention of blood loss & maintenance of blood in the fluid
state. Blood loss is prevented by arrest of bleeding (e.g. by
platelets and clotting factors) and maintenance of blood in the
fluid state occurs by the natural anticoagulants.

The core of medical physiology (1) – 3rd edition Page 133


Formation of blood (hematopoiesis)
 Sites of hematopoiesis
 During pregnancy, blood is formed at the following sites:
Yolk sac:
- Forms blood during the intrauterine life, first trimester
Liver & spleen:
- Form blood during the intrauterine life, second trimester
- After delivery, the liver and spleen may resume their
hemopoietic activity in children suffering from chronic anemia
[this is known as extramedullary hematopoiesis]
Bone marrow):
- Forms blood during the intrauterine life, third trimester (and
then after delivery); called red marrow (active marrow)
- Found in almost all bones during pregnancy
 After delivery, blood is formed at the following sites:
The red bone marrow
- After delivery, the red bone marrow starts to be replaced
gradually by yellow marrow (inactive marrow)
- At about the age of 20y the red marrow is confined to:
o Flat bones
- Skull, sternum, ribs, scapula, vertebrae & iliac bone
o Ends of long bones “epiphyses” not the shaft “diaphysis”
o Small bones of hands & feet
- About 75% of the cells in the red bone marrow are WBC precursors
and only 25% are RBC precursors; inspite of the fact that RBC count
is over 500 times more than the WBC count. This indicates the
longer life span of RBCs.

The core of medical physiology (1) – 3rd edition Page 134


THE RED BLOOD CELLS
Characteristics of RBCs
Shape: Biconcave disks
Diameter: 7.5 micrometer
Thickness: 1 micrometer (center), 2 micrometer (edges)
Fig 5.2:

Contents:
o No nucleus
- Therefore no reproduction
- Their life span is about 120 days
o No organelles
- e.g. no mitochondria and therefore no aerobic
glycolysis. However, they use anaerobic glycolysis to
generate small amount of energy for maintenance of
the integrity of the cell membrane
o Contain hemoglobin
- A red pigment found within RBCs
- Important for transport of oxygen
- The volume of a red blood cell is larger than its contents. This
allows it to squeeze itself through the narrow capillaries.

The core of medical physiology (1) – 3rd edition Page 135


Count of RBCs
- In males: 4.8-5.8 million/mm3 & in females: 4.2-5.2 million/mm3

Functions of RBCs
 Transport oxygen (bound to iron in hemoglobin)
 Transport carbon dioxide (bound to globin in hemoglobin)
 Buffer (a function of hemoglobin)
 Contain antigens on its surface that determine the type of blood
group

Stages of erythropoiesis
Can be divided into:
 Replication phase
o The increase in number of RBCs
 Maturation phase
o The decrease in size of RBC precursors
o Loss of contents (nucleus and organelles)
o Acquisition of hemoglobin
Stages in the bone marrow:
 Pluripotent non-committed stem cells
- Nucleated cells that divide to form 2 major types of committed
stem cells: lymphoid and non-lymphoid (myeloid & erythroid) cells
 Committed stem cells (or progenitor cells)
- Nucleated cells that divide to form only one type of blood cells
(e.g. the lymphoid forms lymphocytes and the erythroid forms
RBCs, and the myeloid forms other WBCs “granulocytes,
monocytes” and precursors of platelets “megakaryocytes”).

The core of medical physiology (1) – 3rd edition Page 136


 Proerythroblasts (or Normoblasts)
- The erythroid committed stem cells divide to form nucleated
cells known as proerythroblasts (or normoblasts).
- These divide many times and start to form hemoglobin; that’s
why they are described as early, intermediate & late or according
to the stain as basophilic and polychromatophilic.
- The normoblasts lose the nucleus & other organelles to become
reticulocytes.
Stages in the blood:
 Reticulocytes
- The first non-nucleated red blood cell precursors.
- They have no organelles except some ribosomes (so they are
still able to synthesize hemoglobin from ribosomal RNA).
- After their formation they stay in the bone marrow for 1-2 days;
then they enter the peripheral blood for the first time and stay for
another 1-2 days before they become mature erythrocytes.
- They constitute about 1% of total RBCs.
- High reticulocyte count (>2% = reticulocytosis) indicates
hyperactivity in the bone marrow (i.e. rapid synthesis of RBCs to
replace the lost ones) as occurs in:
o Hemolytic anemia (reticulocytosis indicates RBC hemolysis)
o Recent treatment of anemia (e.g. following vitamin B12
injections)
 Erythrocytes
- The mature red blood cells.

The core of medical physiology (1) – 3rd edition Page 137


CONTROL OF ERYTHROPOIESIS
Normal erythropoiesis requires:
- Normal bone marrow
- Cytokines and colony stimulating factors
- Erythropoietin & other hormones (e.g. thyroid hormones)
- Certain nutrients (e.g. iron, vitamin B12 and folic acid)
 Normal bone marrow
- Normal bone marrow is essential for normal erythropoiesis.
- Diseases affecting the bone marrow usually results in abnormal
erythropoiesis (see anemia).
 Cytokines and colony stimulating factors
- Interleukins: IL-1, 1L-3, IL-6 and GM-CSF (Granulocyte- Monocyte
Colony Stimulating Factors) stimulate proliferation and conversion of
pluripotent stem cells into committed stem cells.
- These growth factors are produced by bone marrow stromal cells
such as fibroblasts, endothelial cells, activated T cells and
macrophages.
 Erythropoietin (= the principal regulator of erythropoiesis)
Characteristics:
- Glycoprotein (contains 24% carbohydrates and 76% protein).
- Have single chain with 165 amino acids.
- MWt: 34000.
Site of production:
- In adults: produced mainly in the kidneys (90%) & in the liver (10%).
- In fetal life: the liver is the main site of production.
Stimulus:
- Hypoxia (low oxygen at the tissue level).

The core of medical physiology (1) – 3rd edition Page 138


Function
- Stimulates replication of stem cells and formation of erythroblasts.
- Erythropoietin dose not influence the maturation stages of RBCs.
Abnormalities
 Erythropoietin excess (= results in polycythemia)
- Caused by:
o High altitudes: people living at high altitudes have physiological
polycythemia due to the low oxygen tension at these sites.
o Renal problems (e.g. renal tumors or polycystic kidney disease):
these pathological problems are characterized by excessive
release of erythropoietin causing pathological polycythemia.
o Chronic respiratory problems: When these problems cause
hypoxia, patients develop polycythemia (e.g. lung fibrosis).
 - Erythropoietin deficiency (= results in anemia)
- Anemia caused by erythropoietin deficiency is characterized by low
count of RBCs but with normal shape and color “normocytic
normochromic anemia”. The chief cause is chronic renal failure.
 Other hormones needed for normal erythropoiesis:
o Androgens (= male hormones that increase bulk of muscles, e.g.
testosterone). Muscles are more active than fats, they consume
more oxygen. This stimulates erythropoietin production and
therefore RBC formation. That’s why RBC count is higher in
males than females of the same age (notice that this difference
between males and females is absent before puberty!).
o Growth hormone
o Cortisol
o Thyroid hormones

The core of medical physiology (1) – 3rd edition Page 139


 Nutrients needed for erythropoiesis
o Amino acids: for formation of globin (Hb)
o Iron: for formation of heme (Hb). Its deficiency causes microcytic
hypochromic anemia
o Vitamin B12 & folic acid: for DNA synthesis (cell division).
Deficiency causes macrocytic normochromic anemia
o Vitamin C: for iron absorption (it is a reducing factor that
maintains iron in the ferrous state)
o Other B vitamins: Co factors in the metabolic reactions
o Vitamin E: for maintenance of RBC membrane (an anti-oxidant
that protects the cell membrane from oxygen radicals). Deficiency
causes hemolytic anemia.
o Trace elements:
- Copper: promotes iron absorption
- Cobalt: Needed for vitamin B12 synthesis in animals. It may
stimulate erythropoietin production.
Iron metabolism
 Distribution in the body
- Total amount in the body is about 3-5 grams. Distributed in:
o Heme of hemoglobin (65-75%)
o Cellular enzymes (catalase, cytochrome oxidase) (3%)
o Storage (ferritin, hemosiderin) (>20%)
o Plasma (transferrin) (< 1%)
 Dietary sources
- Animal products: e.g. liver, meat, egg
- Plant products: e.g. vegetables, beans
- The average diet provides 10-20 mg/day

The core of medical physiology (1) – 3rd edition Page 140


 Daily requirements
- About 1 mg/day in an adult man, 2 mg/day in an adult woman
- The daily requirements are increased during:
o Pregnancy
o Lactation
o Infancy (the first year of life)
 Absorption
- Amount absorbed = 10-15% of ingested iron; however, it is highly
increased in patients suffering from iron deficiency anemia.
- Site of absorption: upper small intestine (duodenum).
- Mechanism of absorption: 2ndary active transport (Fe2+-H+ symport).
- Factors that increase absorption:
o Ferrous state (Fe2+)
o Gastric acidity (HCl), maintains iron in the ferrous state
o Vitamin C, also maintains iron in the ferrous state
o Iron in heme
- Factors that decrease absorption:
o Presence of phosphates, oxalates or phytates in diet
o Achlorohydria (low hydrochloric acid due to gastric atrophy)
o Diarrhea
o Clay
o Fever
o Tetracycline
 Iron in plasma
- Concentration= 100-130 microgram/dL
- Transported in plasma by a beta globulin known as transferrrin
- Transferrin is normally 30- 40% saturated with iron

The core of medical physiology (1) – 3rd edition Page 141


 Storage
- Sites of storage: Intestinal mucosa, liver, spleen and bone marrow.
- Forms of storage: ferritin &hemosiderin.
- Ferritin is the main form of storage. It stores 65% of stored iron in a
soluble form that’s easily mobilized whereas hemosiderin stores 35%
of stored iron in an insoluble form that’s slowly mobilized.
 Function of iron: formation of heme in Hb
- Iron found in ferrous (Fe2+) form
- Each Hb molecule contains 4 (Fe2+)
- Each atom of iron binds 2 atoms of oxygen (Therefore each Hb
molecule carries 8 atoms of oxygen (or four molecules).
 Excretion
- Total daily loss of iron= 1 mg/day in adult men, 2mg/day in adult
women (i.e. daily intake = daily loss).
- Routes for iron loss: Feces, skin, urine and menstruation
 Abnormalities
o Iron deficiency anemia= (microcytic hypochromic anemia)
- Characterized by: Low Hb, small & pale RBCs (microcytic
hypochromic anemia), low concentration of iron in plasma, low
plasma ferritin and high total iron binding capacity (TIBC).
o Iron overload: 1о: Hemochromatosis & 2о: Hemosiderosis
- Hemochromatosis is a common hereditary condition characterized
by excessive iron absorption resulting in abnormal accumulation of
iron in many organs causing damage (e.g. liver cirrhosis, gonadal
atrophy, cardiomyopathy, skin pigmentation and diabetes mellitus).
- Hemosiderosis describes iron overload secondary to certain causes
(e.g. repeated blood transfusions and excessive iron therapy).

The core of medical physiology (1) – 3rd edition Page 142


Vitamin B12
- Cyanocobalamin
- Water soluble vitamin
 Sources and requirement
- Found in food of animal origin (notice that vitamin B12 is formed by
micro-organisms that contaminate animal tissues, it is not found in
food of plant origin unless it is contaminated by these
microorganisms).
- Daily requirements are very low (about 1- 2 microgram/day)
 Site of absorption
- Terminal ileum
- Absorption is assisted by the intrinsic factor (produced by parietal
cells in the stomach). This binds iron in the stomach and travels with
it to the terminal ileum to be absorbed together by pinocytosis.
 Storage
- In the liver, the amount stored is enough for at least 3 years without
intake.
 Functions
o DNA synthesis: (needed for red blood cell replication)
o Myelination of nerves
 Abnormalities
o Deficiency causes:
- Macrocytic normochromic anemia (megaloblastic anemia)
- Sub-acute combined degeneration of the cord (demyelination)
o Causes of deficiency:
- Low intake (rare cause of deficiency except in strict vegetarians)
- Resection of the terminal ileum (decreases absorption)

The core of medical physiology (1) – 3rd edition Page 143


- Gastrectomy (removal of the stomach decreases absorption
because there is no intrinsic factor).
- Gastric atrophy (caused by auto-antibodies that attack parietal
cells or the intrinsic factor); this is known as pernicious anemia.

Folic acid
- Water soluble vitamin
 Sources and requirement
- Found mainly in food of plant origin
- Daily requirement is about 100 microgram/day
- Requirements are increased during:
o Pregnancy
o Lactation
o Infancy
 Absorption
- In the small intestine (especially the jejunum)
 Function
- DNA synthesis (needed for red blood cell replication)
 Abnormalities
- Deficiency causes: macrocytic normochromic anemia
- Causes of deficiency:
o Decreased intake: malnutrition
o Decreased absorption: villous atrophy
o Increased requirements:
- Pregnancy
- Drugs (e.g. methotrexate)

The core of medical physiology (1) – 3rd edition Page 144


HEMOGLOBIN
- The red pigment found in RBCs.
- Molecular weight is about 65000.
 Concentration
o 14-16 g/dL in adult males
o 13-15 g/dl in adult females
o 18-20 g/dl in neonates
 Structure
- Consists of 4 subunits. Each subunit consists of heme + a
polypeptide chain. [Hb = 4 heme + 4 polypeptide chains]
- Each heme is synthesized from glycine and succinyl- CoA. It has a
porphyrin ring containing iron in the reduced state (Fe2+).
- Each polypeptide chain is identical to another chain in Hb; therefore
the 4 polypeptide chains are 2 pairs; e.g. 2 alpha + 2 beta chains or 2
alpha + 2 gamma chains or 2 alpha + 2 delta chains. The 4 chains
are called collectively globin (i.e. Hb = 4 heme + globin).
Fig 5.3: The four subunits of hemoglobin

The core of medical physiology (1) – 3rd edition Page 145


 Functions of Hb
o Carries 98% of oxygen in the blood
o Carries some carbon dioxide
o Buffer
 Normal types of Hb
 Hb A
- Adult Hb: constitutes about 98% of all Hb in adults
- Contains 2 alpha & 2 beta chains (α2β2)
- Each alpha chain contains 141 aa (formed by genes on
chromosome 16) whereas each beta chain contains 146 aa (formed
by genes on chromosome 11)
 Hb A2
- Adult Hb: constitutes about 2.5% of all Hb in adults
- Consists of 2 alpha & 2 delta chains (α2δ2)
- Delta chain is similar to beta chain, but differs in 10 aa. It is also
formed by genes on chromosome 11
 Hb F
- Fetal Hb
- Starts to be replaced Hb A before birth.
- At birth, it constitutes about 70% of total Hb. This decrease to less
than 2% (i.e. replacement is almost complete) by the age of 6 month.
- Hb F has higher affinity to oxygen than Hb A. This is because it
binds 2,3 DPG (di-phospho-glycerate) less avidly (in other words: 2,3
DPG does not occupy the sites available for oxygen binding as
occurs in Hb A.
- This facilitates delivery of oxygen from the mother to her fetus
through the placental membranes.

The core of medical physiology (1) – 3rd edition Page 146


- Consists of 2 alpha & 2 gamma chains (α2γ2)
- Gamma chain is similar to beta chain except in 37 aa. It is also
formed by genes on chromosome 11.
 Hb A1c (Glycated Hb)
- Subtype of Hb A
- Its terminal amino acid (valine) can bind glucose
- Normally it constitutes about 5% of total Hb
- Higher percentage of Hb A1c indicates poorly controlled diabetes
mellitus
 Embryonic hemoglobins
- Types of hemoglobin during the first 8 weeks of pregnancy include:
 Gower 1 Hb: consists of 2 zeta and 2 epsilon chains (ξ2ε2)
 Gower 2 Hb: consists of 2 alpha and 2 epsilon chains (α2ε2)
 Portland Hb: consists of 2 zeta and 2 gamma chains (ξ2γ2)
 Abnormal types of Hb
 Hb S
- Sickle cell Hb
- It is Hb A but the amino acid number 6 in beta chains (glutamic
acid) is replaced by (valine)
- Hb S precipitates in cases of hypoxia. This changes the shape of
RBCs to a sickle shape
- Complications of the sickle shape:
o Hemolytic crisis: hemolytic anemia known as sickle cell
anemia
o Jaundice: pre-hepatic jaundice due to the excessive hemolysis
o Vaso-occlusive crisis: obstruction of small capillaries causing
atrophy of organs supplied with these blood vessels

The core of medical physiology (1) – 3rd edition Page 147


- Symptoms appear after the age of 6 month when HbF is almost
totally replaced by Hb A (This is because Hb F prevents sickling of
RBCs since it does not contain beta chains)
- The disease is less severe in heterozygotes (who have one
chromosome affected and therefore 50% of their Hb is Hb S) than
homozygotes (who have defect in the two chromosomes that form
Hb A and therefore most of their Hb is Hb S).
Notes to remember about Hb S & sickle cell disease:
 In these patients, Hb F persists in high concentrations after the
age of 6 month and until adulthood as compensation.
 The disease in heterozygotes is known as sickle cell trait whereas
in homozygotes as sickle cell disease
 Sickle cell disease is a well known problem in certain tribes in
Western Sudan and Western Africa. It exists especially in malaria
endemic areas; this is explained by the fact that hemoglobin S
cannot be digested by malaria parasites (i.e. Hb S patients “the
sicklers” are rarely infected with malaria).
 Thalassemia
- Decreased production of globin chains; e.g. deficiency of alpha
chains (alpha thalassemia) or beta chains (beta thalassemia).
- Thalassemia is characterized by excessive hemolysis of RBCs
resulting in hemolytic anemia.
- The type of anemia is microcytic hypochromic anemia.
 Other abnormal types of hemoglobin
- Are numerous. They include Hb C, E, D and H.
- The abnormal types of Hb can be identified by electrophoresis (use
of an electrical current to separate normal and abnormal types of

The core of medical physiology (1) – 3rd edition Page 148


hemoglobin in the blood, because they have different electrical
charges and different rates of movement).
Remember the following abnormal conditions of Hb:
 Methemoglobin
- Here iron is oxidized (from the ferrous state (Fe2+) to a ferric state
(Fe3+)) by some oxidizing agents (e.g. drugs). Normally this is
converted back by NADH- Methemoglobin Reductase System within
the RBCs. Congenital absence of this enzyme causes hereditary
methemoglonbinemia.
- The ferric state can not bind oxygen in a reversible reaction as the
ferrous state.
- Methemoglobin gives the skin a dark color resembling cyanosis.
 Carboxyhemoglobin
- Formed by reaction of hemoglobin with carbon monoxide.
- The affinity of Hb for CO is very high. That’s why it displaces
oxygen from its sites resulting in reduction of oxygen carrying
capacity of the blood (= anemic hypoxia).

RBC DESTRUCTION
Stages of destruction
- Every second, a large number of RBCs are broken down in the
tissue macrophage system (previously known as the
reticuloendothelial system); e.g. the spleen.
- At the same time the same number is replaced by new RBCs
produced in the bone marrow.
- Anemia occurs if breakdown is more than production whereas
polycythemia occurs if breakdown is less than production.

The core of medical physiology (1) – 3rd edition Page 149


In the spleen (the main site of RBC destruction)
- Old and abnormally shaped RBCs that cannot pass through small
capillaries in the spleen are destroyed by macrophages and the main
product of destruction is hemoglobin which gives heme and globin.
- Globin is hydrolyzed into amino acids which are reutilized.
- Heme releases iron which is needed by the body for synthesis of
new RBCs. The remaining porphyrin ring is converted to biliverdin
and then to bilirubin.
- Bilirubin, a yellow pigment that’s water insoluble, is transported
towards the liver carried by albumin.
- Carriage of the insoluble bilirubin in the blood by plasma proteins
prevents its appearance in urine (can not be filtered).
In the liver and bile
- In the liver, bilirubin is conjugated to glucouronic acid to be water
soluble.
- Conjugation is catalyzed by the enzyme glucouronyl transferase.
- Conjugated bilirubin is then secreted in bile. It gives bile its
yellowish color.
- For this reason it is called bile pigment. However, some of it
escapes into the systemic circulation and appears normally in urine.
In the intestine
- When bile reaches the intestine, bilirubin is converted by bacteria to
stercobilinogen & urobilinogen (both called urobilinogens).
- Stercobilinogen is excreted in stool, giving it its characteristic color.
- Urobilinogen is absorbed to the entero-hepatic circulation. However,
some of it escapes into the systemic circulation and appears
normally in urine.

The core of medical physiology (1) – 3rd edition Page 150


Fig 5.4: Excretion of bilirubin

Jaundice
- Yellowish coloration of the skin, sclera & mucus membranes.
- Occurs due to high level of bilirubin in the plasma (normal bilirubin
level is about 1mg/dL).
- Jaundice appears when the concentration is higher than 2mg/dL
Types of Jaundice
o Pre-hepatic (hemolytic)
o Hepatic
o Post-hepatic jaundice (obstructive)
 Pre-hepatic Jaundice
- Caused by excessive hemolysis of RBCs resulting in excessive
release of unconjugated bilirubin (gives indirect reaction in the van
den Bergh test).
- The high amount of unconjugated bilirubin exceeds the capacity of
the liver for conjugation and excretion into bile. For this reason the

The core of medical physiology (1) – 3rd edition Page 151


unconjugated bilirubin appears in blood causing the yellowish color.
However, it does not appear in urine because it is bound to albumin
in the blood (acholuric jaundice).
- Conjugated bilirubin that’s secreted by the liver in bile is higher than
normal. Therefore production of urobilinogens in the intestine and
consequently its excretion in urine are also higher than normal.
- Prehepatic jaundice is usually associated with hemolytic anemia.
- Causes include:
 Abnormal RBC shape
o Sickle cell anemia (due to Hb S)
o Congenital spherocytosis (Due to lack of spectrin)
o Congenital elliptocytosis (due to lack of spectrin)
 Lack of RBC enzymes
o Glucose 6 phosphate dehydrogenase deficiency
o Pyruvate kinase deficiency
 Mechanical destruction of RBCs
o Malaria
o Hypersplenism
 Other causes like T
o Thalassemia
o Autoimmune hemolytic anemia …
 Hepatic Jaundice
- Caused by liver cell damage or lack of enzymes resulting in failure
of bilirubin conjugation in liver cells (thus increasing unconjugated
bilirubin in the blood) or failure of bilirubin secretion in bile after its
conjugation (thus increasing conjugated bilirubin in the blood).
- i.e. Bilirubin in hepatic jaundice may conjugated or unconjugated.

The core of medical physiology (1) – 3rd edition Page 152


- Conjugated bilirubin appears in urine whereas unconjugated
bilirubin does not (because it is bound to albumin).
- Causes of hepatic jaundice include:
o Viral hepatitis (e.g. A, B, C, …) or alcoholic liver disease; cause
cell damage = failure of conjugation = unconjugated jaundice
o Dubin Johnson & Rotor syndromes: due congenital defect in the
mechanism of bilirubin secretion in bile = conjugated jaundice
o Crigler-Najjar syndrome (type I & II); due to congenital deficiency
of the conjugating enzyme = unconjugated jaundice
o Gilbert's syndrome; due to decreased activity of the conjugating
enzyme= one of the commonest causes of unconjugated jaundice
 Post-hepatic Jaundice
- Caused by obstruction of the biliary tracts. This prevents conjugated
bilirubin from reaching the intestine. Therefore it appears in blood
causing the characteristic yellowish color.
- The conjugated bilirubin, which is water soluble, appears in urine
(Choluric jaundice). It gives urine a characteristic dark yellowish
color. Urobilinogen is not produced in the intestine and so it is absent
from urine.
- Since bile does not reach the intestine, stool becomes pale (due to
absent bile pigments like bilirubin) and full of fat (due to absent bile
salts which are essential for fat digestion and absorption).
- Causes of obstructive jaundice include:
o Gall stones obstructing the common bile duct
o Parasites obstructing the common bile duct
o Tumors obstructing the common bile duct (e.g. carcinoma head of
the pancreas)

The core of medical physiology (1) – 3rd edition Page 153


ANEMIA
Definition
- State of reduction in hemoglobin concentration below the normal
range (according to age and sex)
- This occurs because of:
o low Hb within each RBC (and/or)
o low total count of RBCs
Classification of anemia
o Decreased production of RBCs
o Increased destruction of RBCs
o Blood loss
1- Anemia due to decreased RBC production
 Diseases of the bone marrow
- Examples include:
o Bone marrow infiltration
- Tumors (primary tumors like leukemias or secondary
tumors from other sources like the bone or the thyroid)
- Myelofibrosis
o Bone marrow failure (Aplastic anemia)
- Congenital (Fanconi's anemia)
- Drugs (e.g. chloramphenicol)
 Lack of nutrients (Nutritional anemias)
- Examples include:
o Iron deficiency anemia
o Vitamin B12 deficiency anemia
o Folic acid deficiency anemia
o Deficiency of proteins

The core of medical physiology (1) – 3rd edition Page 154


 Lack of hormones
- Examples include:
o Erythropoietin deficiency (e.g. chronic renal failure)
2- Anemia due to increased RBC destruction
- Causes include:
 Abnormal RBC shape
o Sickle cell anemia (due to Hb S)
o Congenital spherocytosis (Due to lack of spectrin)
o Congenital elliptocytosis (due to lack of spectrin)
 Lack of RBC enzymes
o Glucose 6 phosphate dehydrogenase deficiency
o Pyruvate kinase deficiency
 Mechanical destruction of RBCs
o Malaria
o Hypersplenism
 Other causes
o Thalassemia, Autoimmune hemolytic anemia …
3- Anemia of blood loss
 Acute blood loss
- Acute bleeding occurs following trauma
- The drop in Hb concentration does not occur immediately. It
occurs after about 3 days (i.e. after restoration of plasma
volume and before restoration of cells). The cells take about 4
weeks to be restored.
- Severity of anemia is best assessed by history (information
given by the patient or his relatives about the blood loss).
 Chronic blood loss

The core of medical physiology (1) – 3rd edition Page 155


- Occurs due to daily loss of small amount of blood over long
period of time. This results in iron deficiency anemia (due to
loss of iron with blood; about 0.5 mg iron in each ml of blood).
- Examples include: Worms like ankylostoma (the hook worm),
chronic peptic ulcer and bleeding from a GIT tumor.

Diagnosis of anemia
Step 1 (symptoms):
- Anemia is suspected when patients complain of:
o Fatigability, palpitations, breathlessness and headache.
Remember that: Mild anemia and even severe anemia that’s
developed gradually over long time, may be asymptomatic due to
compensatory mechanisms.
Step 2 (signs):
- Doctors do clinical examination looking for:
o Pallor of mucus membranes
o Tachycardia
o High pulse pressure (increased systolic and decreased
diastolic pressure)
o Signs of the cause or signs of complications
- During clinical examination, doctors look for signs of a cause
that may be responsible for the anemia (e.g. jaundice
indicates hemolytic anemia and severe loss of weight
indicates malignancy). At the same time they look for a
possible complication of anemia, especially if it is severe. A
well known example is anemic heart failure presenting with
generalized edema.

The core of medical physiology (1) – 3rd edition Page 156


Step 3 (investigations):
 Investigations are required to confirm the diagnosis of anemia.
These include: Hb estimation, PCV and RBC count
 Investigations are also needed to know the cause of anemia.
Examples include:
o Peripheral blood picture
- The shape and color of RBCs under the microscope may give
clue about the cause of anemia, look at this table:
Table 5.1:
Shape of RBCs Color of RBCs Possible causes
Microcytic Hypochromic Iron deficiency
Thalassemia
Macrocytic Normochromic Vitamin B12 deficiency
(Megaloblastic) Folic acid deficiency
Normocytic Normochromic Anemia of chronic illness
Chronic renal failure

- Direct microscopy may give additional information. For example


fragmented RBCs indicate hemolysis.
- High reticulocyte count and bilirubin level in plasma indicate
hemolytic anemia.
 RBC indices
- Can be calculated to give similar information as the microscopic
peripheral blood picture (i.e. information about the shape and color of
RBCs). - They include the following indices:
MCV (Mean corpuscular volume)
= PCV x 10/RBC; normally = 80 – 95 fL (may be up to 100 fL)

The core of medical physiology (1) – 3rd edition Page 157


- RBCs with higher results are described as macrocytic, lower
results are microcytic and normal results are normocytic.
MCH (Mean corpuscular hemoglobin)
= Hb x 10/RBC
- Normally = 25-32 pg
- RBCs with lower results are described as hypochromic while
normal results are normochromic.
MCHC (Mean corpuscular hemoglobin concentration)
= Hb x 100/PCV
- Normally = 34% (or 34 g/dL) range: 32-36 g/dL
- RBCs with lower results are described as hypochromic while
normal results are normochromic.
 Investigations for specific causes
- For example:
o Investigations for iron deficiency like TIBC, iron concentration and
ferritin level in plasma (see iron metabolism).
o Investigations for chronic renal failure like serum creatinine, urea
and electrolytes.
o Investigations for leukemia like total WBC count and differential
count.
Treatment of anemia
1- Treatment of the cause (e.g. stop the site of bleeding)
2- Replacement of the deficient nutrient (e.g. iron and/or folic acid)
3- Correction of the low Hb by blood transfusion (if Hb is very low).
This is important to prevent anemic heart failure. However, if heart
failure has already developed, it is preferred to transfuse packed
cells rather than whole blood, to avoid additional fluid overload.

The core of medical physiology (1) – 3rd edition Page 158


QUESTIONS FOR SELF ASSESSMENT-6 (BEST OF FIVE)

1. A male fetus during the second trimester of intra-uterine life has:


a. about 60% of his total body weight as water
b. red blood cells synthesized in the liver
c. “Gower” type of haemoglobin
d. anti A antibodies if he is blood group B
e. anti D antibodies if he is rhesus negative
2. All the following about haemoglobin s are correct except:
a. it consists of two alpha and two beta chains
b. its alpha chains are normal
c. it is absent in all neonates carrying the genes of sickle cell
disease
d. it precipitates due to hypoxia causing “sickling” of leucocytes
e. it results in sickle cell anemia
3. Iron:
a. is found in high amounts in mother’s milk
b. is absorbed mainly in the terminal ileum
c. is transported in plasma with albumin
d. is not stored in the liver
e. deficiency causes reduction in volume and color of red blood
cells
4. Which of the following is NOT a characteristic of obstructive
jaundice:
a. pale fatty stool
b. normal urine color
c. absent urobilinogen from urine
d. high conjugated bilirubin in blood
e. bleeding tendency
5. The principal hormone needed for red blood cell formation is:
a. cortisol
b. thyroxin
c. vitamin D
d. erythropoietin
e. parathyroid hormone
6. Iron deficiency anemia is characterized by:
a. high PCV
b. aplastic bone marrow
c. pale fatty stool
d. low ferritin level in the plasma
e. macrocytic normochromic RBCs

The core of medical physiology (1) – 3rd edition Page 159


7. Concerning folic acid , it is:
a. a fat soluble vitamin
b. found in green leafy vegetable
c. absorbed better in presence of reducing factors
d. antagonized by vitamin B12
e. required for myelin formation
8. Hemolytic jaundice is characterized by:
a. high concentration of conjugated bilirubin in urine
b. high concentration of unconjugated bilirubin in plasma
c. absence of urobilinogen in urine
d. pale and fatty stool
e. abnormal function of the liver
9. The red bone marrow in an adult is found at all these sites except:
a. skull
b. ribs
c. pelvic bones
d. shaft of the femur
e. sternum
10. Reticulocytes are:
a. precursors of white blood cells
b. not present in normal blood
c. increased in bacterial infections
d. decreased in haemolytic anemia
e. increased following correction of nutritional anemia
11. The mature red blood cell in children:
a. is synthesized in the spleen
b. has few mitochondria
c. its volume increases in folic acid deficiency
d. of males has more hemoglobin than that of females
e. lives for about 8 hours
12. Concerning the packed cell volume (PCV), it is:
a. the percentage of RBCs out of total cells in the blood
b. higher in neonates than in infants
c. normally about 37% in the adult male
d. high in patients with haemolytic anemia
e. low in all subjects living at high altitude
13. Erythropoietin:
a. is stimulated by increased oxygen tension.
b. Is needed for Hb synthesis
c. reduces maturation time of red blood cells.
d. its level is always high in people living at high altitudes
e. its level is always low in patients with chronic kidney failure.

The core of medical physiology (1) – 3rd edition Page 160


14. Hb S:
a. is similar to Hb F
b. is characterized by abnormal alpha chains
c. starts to appear in normal children after the age of 6 months
d. precipitates when there is hypoxia
e. is an important protective mechanism against malaria
15. Regarding iron absorption, it is:
a. hormonally controlled
b. decreased by HCL
c. Increased by intrinsic factor of the stomach
d. decreased by penicillin
e. about 10% of daily ingested iron
16. Hyperactivity in the bone marrow is indicated by:
a. jaundice
b. anaemia
c. leucocytosis
d. thrombocytosis
e. reticulocytosis
17. Microcytic hypochromic anemia occurs due to:
a. acute renal failure
b. hepatic cirrhosis
c. congestive heart failure
d. chronic peptic ulcer
e. filariasis
18. Vitamin B12:
a. is found in green vegetables
b. is absorbed in the stomach
c. is needed for haemoglobin synthesis
d. deficiency causes normocytic normochromic anemia
e. deficiency results in demyelination neuropathy
19. Hb concentration in a pregnant lady is 9 g/dL, PCV is 36% and
RBC count is 3 x 106 cell/mm3. Which of the following is not true:
a. mean corpuscular hemoglobin is normal
b. iron status is probably normal
c. volume of red blood cells is above normal
d. this lady most probably suffers from vit. B12 deficiency
e. folate deficiency is expected in this patient

Question 1. 2. 3. 4. 5. 6. 7. 8. 9. 10.
Answer b d e b d d b b d e
Question 11. 12. 13. 14. 15. 16. 17. 18. 19.
Answer c b e d e e d e d

The core of medical physiology (1) – 3rd edition Page 161


LEUCOCYTES
Definition
- The motile units in the blood that protect the body against invaders.
Count
- 4000-11000/mm3
- Higher count (leucocytosis) is caused by leukemia, infections,
inflammation...
- Lower count (leucopenia) is caused by certain diseases of the bone
marrow and certain infections (e.g. typhoid fever).
Formation of WBCs
 Site: bone marrow
 Stages: (Proliferation & Maturation)
- Like other types of cells, formation starts by differentiation of
pluripotent stem cells into committed stem cells (progenitor cells).
- There are separate committed stem cells for lymphocytes,
basophils and eosinophils, whereas neutrophils and monocytes have
a common precursor.
 Regulation: by certain colony stimulating factors (GM-CSF) and
cytokines (IL3, IL4 and IL5).
Classification
- According to presence or absence of cytoplasmic granules, they are
classified into two major types:
o Granulocytes
o Agranulocytes
 The granulocytes:
- Have granulated cytoplasm
- Have polysegmented nucleus

The core of medical physiology (1) – 3rd edition Page 162


- known as poly-morpho-nuclear leucocytes [PMNL]).
- Have very short life span (Hours)
- Have 3 types (according to reaction of the cytoplasmic granules
with acidic or basic dyes):
1- Neutrophils: the granules react with basic and acidic dyes
2- Eosinophils (Acidophils):the granules react with acidic dyes (e.g.
eosin dye)
3- Basophils:the granules react with basic dyes (e.g. mythelene blue
dye)
 Neutrophils
- Constitute 50-70% of total WBCs (the most abundant type)
- Nucleus: segmented (3-5 lobes)
- Cytoplasm: granulated
- Granules: react with acidic and basic dyes, purple in color
Fig 5.5: Neutrophil

- Can phagocytose bacteria and then perform intracellular killing (see


below).
- Increased in:
o Bacterial infections
o Stressful stimuli
o Hormones (adrenaline/cortisol)

The core of medical physiology (1) – 3rd edition Page 163


 Eosinophils (Acidophils)
- Constitute 1-4% of total WBCs
- Nucleus: segmented (2-3 lobes)
- Cytoplasm: granulated
- Granules: bright red in color. React with acidic dyes (like eosin).
Fig 5.6: Eosinophil

- Increased in:
o Parasitic infections (attack parasites that are too large
to be phagocytosed).
o Allergic reactions (e.g. asthma).
- They release proteins, cytokines and chemokines that can induce
inflammation and kill microorganisms (e.g. Major basic protein and
Leukotriene C4)
 Basophils
- Constitute (0-0.4%) of total WBCs (i.e. they are the least abundant
WBCs)
- Nucleus: may be segmented but hidden by the granules
- Cytoplasm: granulated. The granules: rough, dense and dark blue
in color. They obscure the nucleus behind them.

The core of medical physiology (1) – 3rd edition Page 164


Fig 5.7: Basophil

- The granules contain heparin and histamine (similar to mast cells).


They react with basic dyes like methylene blue dye.
- They release their contents in response to stimulation from T
lymphocytes.
- They are essential in immediate hypersensitivity reactions
- Generally increased in allergic reactions (e.g. allergic rhinitis and
allergic dermatitis).

 Agranulocytes
- Generally have non-granulated cytoplasm (they may have few fine
granules).
- Have one nucleus (not segmented)
- Have longer life span (days, months or years)
- Two types: Monocytes and lymphocytes
 Monocytes
- Have larger diameter than other types of WBCs
- Constitute 2-8% of total WBCs
- Nucleus: kidney shaped
- Cytoplasm: not granulated
- They can phagocytose bacteria and kill it intracellularly

The core of medical physiology (1) – 3rd edition Page 165


Fig 5.8: Monocyte

- They stay in the circulation for (2-3 days) and then enter the tissues
where they transform into tissue macrophages. However, the life
span of tissue macrophages is uncertain; may be 3 months or more.
- Tissue macrophages include: Kupffer cells in the liver, Langerhans
cells in the skin, Microglia in the brain, Osteoclasts in bone and
Alveolar macrophages in the lung (= members of the mononuclear
phagocyte system, used to be known as reticulo-endothelial system).
- Functions of the tissue macrophages (or the phagocyte system):
 Phagocytosis (activated of phagocytes by T lymphocytes)
 Presentation of antigens to lymphocytes
 Release of cytokines and other substances (see immunity).
 Lymphocytes
- Constitute 20-40% of total WBCs
- Size: may be small or large
- Nucleus: rounded
- Cytoplasm: scanty and not granulated.

The core of medical physiology (1) – 3rd edition Page 166


Fig 5.9: Lymphocyte

- Before birth, they are produced in the bone marrow as lymphocyte


precursor cells.
- Then after birth, some still arise from bone marrow. However, most
lymphocytes are formed in the lymph nodes, thymus or spleen, from
the original precursor cells that developed in the bone marrow and
processed in one of the following sites: Thymus gland or Burza of
Fabricius (in birds) /or burza equivalent tissues like the fetal liver and
the bone marrow (in humans)
- Accordingly there are 2 types of lymphocytes:
- T-Lymphocytes (processed in the thymus gland) &
- B-Lymphocytes (processed in the Burza equivalent tissues).
The two types circulate between blood and lymph.
 T-Lymphocytes:
- Constitute about 80% of total lymphocytes
- Mature in the thymus gland
- Types: Helper T cells (CD4 cells) & cytotoxic T cells (CD8 cells).
- When activated, the T lymphocyte form memory T cells; for future
recognition of the invader (= the secondary immune response)
- They are responsible for cell mediated immunity (see below)

The core of medical physiology (1) – 3rd edition Page 167


 B-Lymphocytes:
- Constitute about 20% of total lymphocytes.
- Maturate in the bone marrow
- When activated, they differentiate into plasma cells (for production
of antibodies) and memory B cells (for future recognition of the
invader, = the secondary immune response)
- Responsible for humoral immunity; i.e. secretion of antibodies that
are mixed with body fluids (see below)

How can white blood cells perform their functions?


 Chemotaxis
- Attraction of neutrophils to the site of infection by certain
substances including components of the complement system (C5a),
leukotrienes and products of damaged tissues, bacteria or
leucocytes.
 Diapedisis
- Passage of white blood cells from the blood to the tissues through
the wall of capillaries (e.g. neutrophils, basophils, eosinophils and
monocytes).
 Opsonization
- Preparation for eating or making the foreign cell palatable or tasty
for phagocytosis. Opsonins that coat foreign cells include IgG and
some proteins of the complement system.
 Phagocytosis
- The process of ingestion of bacteria or any abnormal cell by a
phagocytic cell (e.g. neutrophils, Monocytes or macrophages).
- Preceded by recognition and followed by intracellular killing.

The core of medical physiology (1) – 3rd edition Page 168


 Intracellular killing
- Fusion of cytoplasmic granules in the phagocytic cell with the
phagocytic vacuole, then release of the antimicrobial agents found
within these granules into the vacuole.
- The antimicrobial agents found within the granules of neutrophils
include:
o Defensins
o Proteases (elastase, metalloproteinases)
- Other antimicrobial agents released by neutrophils:
o Toxic oxygen metabolites (O2- , H2O2)
o Oxidants (HOCl, HOBr, …)
- The toxic oxygen metabolites (O2- , H2O2) are oxidants formed
following activation of a cell membrane bound enzyme (NADPH
oxidase).
- It is associated with a process known as the respiratory burst,
characterized by marked increase in oxygen uptake and metabolism
in the neutrophil.
- Steps of formation:
NADPH + H+ + 2O2 = NADP+ + 2H+ + 2O2-
O2- + O2- + H+ + H+ = H2O2 + O2
(The second step is catalyzed by the enzyme superoxide dismutase
(SOD))
- Formation of the other oxidants (HOCl, HOBr) is catalyzed by the
enzyme myeloperoxidase.
- Notice that certain types of bacteria can resist intracellular killing
and remain viable within the WBCs.

The core of medical physiology (1) – 3rd edition Page 169


IMMUNITY
- The physiological process by which the body destroys or
neutralizes foreign particles. Includes two types:
o Non specific immunity (Innate immunity)
(Does not depend on recognition of the invader (i.e. non specific))
o Specific immunity
(Depends on recognition of the invader)
Non specific immunity (Innate immunity)
- The first line of defense against invaders. It includes:
 Natural barriers
 Inflammation
 Natural killer cells
 Complement system
 Natural barriers:
- Prevent invasion of tissues by microorganism. They include:
o Anatomical barriers (e.g. skin and mucus membranes)
o Biochemical barriers (e.g. saliva, tears and gastric acid)
o Mechanical barriers (e.g. cough and vomiting)
 Inflammation:
= The response of the body to infection. It acts to localize infection. It
involves:
o Attraction of WBCs to the site of infection
o Release of active substances by WBCs
o The active substances destroy the microorganisms. However,
they also damage tissues and result in changes in blood vessels
(e.g. vasodilatation and increased permeability).
- Signs of inflammation: redness, swelling, pain & loss of function.
The core of medical physiology (1) – 3rd edition Page 170
 Natural killer cells (= NK cells)
- A small fraction of circulating lymphocytes is neither T nor B
lymphocytes; most of these are natural killer cells (NK cells).
- They resemble cytotoxic lymphocytes in killing their targets by
releasing granules containing cytolytic enzymes (perforins).
However, they differ from them in that:
o They kill naturally (generally they do not need activation,
however, they may become activated by interferons α and β.
o They attack cells which lack class I MHC antigens on their
membranes; these include some virus infected cells (because
some viruses inhibit expression of MHC-I antigens) & tumor
cells (because these have low or no class I MHC expression).
 The complement system
- Group of proteins (30 or more), activated to produce a cell killing
effect.
- They include: (C1 (has 3 subunits: q, r and s), C2, C3a, C3b, C4,
C5, C6, C7, C8, C9, factor B, factor D, factor H, factor I …).
- Most of these proteins are inactive beta globulins (= 5% of all
globulins).
- They are activated by:
 Classic pathway
- Triggered by antigen-antibody complexes.
- The first factor to be activated is C1 (see fig 5.10).
 Alternative pathway
- Triggered by the surface of pathogens like bacteria or viruses.
- The first factor to be activated is C3 (see fig 5.10).
 Lectin pathway

The core of medical physiology (1) – 3rd edition Page 171


- Triggered by contact of lectin with certain carbohydrate groups
(mannose) in bacterial wall (mannose-binding lectin (MBL)).
- MBL acts like C1q it activates C2 & C4, then other steps follow.
Fig 5.10: Activation of the complement ystem

Mechanism of action:
- The cell killing effect of the complement proteins is achieved by:
 Opsonization (e.g. by C1q and C3b)
 Chemotaxis (e.g. by C5a)
 Cell lysis (by the perforating complex C5b6789)
 Degranulation of mast cells to release histamine (e.g. by C3a)

The core of medical physiology (1) – 3rd edition Page 172


Remember that:
- Complement proteins are tightly regulated by complement control
proteins that inhibit their damaging effects on the normal cells.
- Deficiency of some of the complement proteins may predispose to
repeated infections (e.g. meningitis).

Specific immunity
- Depends on the action of lymphocytes which, in addition to their
immediate defensive role, form memory cells for future recognition of
the invaders.
- This explains the fact that a secondary immune response (induced
by subsequent exposure to an antigen) is always more rapid and
stronger than a primary immune response (induced by first exposure
to the antigen).
- Antigens are presented to lymphocytes by antigen presenting cells
which, after phagocytosis, present some of the invader antigens on
their surface; to be recognized by the lymphocytes.
- The antigen presenting cells are:
o Dendritic cells (found in lymph nodes spleen and skin)
o Macrophages (in different tissues)
o Native B lymphocytes (can bind antigen directly through
IgM antibodies expressed on their surface. However, they
need T helper cells for full activation (see below)).
- After recognition of the antigen, the lymphocytes proliferate and
mediate two types of specific immunity:
o Cell mediated immunity (mediated through activation and
proliferation of T lymphocytes).

The core of medical physiology (1) – 3rd edition Page 173


o Humoral immunity (mediated through activation and
proliferation of B lymphocytes).
Cell mediated immunity
- T helper lymphocytes recognize antigens presented by the antigen
presenting cells when these antigens are linked with the major
histocompatibility proteins type II (MHC-II).
- Certain markers on the surface of T helper cells (known as cluster
of differentiation 4 or CD4) facilitate this recognition.

Fig 5.11: Proliferation of T helper cells

- The activated T helper cells release cytokines for activation of T


cytotoxic lymphocytes, natural killer cells and macrophages. These
attack primarily virus infected cells, intracellular bacteria, fungi,
protozoa and cancer cells. In addition they play an important role in
rejection of transplanted organs. The cytokines also activate the B
lymphocytes.

The core of medical physiology (1) – 3rd edition Page 174


- The Cytotoxic T cells can recognize antigens presented by the virus
infected cells when these antigens are linked with the major
histocompatibility proteins type I (MHC-I).
- Certain markers on the surface of cytotoxic T cells (known as
cluster of differentiation 8 (CD8)) facilitate this recognition. They kill
their targets by releasing proteins known as “perforins” into them.
These make pores into the virus infected cells and initiate apoptosis.
Fig 5.12: Proliferation of cytotoxic T cells

- It is important to notice that T helper lymphocytes are two types:


o T helper 1 (TH1): the principal cells activated in cellular
immunity
o T helper 2 (TH2): interacts with B lymphocytes in relation to
humeral immunity
- Examples of the cytokines released By T H1:
o IL-2 (activates lymphocyte proliferation (autocrine effect),
natural killer (NK) cells and macrophages)
o  Interferon (IFNγ activates macrophages and inhibits TH2)

The core of medical physiology (1) – 3rd edition Page 175


- Examples of the cytokines released by TH2:
o IL 4 (activates B lymphocytes to produce IgE antibodies)
o IL-5 (stimulates differentiation of eosinophils)
- Human immuno-deficiency virus (HIV) attacks CD4 cells (T helper
cells); resulting in failure of cytokine production and therefore failure
of activation and proliferation lymphocytes and macrophages. This
causes acquired immune-deficiency syndrome (AIDS) and renders
the body susceptible for infection (with even the nonpathogenic
bacteria) and cancer.
Humoral Immunity
- Activated B-lymphocytes transform into plasma cells and produce
large number of antibodies
- The antibodies attack the invaders in different ways (see below).
- Unlike cell mediated immunity, humoral immunity is directed mainly
against extracellular organisms like bacteria).
 Note:
- Proteins of the major histocompatibility complex (MHC) are
encoded by genes located on the short arm of chromosome 6. These
proteins are found on the outer surface of body cells and they are
unique for each person. In humans they are known as human
leukocyte antigens (HLA).
- The MHC genes are classified into 3 classes:
 Class I
o Encodes for MHC-I proteins, which are coupled with antigens
formed within the cell (e.g. formed by intracellular virus).
These proteins are found on all nucleated cells. They are
recognized by CD8 cells.

The core of medical physiology (1) – 3rd edition Page 176


 Class II
o Encodes for MHC-II proteins, which are coupled with antigens
of extracellular organisms like bacteria. The antigens are
selected after phagocytosis of bacteria and then presented in
association with MHC-II proteins. Therefore these proteins are
found on antigen presenting cells. They are recognized by
CD4 cells.
 Class III
o Encodes for some immunological proteins (e.g. some
cytokines and complement proteins).

The cytokines
- Cytokines are proteins produced by a wide variety of immune and
non-immune cell types.
- They are involved in regulation of growth, development, and
activation of immunity and in the mediation of inflammation.
- A single cytokine can be released by different cells and different
cytokines may have similar functions and can act on different cells.
- Actions of cytokines may be:
o Autocrine: the cytokine acts on the same cell that secretes it
o Paracrine: the cytokine acts on a nearby cell
o Endocrine: the cytokine circulates in blood and acts on distant
cells
- Cytokines have names that describe their targets or their functions.
For example, cytokines acting on leucocytes are interleukins (IL-1, 2,
3, 4 …); whereas cytokines stimulating hematopoiesis are colony
stimulating factors (granulocyte-monocyte colony-stimulating factor).

The core of medical physiology (1) – 3rd edition Page 177


- Chemokines are cytokines that regulate cell movement (e.g.
Chemotaxis which is regulated by the chemokines IL-8 & RANTES).

Table 5.2: Examples of some cytokines and their functions


Cytokine Cell source Function
IL-1 Macrophages Hematopoiesis
Epithelial cells Fever & hepatic acute phase proteins
Fibroblasts Activation of WBCs
IL-2 T cells Activation & proliferation of T cells
Activation of B cells, NK cells and
macrophages
IL-3 T cells Hematopoiesis
NK cells
IL-4 T cells Stimulation of B cells to secrete anti-
Mast cells bodies
Stimulation of TH2 proliferation
IL-5 T cells, mast cells Activation of eosinophils
& eosinophils
IL-6 B cells Fever & induction of acute phase
Epithelial cells proteins from the liver
Fibroblasts T & B cell differentiation
IL-8 Macrophages, Chemotaxis
WBCs, fibroblasts Release of histamine from basophils
& endothelial cells
IL-12 Macrophages Induces TH1 cell formation
Neutrophils
IFN α & β All cells Anti-viral activity. Stimulate T cell,
macrophage, and NK cell activity.
Direct anti-tumor effects

IFN γ T cells Regulates macrophage and NK cell


(interferon NK cells activations. Stimulates immunoglobulin
- γ) secretion by B cells
TNFα Macrophages Fever, anorexia, shock, capillary leak
(tumor Most WBCs Leukocyte cytotoxicity, enhanced NK cell
necrosis Epithelial cells function, acute phase protein synthesis,
factor α) Fibroblasts pro-inflammatory cytokine production
RANTES Macrophages, T Chemotaxis
cells, fibroblasts & Induces release of histamine from
basophils basophils

The core of medical physiology (1) – 3rd edition Page 178


The antibodies
- Gamma globulins, called immuno-globulins or (Ig); produced by
activated B-lymphocytes (known as plasma cells).
Structure of the basic unit of antibodies:
- 2 heavy chains & 2 light chains connected by disulfide bridges.
- Each chain consists of both variable (V) and constant (C) regions.
- The C regions form the “Fc portion” for cell binding whereas the V
regions form the “Fab portion” for antigen binding.
- The variable regions have variable sequences of amino-acids; this
makes the Fab portion in each immunoglobulin molecule differs from
Fab portions in other molecules. That’s why antibodies secreted
against specific antigen do not attack other antigens.
 Remember that: Each antibody attacks only one specific antigen
by its antigen binding site (Fab portion).

Fig 5.13: The basic structure of antibodies

The core of medical physiology (1) – 3rd edition Page 179


- The immunoglobulins (Ig) are classified according to the types of
heavy chains into 5 types: G, M, A, E and D; therefore they are
described as IgG, IgM, IgA, IgE and IgD.
 IgG:
- The most abundant (constitutes more than 75% of total
immunoglobulins in plasma).
- The smallest type (monomer: has two sites for binding antigens)
- The only type that can cross the placenta from the mother to the
fetus to protect him from micro-organisms (= passive immunity)
- IgG is the main antibody in the secondary immune response (i.e.
the immune response following exposure to an antigen for the
second time or more)
 IgM:
- The largest type
- Pentamer (5 of the basic units connected together)
- It is the main antibody in the primary immune response (i.e. the
immune response following exposure to an antigen for the first
time).
 The secondary immune response is more faster and stronger
than the primary immune response due to presence of memory
cells formed during the primary response. They immediately react
with the antigen and result in secretion of adequate amounts of
IgG antibodies against the antigen
 IgA:
- Monomer, dimer or trimer
- In addition to plasma, it is also found on the mucosal surfaces
(e.g. mucosa of the GIT, genitor-urinary and respiratory systems),

The core of medical physiology (1) – 3rd edition Page 180


and in body secretions like saliva, milk and tears. For this reason
it is called the secretory immunoglobulin.
- Plays an important defensive role at these sites against
colonization of mucosa by pathogens.
 IgE:
- After its formation during a primary immune response induced
by certain allergens, it becomes attached to membranes of mast
cells & basophils through the Fc portion. Then during a
subsequent exposure to the same allergen, it binds the allergen
through the Fab portion. This induces release of histamine from
these cells causing symptoms and signs of immediate
hypersensitivity reaction and sometimes a severe anaphylactic
shock due to profound vasodilatation.
- In addition to its role in allergic reactions, it also offers protection
against parasites (e.g. worms).
 IgD:
- Has unknown function; may be antigen antibody recognition
(may act as antigen binding receptor on B lymphocytes).
Mechanisms of action of the immunoglobulins
 Opsonization of bacteria to facilitate phagocytosis (e.g. IgG)
 Neutralization of some toxins
 Blocking attachment of some viruses and bacteria to cells
 Activation of the complement system (IgG and IgM antibodies)
 Agglutination of group of cells or substances together to prevent
their spread; to be captured by phagocytes in one action
 Precipitation of some antigen-antibody complexes
 Degranulation of mast cells and basophils (release of histamine)

The core of medical physiology (1) – 3rd edition Page 181


QUESTIONS FOR SELF ASSESSMENT-7 (BEST OF FIVE)
1. Concerning the white blood cells, which of the following is true:
a. neutrophils are increased in viral infections
b. basophils are increased in parasitic infections
c. eosinophils have kidney shaped nuclei
d. monocytes phagocytose bacteria
e. lymphocytes release histamine
2. B- Lymphocytes are:
a. phagocytic cells produced by the bone marrow
b. premature cells activated in the thymus gland
c. the dominant type of lymphocytes
d. responsible for humeral immunity
e. increased in bacterial infections
3. The major basic protein is:
a. found in the granules of eosinophils
b. involved in allergic reactions
c. an anti-parasitic substance
d. a cause of bronchospasm in asthmatic patients
e. all the above statements are true
4. Which of the following antibodies is found in mother’s milk:
a. IgA
b. IgM
c. IgG
d. IgD
e. IgE
5. A stained blood film of a patient with hypersplenism shows:
a. neutropenia with reticulocytosis
b. thrombocytopenia with erythroblastosis
c. anemia with neutrophilia
d. anemia with agranulocytosis
e. pancytopenia with reticulocytosis
6. Concerning natural killer cells, which of the following is not true:
a. they are non T, non B lymphocytes
b. they attack tumor cells
c. they attack antigens presented with class I MHC proteins
d. they kill virus infected cells by releasing perforins
e. they are similar to T-cytotoxic lymphocytes
7. B lymphocytes differ from T lymphocytes in that they:
a. have large oval nuclei
b. may act as antigen presenting cells
c. form memory cells
d. populate lymph nodes
e. play an important role in immunity

The core of medical physiology (1) – 3rd edition Page 182


8. Regarding T-Lymphocytes, which of the following is correct:
a. helper cells recognize antigens when presented in association with
MHC class I
b. cytotoxic cells expose CD4 molecules on their membranes
c. major cytokines are released by the CD8 cells
d. TNFα is immediately released by T helper cells after their activation
e. T lymphocytes constitute the majority of lymphocytes in circulation
9. Acute bacterial infections are characterized by high count of:
a. neutrophils
b. basophils
c. monocytes
d. lymphocytes
e. both a & c
10. A stained normal blood film shows:
a. 20% nuetrophils.
b. 4% basophils.
c. 8% lymphocytes.
d. 4% monocytes.
e. 11% eosinophils.
11. "T Lymphocytes" differ from "neutrophils" in all the following
except:
a. their cytoplasm is not granulated
b. they are not markedly increased in bacterial infections
c. they are attacked by the Human Immunodeficiency Virus (HIV)
d. they cannot produce antibodies
e. their nuclei are not segmented
12. Neutrophils:
a. have granules containing heparin
b. are produced in the thymus gland
c. can leave the blood and enter tissues
d. live up to three days
e. are increased in viral infections
13. Immunogloblin:
a. D has the largest size
b. E is found in milk
c. G is found mainly in primary immune response
d. M is the only type that activates complement proteins
e. A is found in the mucosa of the respiratory tract
14. Immunoglobulin M (IgM) is:
a. described as the secretory immunoglobulin
b. most abundant in the secondary immune response
c. the dominant type of immunoglobulin in plasma
d. the antibody type directed against antigens of the ABO system
e. found on surface of mast cells

The core of medical physiology (1) – 3rd edition Page 183


15. The following cytokine is not released by T lymphocytes:
a. IL-1
b. IL-2
c. IL-3
d. IL-4
e. IFNγ
16. Attraction of neutrophils to the site of infection occurs by:
a. phagocytosis
b. coagulation
c. diapedesis
d. agglutination
e. Chemotaxis
17. Phagocytosis is the major function of:
a. B lymphocytes
b. neutrophils
c. endothelial cells
d. basophils
e. eosinophils
18. Development of acquired immunity requires all the following
except:
a. B lymphocytes
b. T lymphocytes
c. Macrophages
d. Major histocompatibility complex
e. Basophils
19. This cell presents antigens to T helper with MHC class II proteins:
a. virus infected cell
b. tumor cell
c. dendritic cell
d. mast cell
e. NK cell
20. Which of the following factors is not a chemotactic agent:
a. IL-8
b. C5b
c. RANTES
d. C3a
e. Leukotrienes
Question o o o o o o o o o o
Answer d d e a e c b e a d
Question o o o o o o o o o o
Answer d c e d a e b e c d

The core of medical physiology (1) – 3rd edition Page 184


BLOOD GROUPS AND BLOOD TRANSFUSION
BLOOD GROUPS
= Antigens found on the surface of red blood cells
 Importance
- Blood transfusion
- Exclusion of paternity (paternity can be excluded by blood groups;
however, its confirmation requires DNA fingerprinting).
 Types
- There is large number of these antigens.
- The most important are: ABO system & rhesus system.
- Others (of less importance): MNS, Lutheran, Kell, Kidd, Duffy …
The ABO System
 Antigens (Agglutinogens): A & B
- Two antigens on surface of RBCs: A & B.
- Accordingly there are 4 blood groups:
 Blood group A has antigen A
 Blood group B has antigen B
 Blood group AB has both antigens A & B
 Blood group O has neither antigen A nor B.
-These antigens are also found in other tissues (salivary glands,
kidney, liver, lung, pancreas, testes and amniotic fluid).
- They are glycosphingolipids in RBCs but glycoproteins in the other
tissues.
- Basically, all groups have H antigens. In individuals with blood
group O, all the H antigens persist without conversion into antigen A
or B. In individuals with blood group A or B, the H antigens are
converted into antigens A or B respectively. This is achieved by

The core of medical physiology (1) – 3rd edition Page 185


adding N-acetylgalactosamine to H antigens in blood group A, or
adding galactose to H antigens in blood group B.
- A and B glycoproteins are secreted by tissue cells into the
circulation. Some individuals are “non-secretors”. They are
susceptible to a variety of infections because the glycoproteins may
bind to polysaccharides on cells and block attachment of bacteria to
these polysaccharides.
 Antibodies (Agglutinins): anti A & anti B
- Anti A and anti B antibodies (of IgM type) are acquired naturally
after birth by unknown mechanism.
- Suggested mechanisms for their formation in neonates after birth:
 Food contains antigens similar to A & B antigens. Ingestion and
absorption of these antigens induce the immune system to attack
them by antibodies that circulate in the blood as anti A & anti B
antibodies.
 Certain intestinal bacteria contain antigens similar to A & B
antigens. If these bacteria infect the neonate, the immune system
will form antibodies against the bacterial antigens. These
antibodies circulate as anti A and anti B antibodies.
- The most interesting fact is that: The body forms antibodies against
the ABO antigens not present on surface of red blood cells. For
example a subject with group A has antigen A on RBC membranes,
so his body forms antibodies against B antigen (anti B antibodies).
- Accordingly, plasma contains the following types of antibodies:
 Anti A antibodies in subjects with blood group B
 Anti B antibodies in subjects with blood group A
 Anti A & anti B antibodies in subjects with blood group O

The core of medical physiology (1) – 3rd edition Page 186


 No antibodies in subjects with blood group AB
 Donors (In blood transfusion)
- Selection of a donor depends on absence of antigens on RBCs of
the donor that may react with antibodies in plasma of the recipient
(i.e. reactions between donor’s cells & recipient’s plasma “or serum”).
- On the other hand, reaction between the donor’s plasma and
recipient’s cells is neglected. This is because the volume of blood
collected from the donor is about 500 ml and therefore the amount of
antibodies in the plasma of this blood is very small. In addition to
that, it will become diluted in the plasma of the recipient.
- Accordingly, donors are selected for each blood group as follows:
 Donors for blood group (A) are those with blood groups A & O
 Donors for blood group (B) are those with blood groups B & O
 Donors for group (AB) are those with blood groups A, B, AB, O
 Donors for blood group (O) are those with blood group O
Notes to remember about the donors:
 Subjects with blood group AB can receive blood from every
blood group (they are described universal recipients)
 Subjects with blood group O can give blood to every blood
group (they are described as universal donors).
 Inheritance: Autosomal dominant
- Blood group antigens are inherited from each parent as autosomal
dominant characters in chromosome 9p (i.e. A & B antigens are
inherited in the two copies of chr 9, from the father and the mother).
- Therefore, the genotype for each blood group will be as follows:
 Blood group A: the genotype is [AA] or [AO]
 Blood group B: the genotype is [BB] or [BO]

The core of medical physiology (1) – 3rd edition Page 187


 Blood group AB: the genotype is [AB]
 Blood group O: the genotype is [OO]
- If a father and his wife have blood groups A and B respectively,
their children may have any type of blood groups (A, B, AB or O).
But, if they are A and A, their children may have either A or O only.
 Distribution (%)
- The ABO blood groups are distributed as follows:
 Blood group A is found in about 42% of population
 Blood group B is found in about 8% of population
 Blood group AB is found in about 3% of population
 Blood group O is found in about 47% of population
Notes to remember about distribution of blood groups:
 The most dominant blood group is O (blood group of the
universal donors) and the least dominant is AB (blood group of
the universal recipients).

The Rhesus System


- First studied in the rhesus monkeys.
 Antigens: D
- Include antigens C, c, D, E, e.
- Unlike the ABO antigens, these proteins are only found on RBCs
(not found in other tissues).
- The most important of these antigens is antigen D (has the highest
antigenisty).
- Accordingly, there are (2) types of rhesus groups:
 Rh +ve (antigen D is present on RBC surface)
 Rh -ve (antigen D is absent from RBC surface)

The core of medical physiology (1) – 3rd edition Page 188


 Antibodies: anti D
- Unlike the anti A & anti B antibodies, antibodies of the rhesus
system (anti D antibodies) are of the IgG type (i.e. the smallest type
that can cross the placenta from maternal blood to fetal blood).
- Also unlike anti A and anti B antibodies, anti D antibodies are not
naturally acquired after birth.
- They develop only in (Rh -ve) subjects (never in Rh +ve subjects).
- Causes of their development in rhesus –ve subjects:
 Blood transfusion of Rh +ve blood to Rh –ve subjects
 Delivery of (Rh +ve) baby by Rh –ve mothers (During labor, the
placenta separates from the uterus resulting in bleeding and
entrance of some fetal blood in maternal circulation
 Donors
- Rh +ve subjects do not have anti D antibodies. They can receive
blood from Rh +ve & Rh –ve subjects.
- Rh -ve subjects do not have anti D antibodies unless they receive
Rh +ve blood. Therefore, they should receive Rh -ve blood only.
- During emergencies, Rh –ve blood is not always available; that’s
why rhesus –ve subjects may receive Rh +ve blood for the first time.
This induces formation of anti D antibodies in his plasma within 3
days after the transfusion.
- Therefore, any future transfusion of Rh +ve blood is contra-
indicated because it will be associated with severe reaction.
 Inheritance
- Autosomal dominant character in chromosome 1.
- Chrmosome 1 carries E or e plus D or without D (absence of D is
indicated as d) plus C or c. For example: Cde or cDe or CdE …

The core of medical physiology (1) – 3rd edition Page 189


- Since antigen D determines the rhesus group, genotype for each
group (chromosomes from father and mother) will be as follows:
 Rh +ve genotypes: DD or Dd
 Rh –ve genotype: dd
Remember that:
 A child of a rhesus +ve father & rhesus +ve mother can be
either Rh +ve or a Rh –ve (if the genotype of both father and
mother is “Dd”).
 A child of a rhesus -ve father & rhesus -ve mother should be
Rh –ve.
 Distribution (%)
- Percentage of subjects with Rh +ve blood group, in all populations,
is far higher than that of Rh –ve subjects.
- For example: 99% are Rh +ve & only 1% are Rh –ve in Europe. In
other communities, the percentage of rhesus positive subjects is
about 85% and 15% for Rh –ve subjects.
Remember that:
 In the ABO system of blood grouping, transfusion of a
mismatched blood (e.g. group A subject receiving blood group
B) is always associated with severe reaction.
 In the Rhesus system, blood transfusion reactions are:
- Never expected for Rh +ve subject receiving Rh –ve blood
- Not expected in the first blood transfusion for Rh -ve subject
receiving Rh +ve blood
- Always expected in subsequent transfusions for Rh –ve
subject receiving Rh +ve blood

The core of medical physiology (1) – 3rd edition Page 190


Hemolytic disease of the newborn
- Also known as erythroblastosis fetalis because the early stages of
RBCs (erythroblasts) appear in fetal blood together with reticulocytes
as compensation to the excessive hemolysis.
- It occurs in Rh +ve neonates of Rh –ve mothers.
- During delivery of the first Rh +ve neonate, some of his blood
enters maternal circulation. The delivered neonate is normal; but the
mother forms anti D antibodies of IgG type to get rid of the baby’s
RBCs (i.e. she becomes immunized).
- When she becomes pregnant again with a Rh +ve fetus, these
antibodies cross the placenta, enter the fetal circulation and attack
RBCs of the fetus who develops hemolytic anemia and jaundice.
- The hemolytic anemia is associated with hyperactive bone marrow
to replace the lost RBCs. This results in appearance of the early
stages of RBC in fetal blood (erythroblastosis & reticulocytosis).
- Anemia may become complicated by heart failure associated with
generalized edema (hydrops fetalis) and the fetus may die in utero.
- The jaundice is caused by unconjugated bilirubin which is insoluble
in water. It may precipitate in the basal ganglia in the brain causing
Kernicterus (characterized by motor problems and may be
associated with mental retardation).
- If the newborn is delivered with jaundice, he can be treated with:
 Phototherapy: converts the insoluble bilirubin to a water
soluble substance, lumirubin, that’s soluble (do not precipitate
in the brain) and can be excreted easily in urine
 Exchange transfusion: taking Rh +ve blood from the neonate
and giving him Rh –ve blood

The core of medical physiology (1) – 3rd edition Page 191


 Plasmapheresis: usage of a machine that extracts the
antibodies from blood of the neonate
- Prevention is better than cure. Here an injection of anti D antibodies
given to the mother immediately following labor and not more than 72
hours after it will attack fetal RBCs that carry the rhesus antigen and
therefore prevent formation of anti D antibodies by the mother
herself.
Remember that:
 Antibodies injected to a subject stay in the circulation for short
time and then disappears completely.
 Antibodies formed by a subject himself stay in the circulation
for very long time because memory cells continue to secrete
the antibodies for life.

- Sometimes, ABO incompatibility between the mother and the


neonate offers some natural protection against the hemolytic
disease. For example when the mother is blood group A and the
neonate is blood group B, here the naturally occurring anti B
antibodies in maternal blood attack RBCs of the neonate as soon as
they enter maternal circulation and therefore prevent formation of anti
D antibodies.

The core of medical physiology (1) – 3rd edition Page 192


Blood transfusion
 Precautions
- When blood is infused intravenously into patients, plasma of the
patients should not contain antibodies that may react with blood
group antigens on RBCs of the donor.
- To achieve this, suitable donors should be selected for each blood
group, and this is further confirmed by cross matching test.
- In cross matching test the plasma of the recipient (containing
antibodies) is mixed with the donor's blood and then examined under
the microscope for evidence of agglutination of the RBCs.
Agglutination indicates that the blood of the donor is not suitable for
the recipient.
- In spite of this, errors may occur and unfortunate patients may
receive mismatched blood by accident. For this reason, blood
transfusion should always be taken seriously, with certain
precautions, including release of a few drops at first to make sure
that there is no reaction and close follow up of the process, putting
life saving drugs at reach beside the patient.
 Complications of mismatched blood transfusion
 Agglutination of RBCs, followed by hemolysis:
o In mild cases, agglutination of RBCs may be complicated by post-
transfusion jaundice.
o In severe reaction, a few ml of blood are sufficient to cause
severe pain in the back or in other sites and tightness in the chest
(due to block of capillaries by the agglutinated RBCs). This is also
followed by hemolysis of the RBCs and release of hemoglobin in
plasma.

The core of medical physiology (1) – 3rd edition Page 193


 Acute renal failure:
o Some of the intravascular hemoglobin is filtered at the glomeruli
and excreted in urine (hemoglobinuria).
o Some precipitates in the renal tubule resulting, in addition to the
low blood pressure, in acute renal failure.
o Acute renal failure is characterized by low urine output, high urea
and high potassium ions. Death may occur in few days.
 Disseminated intravascular coagulation (DIC):
o In severe cases, antigen-antibody reactions may result in
activation of coagulation causing DIC and therefore fatal bleeding
due consumption of the clotting factors.
 Complications of compatible blood transfusion
o Hyperkalemia due to potassium released from hemolysed RBCs
during storage of blood
o Hypocalcemia due to excess anticoagulant (e.g. citrate, which is a
calcium chelating agent)
o Fever (febrile reaction) due to pyrogens and/or cytokines in the
donor’s blood
o Allergy (allergic reaction) due to allergens in the donor’s blood;
sometimes severe anaphylaxis may occur.
o Transmission of infection due to transfusion of infected blood (e.g.
HIV, Hepatitis B, malaria …)
o Air embolism
o Thrombophlebitis at the injection site
o Fluid overload especially in patients with heart failure
o Iron overload due to repeated blood transfusions (hemosiderosis)

The core of medical physiology (1) – 3rd edition Page 194


HEMOSTASIS
 Definition
- Prevention of blood loss by arrest of bleeding and maintenance of
blood in the fluid state.
 Mechanisms (or steps) of hemostasis following injury
o Vasospasm
o Formation of platelet plug
o Formation of blood clot (Coagulation)
o Fibrinolysis or fibrous tissue formation
 Vasospasm
- Occurs immediately following injury due to:
o Myogenic response (the vascular smooth muscle responds to
trauma by spasm)
o Humoral factors (serotonin & catecholamines like epinephrine
and norepinephrine cause vasoconstriction)
o Neural factors (neural reflex involving the sympathetic
neurons)
- The vasospasm is expected to be weak in cases of:
o Hypoxia
o Metabolic acidosis
o Small surface area (spasm caused by a knife is weaker than
that caused by a stone; because the surface area of blood
vessels injured by the knife is smaller than by the stone)
 Formation of platelet plug
- Aggregation of platelets to close a hole in the blood vessel
- Enough to close small holes in blood vessels
- Large holes need the third step: blood coagulation

The core of medical physiology (1) – 3rd edition Page 195


 Platelets (Thrombocytes)
- Small cells (2- 4µm in diameter)
- Count: (200,000 – 400,000/mm3)
- Life span: about 5 days (up to 10 days)
- Arise from stem cells & then megakaryocytes in the bone marrow.
- The cytoplasm of each megakaryocyte breaks down into large
number of platelets (i.e. they are non-nucleated).
- Formation is regulated by colony stimulating factors and a hormone
known as thrombopoietin, a protein released by the liver and kidneys.
- About one third are stored in the spleen. That’s why splenectomy
(removal of the spleen) is sometimes used for treatment of
thrombocytopenia to increase platelet count.
- Their membranes contain receptors for collagen and von Willebrand
factor (vWF); both are important for attachment of platelets to
collagen at sites of blood vessel injury. Also they have receptors for
ADP and fibrinogen.
- Platelets contain two types of granules:
o Alpha granules (contain protein substances like the clotting
factor number 13, platelet derived growth factor (PDGF) which
is a potent stimulus for wound healing & vWF which is
synthesized by megakaryocytes and also by endothelial cells)
o Dense granules (contain non-protein substances like serotonin
and ADP)
- Functions of platelets:
 Participates in the various mechanisms of hemostasis as follows:
o Vasospasm: they release a vasoconstrictor (serotonin)
o Platelet plug formation: major function of platelets (see below)

The core of medical physiology (1) – 3rd edition Page 196


o Blood coagulation (platelets release some clotting factors (e.g.
factor 13 and platelet factor 3 “phospholipids”)
 Clot retraction: after formation of a blood clot and incubation of
blood for half an hour or so, platelets contract because they
contain actin and myosin fibers in their cytoplasm. This decreases
the size of the clot (to about 10% of its original size), and
expresses serum (read table 5.2).
- Platelet deficiency (thrombocytopenia) results in increased bleeding
tendency. A count less than 50,000 cell/µL is associated with
hemorrhage after minor injuries and possibly multiple petechial
hemorrhages under the skin and mucus membranes
“thrombocytopenic purpura”; however, purpura is very common when
the platelet count is less than 20,000 cell/µL.
- On the other hand, high count of platelets (thrombocytosis)
predisposes to thrombotic problems.
Mechanism of platelet plug formation:
1. Platelets attach to collagen fibers (this step is facilitated by
collagen receptors on platelets & vWF)
2. Activation of platelets (attachment to collagen fibers activates
platelets to swell, become irregular in surface and become sticky)
3. Release of contents (serotonin, thromboxane A2 “TXA2”, ADP ...)
4. Aggregation of platelets (serotonin causes vasospasm whereas
TXA2 & ADP activate nearby platelets to swell and stick to the
previous ones and release the same contents, then TXA 2 & ADP
activate other platelets and the cycle repeats itself in a form of
“positive feedback mechanism” until the small hole in the blood
vessel is sealed by the plug (large holes need coagulation).

The core of medical physiology (1) – 3rd edition Page 197


Table 5.2: Comparison between serum and plasma
Difference Plasma Serum
Formation By centrifugation By incubating a clotted
of blood sample for sometime
Clotting proteins Present Absent (consumed)
(e.g. fibrinogen)

Serotonin Low High (released by


activated platelets)

 Blood coagulation
- Biochemical reactions that involve activation of clotting factors
(present in blood in inactive form) in a cascade way to form a blood
clot.
- The clotting factors are indicated by roman numbers from I to XIII or
by names; for example:
o Fibrinogen Factor I
o Prothrombin Factor II
o Tissue factor Factor III or tissue thromboplastin
o Calcium Factor IV
o Factor V Proaccelerin or labile factor
o Factor VII Serum prothrombin conversion accelerator
(SPCA) or stable factor
o Factor VIII Anti-hemophilic factor A
o Factor IX Christmas factor or antihemophilic factor B
o Factor X Stuart factor
o Factor XI Antihemophilic factor C

The core of medical physiology (1) – 3rd edition Page 198


o Factor XII Hageman factor
o Factor XIII Fibrin-stabilizing factor
- Most of these factors are produced by the liver (exceptions include
factors: III, IV and XIII).
- Remember that: there is no clotting factor known as factor VI.
- Activation of the clotting factors occurs through two pathways:
o Intrinsic pathway
o Extrinsic pathway
 The intrinsic pathway
- All factors involved are present in the blood (i.e. there is no need for
an extrinsic factor from the tissues).
- Factors activated through this pathway include: XII, XI, IX, VIII, X,
V, II and I; plus calcium ions (factor IV) & phospholipids (provided by
platelets as platelet factor 3).
- The first factor to be activated is factor XII; it is activated when
blood comes in contact with collagen or wettable negative surface
like glass. See the other steps in figure 5.14:
 The extrinsic pathway
- It is called extrinsic because a tissue factor (known as
thromboplastin or factor III) is needed for its activation. This factor
consists of phospholipids and protein.
- Factors activated through this pathway include: VII, X, V, II and I;
plus calcium ions (factor IV) & phospholipids (provided by the tissue
factor).
- This pathway is faster than the intrinsic pathway.
- The first factor to be activated is factor VII; it is activated by the
tissue factor. See the other steps in figure 5.14.

The core of medical physiology (1) – 3rd edition Page 199


- Factor X, II, I and V are activated through both intrinsic and extrinsic
pathways; that’s why they are described as factors of the common
pathway.
Fig 5.14: The pathways of coagulation

The core of medical physiology (1) – 3rd edition Page 200


Important notes about coagulation
 Calcium ions are needed in all steps of coagulation except the
first two steps of the intrinsic pathway which need high molecular
weight kininogen (HMWK) and Prekallikrein (PK).
 Phospholipids are needed in many steps (e.g. activation of factor
X and II).
 Phospholipids in the intrinsic pathway are provided by platelets
and in the extrinsic pathway are provided by the tissue factor.
 Factor V and VIII are cofactors, their activation by thrombin
results in amplification and repetition of the reactions through a
positive feedback mechanism.
 Thrombin, the active form of factor II, is an alpha globulin
synthesized in the liver as prothrombin. Its activation requires
many factors: aX, aV, calcium and phosplolipids. All these factors
are known collectively as prothrombin activator complex or
thrombokinase.
 Activation of prothrombin to thrombin is the most important step in
coagulation. Actions of thrombin include:
o Activation of fibrinogen (factor I)
o Activation of factor V
o Activation of factor VIII
o Activation of plasminogen (see below)
o Activation of platelets
o Activation of protein C (through thrombomodulin, see below)
 Thrombin activates fibrinogen to form fibrin strands. These
combine together to form a net at the site of injury. The net
catches cells and protein until it is completely sealed as clot.

The core of medical physiology (1) – 3rd edition Page 201


 The blood clot is stabilized by factor XIII (fibrin stabilizing factor)
and Ca++. Factor XIII is released by platelets. It forms covalent
bonds between fibrin strands.
Fig 5.15: The blood clot

 Fibrinolysis
- After formation of the blood clot, bleeding stops; but the clot causes
partial obstruction within the blood vessel. Therefore, it should be
removed by the process of fibrinolysis; however, some clots may
become invaded by fibroblasts (invited by platelet derived growth
factors). These cells convert the clot into a permanent fibrous tissue.
- Fibrinolysis involves enzymatic degradation of fibrin by the enzyme
plasminn resulting in formation of fibrin degradation products.
- Plasmin is produced by the liver as an inactive protein
(Plasminogen).
- Plasminogen is activated by factors called (Plasminogen activators)
- These factors can be obtained from:
o Blood
o Tissues
o Urine
o Bacteria

The core of medical physiology (1) – 3rd edition Page 202


 Plasminogen activators in blood
- These are: aXII and thrombin.
- Although they are responsible for formation of the blood clot, they
participate in its lysis by activating plasminogen into plasmin.
However, lysis does not occur immediately following blood clot
formation because plasmin is inhibited by a plasma protein known as
α2 anti-plasmin. The level of plasmin has to increase gradually until it
becomes sufficient to cause lysis of the clot, this usually occurs after
48 hours. During this time regeneration occurs and the hole in the
blood vessel is closed.
 Plasminogen activator in the tissues
- This is the tPA (tissue Plasminogen activator). It is released by the
tissues, e.g. vascular endothelium, to cause lysis of the clot.
- It can be synthesized by recombinant DNA from melanoma cells for
medical purposes (to be used for treatment of acute myocardial
infarction). However, it is very expensive to be used in the developing
countries.
 Plasminogen activator in urine
- This is urokinase which is synthesized by the transitional epithelium
of the urinary tract and released in urine.
- Its physiological significance in urine is unknown, may be to cause
immediate lysis for clots that may obstruct the renal tubules.
 Plasminogen activator from bacteria
- This is streptokinase, a toxin produced and released by streptococci
to break clots formed by the body to limit its spread.
- The bacteria are cultured and streptokinase is extracted for
treatment of acute myocardial infarction. It is rather cheaper than

The core of medical physiology (1) – 3rd edition Page 203


tPA. However, since it is extracted from bacteria, its IV infusion
induces formation of antibodies against it. Therefore, its future use in
the same patient requires certain precautions.
 The normal endothelium can activate plasminogen
- The endothelial cells have Plasminogen receptors on their surface.
These activate Plasminogen when it binds them.

ANTICOAGULANTS
 Natural anticoagulants
- These are present naturally within the body. They prevent clotting
within the circulation.
- Abnormalities associated with a defect or absence of one or more
of these factors are usually associated with intravascular clotting.
- They include:
 The normal endothelium
- Prevents clotting by the following:
o Smooth surface prevents activation of the clotting factors
o Release of prostacycline & nitric oxide (these cause
vasodilatation and inhibit platelet aggregation)
o Have plasminogen receptors on their surface that activates
plasmin when it binds them
o Thrombomodulin (see below)
 The blood flow
- Rapid blood flow inhibits coagulation whereas stasis of blood favors
coagulation. Stasis may occur due to:
o Mechanical obstruction of a blood vessel
o Prolonged sitting (e.g. during prolonged flight or bus journey)

The core of medical physiology (1) – 3rd edition Page 204


o Prolonged lying down (e.g. comatose patients or those with
fractures)
 Anti-thrombin III
- An alpha globulin synthesized by the liver.
- Inactivates IX, X, XI and XII.
- Its activity is increased about 1000 times by heparin. For this
reason it is called the heparin cofactor
- Hemoglobin may also facilitate the activity of antithrombin III.
 Heparin
- Sulfated polysaccharide (MWt: 15,000-18,000)
- Found in the granules of mast cells and basophils
- Acts through activation of antithrombin III
- Its low level in plasma indicates that it is not very important as a
natural anticoagulant at normal conditions.
- It is very important for prevention of coagulation in containers used
for blood collection and when given intravenously in therapeutic
doses (e.g. in cases of deep vein thrombosis (DVT)).
- When given accidentally in toxic doses, protamine sulphate is used
to neutralize it (= antidote).
 Alpha II macroglobulin
- Similar to anti-thrombin III
- Inactivates some clotting factors.
 Thrombomodulin
- A protein expressed on surface of most endothelial cells (except
those in cerebral capillaries)
- Binds thrombin to form a complex that activates protein C.

The core of medical physiology (1) – 3rd edition Page 205


 Protein C and its cofactor: protein S
- Produced by the liver
- Require vitamin K for their synthesis.
- Inactivate the following:
o Factor V
o Factor VIII
o Inhibitor of tissue Plasminogen activator (i.e. activates tPA
and therefore increases plasmin)
 Fibrin, plasmin and FDPs
- Fibrin adsorbs some clotting factors and this prevents propagation
of the clot within the blood vessel.
- Plasmin breaks down fibrin resulting in dissolution of clots.
- Fibrin degradation products (FDPs) also inhibit coagulation (by
inhibiting thrombin).
 Synthetic anticoagulants
 Vitamin K antagonists
- For example warfarin (Coumarin derivative)
- This interferes with the action of vitamin K that’s needed for
synthesis of the following clotting factors in the liver (1972):
o Factor II (prothrombin) (2)
o Factor VII (7)
o Factor IX (9)
o Factor X (10)
- Warfarin is the only anticoagulant that can be taken orally. It is used
for prevention of thrombosis and embolism.
- Warfarin interacts with many drugs that may potentiate or inhibit its
effects (e.g. aspirin); that’s why drugs taken with it should be revised.

The core of medical physiology (1) – 3rd edition Page 206


- Warfarin therapy should be monitored by testing the INR
(International Normalized Ratio). This is calculated from prothrombin
time (PT) as follows: INR = (PT test/PT control)ISO Where ISO is the
international sensitivity index.
Very important notes to remember:
 Effects of warfarin start to appear 3 days after initiating therapy.
That’s because warfarin acts through inhibiting synthesis of
vitamin K dependent clotting factors in the liver, but factors
already present in blood continue to act during the first few days.
 In addition to that, coagulation tendency is sometimes increased
when starting warfarin therapy, rather than decreased! That’s
because warfarin inhibits activity of the anticoagulants protein C &
S which are also vitamin K dependent factors.
 Therefore, the best way to anti-coagulate the patient during this
period is to give heparin together with warfarin when starting
treatment and to monitor warfarin effects with INR; then stop
heparin and continue with warfarin.
 Calcium chelating agents
- These include: oxalate, citrate and EDTA.
- They bind calcium and make it unavailable for coagulation
- Not used in vivo because they may decrease calcium concentration
in the plasma and cause tetany.
Remember that:
 Anticoagulants used only in vitro: Calcium chelating agents
 Anticoagulants used only in vivo: Vitamin k antagonists
 Anticoagulants used in vitro and in vivo: Heparin
-
The core of medical physiology (1) – 3rd edition Page 207
TESTS FOR HEMOSTATSIS
 Bleeding time (BT)
- A test for platelets because it is done by causing small hole in a
blood vessel (a pinprick at the lobe of the ear or the anterior surface
of the forearm, using a lancet)
- Normally takes 1-7 minutes.
- It is prolonged in the following conditions:
o Thrombocytopenia
o Thromboasthenia
o Von Willebrand disease
o Vitamin C deficiency
 Clotting time
- A test for the clotting factors of the intrinsic pathway or common
pathway (because it involves collection of blood in a test tube and
then initiation of coagulation by contact of blood with glass).
- Normally takes less than 10 minutes.
- It is prolonged in the following conditions:
o Hemophilia
o von Willebrand disease
o Vitamin K deficiency
o Chronic liver disease
o Afibrinogenaemia
o Obstructive jaundice
 Prothrombin time (PT)
- A test for the clotting factors of the extrinsic pathway because blood
is collected in a tube containing citrate to remove Ca++; then Ca++ &
thromboplastin are added to allow blood to clot (= extrinsic pathway).

The core of medical physiology (1) – 3rd edition Page 208


- Normally takes 12-15 seconds.
- It is prolonged in the following conditions:
o Vitamin K deficiency
o Chronic liver disease
o Afibrinogenaemia
o Obstructive jaundice
- As mentioned above, it is used for calculation of the international
normalized ratio (INR).
 Activated partial thromboplastin time (aPTT)
- A test for the clotting factors of the intrinsic pathway.
- It is activated to shorten the time of coagulation by adding certain
substances like kaolin and it is called partial because thromboplastin
is not added.
- It normally takes 25-39 seconds.
- It is prolonged by the same causes that prolong the clotting time.

EXAMPLES OF BLEEDING TENDENCIES


 Hemophilia
- Occurs due to deficiency of:
o Factor VIII (hemophilia A, classical, 85% of cases).
o Factor IX (hemophilia B, less common, 15% of cases, also
known as Christmas disease).
- Inherited as X- linked recessive (therefore it is more common in
males).
- Characterized by normal BT and prolonged clotting time and aPTT.
- Treatment: factor VIII injections or fresh frozen plasma or blood.

The core of medical physiology (1) – 3rd edition Page 209


 Von Willebrand disease
- vWF is a glycoprotein produced by endothelial cells and
megakaryocytes.
- It is needed for stabilization of factor VIII and for attachment of
platelets to collagen.
- Congenital deficiency of this factor causes bleeding tendency with
prolonged bleeding time and clotting time.
 Afibrinogenaemia
- Deficiency of fibrinogen may be congenital (since birth), or acquired
later in life by a disease (e.g. placental abruption during pregnancy
results in clotting of blood behind the placenta. This consumes
fibrinogen and causes its deficiency).
- Characterized by prolonged clotting time and prothrombin time;
however, the best test is thrombin time (TT) which gives information
about availability of fibrinogen.
 Chronic liver diseases
- Impairs synthesis of the clotting factors which are synthesized in the
liver.
 Obstructive jaundice
- Impairs absorption of vitamin K. which is a fat soluble vitamin that
needs bile for its absorption from the intestine.
- Vitamin k is needed for synthesis of clotting factors the II, VII, IX
and X (1972). Therefore obstructive jaundice causes deficiency of
these factors. For this reason, patients with obstructive jaundice
should receive vitamin K injections daily for 5 days before surgery.

The core of medical physiology (1) – 3rd edition Page 210


Examples of clotting tendencies (thrombosis):
 Due to congenital problems
- Deficiency of protein C, protein S or antithrombin III
 Due to atherosclerotic blood vessels
- Atherosclerosis may be complicated by complete occlusion of a
blood vessel.
- It is predisposed by systemic diseases like diabetes and
hypertension.
- Most dangerous at the coronary, renal and cerebral arteries
 Due to sluggish blood flow
- Follows prolonged periods of bed ridden (e.g. due to operations,
delivery or fractures).
- May be complicated by detachment of the clots (embolization).
- A large embolus detached from a lower limb vein that obstructs the
pulmonary artery usually causes immediate death.
- This is known as pulmonary embolism.
 Due to wide spread deposition of fibrin in the circulation
- This is known as disseminated intravascular coagulations (DIC).
- It is caused by septicemia, disseminated cancers or massive tissue
injuries. These stimulate formation of multiple clots within the
intravascular space.
- The clots consume the clotting factors and therefore patients bleed
from the skin, GIT and respiratory tract … (usually fatal).

The core of medical physiology (1) – 3rd edition Page 211


PLASMA
- The fluid part of the blood
 Volume
- About 5% of total body weight in a 70 kg adult male (= 3.5 L)
 Contents
 Water (= 90%)
 Inorganic substances (minerals, electrolytes …)
 Organic substances (proteins, glucose, fats, bilirubin, urea …)
 Plasma proteins
- Types:
 Albumin
 Globulins (,  and )
 Fibrinogen
- Synthesis
 The liver (synthesizes most plasma proteins)
 Plasma cells (Synthesizes gamma globulins which are not
synthesized by the liver (also known as immunoglobulins or
antibodies)
- Concentration:
 Total = 6-8 g/dL
 Albumin: 3.5-5 g/dL
 Globulins (,  and ): 2.3 g/dL
 Fibrinogen: 0.3 g/dL
- Functions:
 Oncotic pressure (albumin because its particles are smaller
and larger in number than other types of plasma proteins)

The core of medical physiology (1) – 3rd edition Page 212


 Viscosity of blood (globulins and fibrinogens because of the
asymmetrical shape and large size of their particles)
 Clotting (fibrinogen and some globulins- most clotting factors
are beta globulins but prothrombin is an alpha globulin)
 Transport of hormone like thyroid, adrenocortical and gonadal
hormones (albumin and globulins)
 Carriage of ions, metals, bilirubin … (e.g. albumin carries
bilirubin and the beta globulin transferrin carries iron)
 Protection by antibodies (gamma globulins)
 Buffer (= 15% of the buffering capacity of the blood, by the
COOH and NH2 groups in all proteins)

QUESTIONS FOR SELF ASSESSMENT-8 (BEST OF FIVE)


1. The intrinsic pathway of blood coagulation:
a. is triggered by contact of factor X with collagen
b. is faster than the extrinsic pathway
c. utilizes 50% of the clotting factors present in the blood
d. requires tissue thromboplastin for its activation
e. is never completed in the absence of Ca++
2. Platelets:
a. are non nucleated biconcave cells
b. arise from megakaryocytes in the bone marrow
c. always stop bleeding
d. their count is higher than red blood cells
e. their size is larger than white blood cells
3. A child whose both father & mother are group B-positive may be:
a. blood group A positive
b. blood group A negative
c. blood group AB positive
d. blood group O-negative
e. blood group AB negative
4. Vitamin K deficiency causes low production of factor:
a. I
b. VII
c. VIII

The core of medical physiology (1) – 3rd edition Page 213


d. XI
e. XII
5. Prolonged clotting time is found in all the following except:
a- heparin therapy
b- obstructive jaundice
c- congenital afibrinogenaemia
d- thrombocytopenia
e- chronic liver disease
6. The following is not a function of thrombin:
a- breakdown of fibrinogen
b- activation of protein C
c- activation of factor VIII
d- activation of plasminogen
e- activation of factor XI
7. The following is not a primary function of globulins:
a- blood coagulation
b- transport of iron
c- immunity
d- colloid osmotic pressure
e- blood viscosity
8. A woman who is blood group O Rh -ve:
a- has no any antigens on surface of RBCs
b- has anti-A, anti-B and anti D antibodies in plasma
c- acts as a universal donor
d- can never have a rhesus +ve child
e- usually gives birth to a jaundiced baby
9. The following statement about the extrinsic pathway is not true:
a- it is triggered by tissue trauma
b- it involves factor II
c- it requires calcium ions
d- it is prolonged in haemophilia
e- it is normal in von Willebrand disease
10. The following anticoagulant acts “in vitro” and “in vivo”:
a- heparin
b- warfarin
c- thrombomodulin
d- EDTA
e- citrate
11. Bleeding tendency commonly occurs due to all the following
except:
a- warfarin therapy
b- obstructive jaundice
c- hypocalcaemia
d- thromocytopenia

The core of medical physiology (1) – 3rd edition Page 214


e- chronic liver disease
12. The following is not transported by plasma proteins
a. iron
b. oxygen
c. carbon dioxide
d. bilirubin
e. thyroxine
13. This factor is not activated through the intrinsic pathway?
a. I
b. II
c. V
d. VII
e. VIII
14. Plasminogen is activated by all of the following except:
a. Urokinase
b. Streptokinase
c. Tissue plasminogen activator (tPA)
d. thromboplastin
e. thrombin
15. All the following are natural anticoaguans except:
a. thromboxane A2
b. prostacycline
c. protein C
d. antithrombin III
e. plasminogen
16. Hemophilia is:
a. more common in males than females
b. due to congenital deficiency of Hageman factor
c. characterized by normal bleeding time
d. characterized by low level of vWF
e. diagnosed when the INR is higher than normal
17. Anti-thrombin III:
a. is a beta globulin
b. activity is inhibited by heparin
c. is similar in structure to alpha 2 macroglobulin
d. deficiency causes bleeding tendency
e. level in plasma is decreased by warfarin

Question 1. 2. 3. 4. 5. 6. 7. 8. 9. 10.
Answer e b d b d e d c d a
Question 11. 12. 13. 14. 15. 16. 17.
Answer c b d d a c c

The core of medical physiology (1) – 3rd edition Page 215


CHAPTER 6
THE CARDIOVASCULAR SYSTEM
THE CARDIAC MUSCLE
Fig 6.1: The heart

 Human heart consists of:


o 4 chambers

• 2 atria (thin walled, receive blood from major veins and allow
it to fill the ventricles)

•2 ventricles (thick walled, pump blood to pulmonary and


systemic circulation)
o 4 valves

• 2 Atrio-ventricular valves (AV valves):


- Mitral (left) and Tricuspid (right)

• 2 Semilunar valves:
- Aortic (left) and Pulmonary (right)

The core of medical physiology (1) – 3rd edition Page 216


Properties of the cardiac muscle
- The cardiac muscle contains two types of muscle fiber:
• Cardiac muscle proper
• Conductive system
- It has special properties that characterize it from other types of
muscles, these properties include:
1- The histology of the cardiac muscle
- Unlike skeletal muscle, there are anatomical connections between
the myocardial fibers (intercalated discs and gap junctions); and
unlike smooth muscle, the cardiac muscle is striated.
Fig 6.2: The muscle differences in histology

2- Functional syncytium
- Stimulation of one cardiac muscle cell results in stimulation of all
the cells. This occurs due to presence of gap junctions & intercalated
discs between the fibers (which are areas of low electrical resistance
that allow movement of ions and therefore transmission of action
potential easily between cells).
- Therefore, the heart contracts as one unit (as a syncytium which is
multiple cells grouped together as one unit within a common cell
membrane forming a single multi-nucleated cell).
- Remember that the heart is not an anatomical syncytium but it is a
functional syncytium.

The core of medical physiology (1) – 3rd edition Page 217


Fig 6.3: The cardiac muscle histology

3- The main source of energy


- The cardiac muscle consumes fat as the main source of energy.
Under basal conditions, 60% of the caloric needs are provided by
free fatty acids, 35% by carbohydrates and 5% by ketones and
amino acids.
- On the other hand, skeletal muscles utilize carbohydrates as the
main source of energy, especially during exercise.
4- Blood flow (supply)
- The cardiac muscle receives its blood supply mainly during diastole
(unlike skeletal muscles which receive their blood supply mainly
during systole).
- The left ventricle receives its blood supply during the diastolic
phase only whereas the right ventricle receives its blood supply
during both systole & diastole.
- This is explained by many physiological & anatomical mechanisms:
 Physiological explanation: During systole, aortic pressure= 120
mmHg, left ventricular pressure= 120 mmHg and right
ventricular pressure= 25 mmHg; therefore blood flows from the
aorta through the right coronary artery but not the left one. On

The core of medical physiology (1) – 3rd edition Page 218


the other hand, during diastole, aortic pressure= 80 mmHg, left
ventricular pressure= 0 mmHg and right ventricular pressure= 0
mmHg; therefore blood flows from the aorta through both right
and left coronary arteries.
 Anatomical explanation: during systole, leaflets of the aortic
valve obstruct openings (sinuses) of coronary arteries.
5- Oxygen extraction
- At basal conditions, the cardiac muscle extracts higher amount of
oxygen than skeletal muscle. It consumes about 9 mL O2/ 100 g
tissue/minute. This is approximately about 70-80% of the oxygen
delivered by each unit of blood.
6- Metabolism
- The cardiac muscle depends on aerobic metabolism for generation
of energy. Anaerobic metabolism provides less than 1% of the total
energy. This figure may increase slightly during hypoxic states;
however, there is no oxygen debt mechanism (obstruction of the
coronary artery reduces oxygen delivery to the cardiac muscle and
results in its necrosis and death). On the other hand, skeletal muscle
depends on both aerobic & anaerobic metabolism for generation of
energy, and there is oxygen debt mechanism (oxygen debt is the
extra amount of oxygen consumed after exercise to repay oxygen
taken from myoglobin & to get rid of lactate accumulating in muscle.
7- Electrical activity (action potential)
- Action potential of the cardiac muscle proper is characterized by the
plateau phase (due to calcium influx), and that of the conductive
system is characterized by the prepotential (see below). Theses
phases are not found in action potentials of other types of muscles.

The core of medical physiology (1) – 3rd edition Page 219


Fig 6.4: Actions potentials in the cardiac muscle

8- The refractory period


- Is prolonged in cardiac muscle (200-400 ms) due to presence of the
plateau phase. This protects the heart from tetanus (summation of
contractions during successive stimulation) and therefore allows the
cardiac muscle to relax; because relaxation is essential for
ventricular filling. The refractory period in skeletal muscles is about 5-
20 ms.
Fig 6.5: the refractory periods in muscles

9- ECF calcium:
- Contraction of the cardiac muscle depends on both ICF & ECF
calcium, whereas contraction of skeletal muscle depends on ICF
calcium only. The T tubules in the cardiac muscle allow influx of
calcium during action potentials to stimulate contraction. Therefore,

The core of medical physiology (1) – 3rd edition Page 220


increasing ECF calcium increases contractility in cardiac muscle and
may stop the heart in systole.
10- No Recruitment in cardiac muscle
- Increasing strength of stimulation in skeletal muscles results in
recruitment (i.e. it increases number of stimulated fibers) whereas in
the cardiac muscle there is no recruitment (no increase in the
number of stimulated fibers). That’s because stimulation of one fiber
in cardiac muscle results in stimulation of all the fibers.
11- Automaticity
- The various parts of the conductive system can discharge action
potential spontaneously. This is explained by the prepotential phases
that characterize action potentials of the different structures of the
conductive system (the conductive system has no resting membrane
potential but gradual elevation of prepotential towards the threshold,
see below). This results in spontaneous generation of action
potentials by the conductive system (especially the sino-atrial node).
However, the cardiac muscle proper, which has no prepotential, can
discharge action potentials spontaneously, but in abnormal
conditions (e.g. ischemia). This is known as idio-ventricular rhythm.
12- Rhythmicity
- The cardiac muscle undergoes regular rhythmic contractions due to
presence of the conductive system, which controls these rhythmic
contractions whereas skeletal muscle cannot contract rhythmically.
13- Conductivity
- This is again a characteristic of the conductive system, which
generates rhythmic action potentials and conducts it to the whole
parts of the cardiac muscle (see below).

The core of medical physiology (1) – 3rd edition Page 221


The conductive system
- The conductive system is formed of cardiac muscle fibers that are
less striated and lack definitive boundaries.
- It consists of the following structures:
 Sino-atrial node (SA node)
 Atrio-ventricular node (AV node)
 Internodal atrial pathways
 Bundle of His
 2 Bundle branches (BB):
o Rt Bundle branch (RBB)
o Lt Bundle branch (LBB).
- The left BB is divided into anterior & posterior fascicles
o Purkinje fibers
Fig 6.6: the conductive system

The core of medical physiology (1) – 3rd edition Page 222


The sino-atrial node (SA Node)
- The SA node is found in the right atrium, at the opening of the SVC.
- It can discharge action potential spontaneously (the discharge
probably originates from small rounded cells in the SA node called P
cells).
- It is supplied by sympathetic neurons from the right side (because it
is developed from structures in the right side of the embryo). The
sympathetic neurons increase its rate of discharge.
- Similarly, it is supplied by parasympathetic neurons from the right
vagus. The parasympathetic neurons decrease its rate of discharge.
- Without innervation, its rate of discharge = 100/min, with innervation
(symp & parasymp) = 70/min. This indicates that the parasympathetic
effect is more dominant than the sympathetic effect on the SA node.
- The rate of discharge of the SA node is faster than other parts in
the heart, e.g.: AV node [45 /min] and Ventricles [15-35 /min]. That’s
why the SA node determines the heart rate (because it is the fastest).
For this reason it is regarded as the cardiac pacemaker.
- Its action potential is characterized by the prepotential (gradual
elevation of the membrane potential towards the threshold) due to
low K+ efflux through the K+ leak channels (remember that potassium
efflux generates the resting membrane potential). In addition to that,
the prepotential is completed by some Ca++ influx (through transient
Ca++ channels (T channels).
- The depolarization phase is due to Ca++ influx (through long lasting
calcium channels (L channels)); with some sodium influx; whereas
the repolarization phase is due to potassium efflux through the K+
voltage gated channels.

The core of medical physiology (1) – 3rd edition Page 223


- The sympathetic makes the prepotential more vertical and therefore
increases heart rate whereas the parasympathetic makes the
prepotential more horizontal and therefore decreases the heart rate.

Fig 6.7: Effects of neurons on the prepotential

The AV Node
- The AV node is found in the Rt atrium on the right side of inter-atrial
septum, above the fibrous ring that separates atria from ventricles.
- It delays conduction from atria to ventricles (because its muscle
fibers have lower number of gap junctions and smaller diameter than
the SA node).
- This delay allows atrial contraction to precede ventricular
contraction and therefore completes ventricular filling with blood.
- When the SA node is diseased as in fibrosis of the SA node in old
subjects (known as sick sinus syndrome), the AV node becomes the
cardiac pacemaker. Here the heart rate decreases to about 45 beats/
min).
- When the AV node is damaged, ventricular rate becomes 15-35/min

The core of medical physiology (1) – 3rd edition Page 224


THE ELECTROCARDIOGRAM (ECG)
- It is a record of the electrical activity of the heart from the surface of
the body using electrodes.
- The record represents the sum of all action potentials of myocardial
fibers at the time of recording as follows:
o The sum of all atrial depolarizations
o The sum of all atrial repolarizations
o The sum of all ventricular depolarizations
o The sum of all ventricular repolarizations
- All these events appear in the ECG as waves and intervals. The
waves are indicated by the letters p, q, r, s, t and sometimes u.
 Methods of Recording
- The ECG is recorded by two types of recording:
o One active electrode + an inactive electrode (= unipolar
recording)
o Two active electrodes + an inactive electrode (= bipolar
recording)
- The inactive electrode is the reference electrode that gives the iso-
electric line in the ECG; it is usually attached to the right foot.
- The isoelectric line determines whether a wave is positive or
negative. A positive wave is that recorded above the isoelectric line
whereas a negative wave is that recorded below it.
- A current of depolarization or repolarization coming towards an
active electrode produces a positive ECG wave whereas a current
passing a way from an active electrode produces a negative wave.
- The active electrodes are placed on the forearms, legs and chest;
at different locations to detect abnormalities in different sites of the

The core of medical physiology (1) – 3rd edition Page 225


heart. Abnormalities appear as changes in the normal configuration
of the different waves and intervals in the ECG.
o The unipolar leads
- These are readings obtained by unipolar recording (i.e. each
reading is recorded by one active electrode). They include:
Limb leads (recorded by electrodes placed on the limbs):
o aVR (augmented vector of the right arm)
o aVL (augmented vector of the left arm)
o aVF (augmented vector of the left foot, remember that the
reference electrode is placed on the right foot)
Chest leads (recorded by electrodes placed on the chest):
o V1 (Vector of electrode placed on the 4th intercostal space to
the right of the sternum)
o V2 (Vector of electrode placed on the 4th intercostal space to
the left of the sternum)
o V3 (Vector of electrode placed midway between V2 & V4)
o V4 (Vector of electrode placed on the 5th intercostal space at
the mid-clavicular line)
o V5 (Vector of electrode placed on the 5th intercostal space at
the anterior axillary line)
o V6 (Vector of electrode placed on the 5th intercostal space at
the mid-axillary line)
o The bipolar leads
- Readings obtained by bipolar recording (2 active electrodes)
- Include three limb leads (I, II & III):
o Lead “I” is recorded by two active electrodes placed on the
right and left arms.

The core of medical physiology (1) – 3rd edition Page 226


o Lead “II” is recorded by two active electrodes placed on the
right arm & the left foot.
o Lead “III” is recorded by two active electrodes placed on the
left arm & the left foot.
Einthoven’s triangle
- An imaginary equilateral triangle formed by the three bipolar
electrodes (I, II and III). The heads of the triangle are the points of
attachment of the unipolar leads aVR, aVL and aVF; with the
assumption that the heart lies in the center of this triangle.
- It is well known in electricity that: "In an equilateral triangle, with a
source of electricity in the center, the summation of potentials at the
heads of the triangle = zero".
- Taking Einthoven's triangle: (aVR+ aVL + aVF = zero) or (I + III = II)
- Einthoven’s triangle is used to determine the cardiac axis (i.e. the
orientation of the heart relative to the horizontal plane), which lies
normally between the angles -30 & +90 degrees (or up to + 110).
- Left ventricular hypertrophy deviates the axis to the left of -30
whereas right ventricular hypertrophy deviates the heart to the right
of 110 degrees.
Fig 6.8: Einthoven’s triangle

The core of medical physiology (1) – 3rd edition Page 227


 Importance of the ECG
 The ECG is used to:
o Calculate the heart rate
o Assess the size of the heart
o Diagnose diseases of the heart (e.g. ischemic heart disease
(IHD), myocardial infarction, pericardial effusion …),
o Diagnose arrhythmias
o Diagnose electrolyte disturbances (e.g. hyper or hypokalaemia)
Remember that:
 The ECG does not detect the mechanical events in the heart (i.e.
it does not show contraction or relaxation of the cardiac muscle);
it only detects the electrical activity (i.e. depolarization and
repolarization).

 The standard ECG


Fig 6.9

The normal waves


o P wave due to atrial depolarization
o QRS complex due to ventricular depolarization
o T wave due to ventricular repolarization
o U wave due to papillary muscle repolarization; however, in
sometimes, it may be absent from the ECG or may be present
with other abnormal ECG signs indicating hypokalemia.

The core of medical physiology (1) – 3rd edition Page 228


o Atrial repolarization is not shown in the ECG; it is obscured by
the QRS complex.
The Intervals
o PR interval
- From beginning of P wave to the beginning of QRS complex.
- Its duration (0.12-0.2 s) represents time taken for spreading of
depolarization from the SA node to the ventricular septum
(through AV node and bundle of His).
- Short PR interval indicates tachycardia (e.g. due to Wolf
Parkinson White syndrome “WPW”) whereas prolonged PR
interval indicates 1st degree heart block (e.g. due to hypokalemia).
o QRS duration
- From beginning of “Q“ wave to the end of “S” wave.
- Its duration (< 0.1 s) represents time taken for spreading of
depolarization throughout both ventricles.
- Prolonged QRS duration (described as broad complexes)
indicates ventricular ischemia (ischemic heart disease “IHD”).
o QT interval
- From beginning of “q” wave to the end of “T” wave.
- Its duration (about 0.43 s) represents time of depolarization &
then repolarization of both ventricles.
- Prolonged QT interval predisposes to ventricular arrhythmias.
o ST segment
- From the end of S wave to the beginning of T wave.
- Equivalent to the plateau phase in action potential of the
ventricular muscle; that’s why it is affected by changes in Ca++
ions (prolonged in hypocalcaemia short in hypercalcaemia).

The core of medical physiology (1) – 3rd edition Page 229


 Calculation of the heart rate
- The heart rate from the ECG =
Paper speed (in mm/s) / duration of one ECG cycle (in mm)
- The paper speed = 25 mm/s (unless the other value “50 mm/s” is
used) whereas the duration of one ECG cycle is best taken as RR
interval (in mm). This gives heart rate in beats per second.
- To calculate the heart rate in beats “per minute” rather than “per
second”, the “second” should be divided by 60; therefore the formula
becomes:
Heart rate (beat/min) = 25 x 60 /RR interval (in mm)
- Remember dimensions of the ECG paper:
Fig 6.10

ABNORMAL ECG
 Normal sinus rhythm
- The normal cardiac rhythm is a sinus rhythm (i.e. the pacemaker is
the SA node).
- Abnormal rhythms like nodal rhythm (pacemaker is AV node) or
idioventricular rhythm (pacemaker is the ventricle) occur when the
SA node is diseased or when there is complete block of conduction
between atria and ventricles (3rd degree heart block).
- Presence of normal P wave in the ECG indicates normal rhythm.

The core of medical physiology (1) – 3rd edition Page 230


 Sinus arrhythmia
- This is a normal physiological condition in healthy children & young
adults. It depends on intact autonomic nervous system.
- It is defined as change in heart rate with respiration (heart rate
increases with inspiration & decreases with expiration). This variation
in the heart rate is lost in patients with autonomic neuropathy.
Possible explanations for the sinus arrhythmia:
1- Inspiration = decreased intra-thoracic pressure = increased
venous return = Bainbridge reflex or effect (see below) = increased
heart rate.
2- Inspiration = discharge of inspiratory impulses from the respiratory
center in the medulla = radiation of these impulses to the nearby
cardiac center = increased sympathetic discharge from this center to
the heart = increased heart rate.
3- Inspiration = inflation of the lung = stimulation of stretch receptors
in smooth muscle of bronchioles = inhibitory impulses pass through
the vagus to the cardio-inhibitory center in the medulla = decreased
parasympathetic supply to the heart = increased heart rate.
4- Inspiration = increased venous return to the right side of the heart
but decreased venous return to the left side = decreased cardiac
output from the left side = decreased blood pressure = less
stimulation of baroreceptors = increased heart rate (read below).
Fig 6.13: variation in heart rate with respiration (sinus arrhthmia)

The core of medical physiology (1) – 3rd edition Page 231


 Abnormal sinus rhythm
Sinus tachycardia
o The heart rate is fast (> 100/min) while the pacemaker is still
the SA node.
o Causes may be physiological (e.g. motions, pain, hypoglycemia
and exercise) or pathological (e.g. hyperthyroidism and drugs
like beta adrenergic agonists and muscarinic blockers).
Fig 6.11

Sinus bradycardia
o The heart rate is slow (< 60 /min) while the pacemaker is still
the SA node.
o Causes may be physiological (e.g. sleep and athletes) or
pathological (e.g. hypothyroidism, hypothermia and drugs like
beta blockers and muscarinic stimulators).
Fig 6.12

The core of medical physiology (1) – 3rd edition Page 232


Arrhythmias
- Abnormal rhythm of the heart that may be determined by a new
pacemaker.
Causes include:
o Damage to the normal pacemaker
o Appearance of a new, abnormal pacemaker, with a faster
discharge (this occurs in atria or ventricles due to ischemia)
o Block in the transmission through the conductive system
o Abnormal pathways for transmission (e.g. the bundle of Kent
in Wolf Parkinson White Syndrome).

 Types of arrhythmias
 Atrial arrhythmias:
 Atrial extrasystole
- Premature discharge from an atrial focus (an ECG cycle that
appears before completion of a previous one).
- Characterized by abnormally shaped P wave (because atrial
depolarization is initiated in an abnormal site, not the SA node).
- The premature beat occurs once and it is followed by a delay in the
new discharge from the SA node (known as compensatory pause).
Fig 6.14: Atrial extrasystole showing the abnormal P wave

The core of medical physiology (1) – 3rd edition Page 233


 Atrail tachycardia
- Regular rapid discharge with narrow QRS complexes (the narrow
complexes generally indicate that there is no problem in ventricles).
- The heart rate from the ECG= the radial pulse and may reach about
200 beat/min.
Fig 6.15: Atrial tachycardia

 Atrial flutter
- Regular rapid discharge with narrow complexes.
- The rate of atrial discharge (rate of P waves in the ECG) is about
(200-350/min) whereas the radial pulse (= rate of QRS complexes) is
less than that.
- This is because of the delay in the AV node (every discharge from
the SA node is not necessarily conducted to the ventricles; therefore
number of P waves is higher than number of QRS complexes).
- P waves give a characteristic appearance in atrial flutter (the saw-
toothed appearance).
Fig 6.16: Atrial flutter showing the saw-toothed appearance

The core of medical physiology (1) – 3rd edition Page 234


 Atrial fibrillation
- Irregular rapid discharge with narrow complexes.
- The rate of discharge in atria is expected to be (300-500/min);
however, its calculation from the ECG is impossible because P
waves are completely absent.
- The ventricular rate (i.e. rate of QRS complexes) is slower because
of the delay in the AV node.
Fig 6.17: Atrial fibrillation

Consequences of atrial arrhythmias:


- Occasional extrasystoles have no ill effects and do not need
treatment; however, other atrial arrhythmias decrease time of the
cardiac cycle (because of the tachycardia); this decreases the
diastolic time which is necessary for ventricular filling and therefore
decreases the stroke volume and the cardiac output. The reduction in
cardiac output may cause heart failure.
Treatment of atrial arrhythmias:
- According to severity of arrhythmia and patient condition, atrial
arrhythmias may be treated with simple maneuvers that increase
vagal discharge (like carotid sinus massage at the angle of the jaw,
eyeball pressure or induction of vomiting); or by anti-arrhythmic drugs
(like digitalis, beta blockers …); or by using the defibrillator.

The core of medical physiology (1) – 3rd edition Page 235


 Ventricular arrhythmias
 Ventricular extrasystole
- Premature discharge from a ventricular focus (an ECG cycle that
appears before completion of a previous one).
- Characterized by abnormally shaped QRS complex (because
ventricular depolarization is initiated in an abnormal site, not the
normal site which is the septum).
Fig 6.18

 Ventricular tachycardia
- Regular rapid discharge with broad QRS complexes (the broad
complexes indicate that there is a problem in ventricles).
- Other ECG waves may be hidden by the broad complexes.
- The heart rate is very fast.
- The commonest cause is ischemic heart disease.
Fig 6.19

The core of medical physiology (1) – 3rd edition Page 236


 Ventricular fibrillation
- Very rapid discharge of multiple ventricular foci.
- Results in appearance of an abnormal bizarre shaped ECG).
- ECG waves are not clear.
- It is a cause of death in patients with myocardial infarction.
Fig 6.20

Consequences of ventricular arrhythmias:


- Occasional extrasystoles have no serous effects and do not need
treatment.
- Ventricular tachycardia decreases cardiac output because the time
available for ventricular filling is decreased and therefore the stroke
volume and cardiac output are decreased. This causes heart failure.
- In addition to that, ventricular tachycardia may be complicated by
ventricular fibrillation.
- Ventricular fibrillation is the most dangerous type of arrhythmia. It
causes death in minutes because the heart fibrillates without ejecting
blood to supply the brain.
Treatment:
- According to severity of arrhythmia & patient condition, ventricular
arrhythmias can be treated with anti-arrhythmic drugs or defibrillators
(= electric shock).

The core of medical physiology (1) – 3rd edition Page 237


Heart Block
- Block of conduction from atria to ventricles.
- May be: 1- Incomplete heart block: first or second degree
2- Complete heart block: third degree
 First degree:
- Prolonged PR interval (> 0.2s)
- Causes include: IHD & hypokalaemia.
Fig 6.21

 Second degree
- Conduction from atria to the ventricles is blocked (e.g. due to IHD).
- This results in generation of P wave two or three times before
appearance of the QRS complex (2:1 or 3:1 conduction).
Fig 6.22

 Third degree
- Complete block of conduction from atria to ventricles (due to IHD).
- Atria continue to contract by the SA node whereas ventricles
continue to contract by a ventricular focus (idioventricular rhythm).

The core of medical physiology (1) – 3rd edition Page 238


- P waves appear at regular intervals with a normal rate (60-100/min)
- QRS complexes are not related to the P waves. They appear
independently with an abnormal shape and a slow rate (15-35/min).
Fig 6.23

- Patients with complete heart block develop Stokes Adams attacks


(become immediately unconscious and collapse with a minimal
degree of exercise). That’s because the low cerebral blood flow is
further decreased with exercise due to shifting of blood towards
muscles.

Myocardial infarction (MI)


- Death of myocardial fibers due to interruption of the blood supply
(block in the coronary artery or some of its branches).
- This produces characteristic changes in the ECG. The changes are
explained by the findings that the infarcted area develops rapid
repolarization (due to accelerated opening of K+ channels) followed
by reduction in the resting membrane potential (RMP) and then
delayed depolarization. The reduction in the RMP makes the normal
area more negative relative to the infarcted area (in the extracellular
surface of cells not the intracellular). Therefore the electrical currents

The core of medical physiology (1) – 3rd edition Page 239


flow from inside the infarcted area to the normal area (from positive
to negative). This means that the currents are flowing towards the
ECG electrodes overlying the infarcted area. The electrodes develop
upward deflection. This appears as ST elevation in the ECG (Note:
the currents are flowing away from the electrodes opposite to the
infarcted area. This appears as ST depression in other ECG leads).
Fig 6.24

- Some days later, the dead muscle becomes electrically silent (after
development of scar tissue). Here the infarcted area becomes more
negative relative to the normal area (extracellularly). This results in
disappearance of the ST elevation and appearance of abnormal Q
waves).
Fig 6.25

The core of medical physiology (1) – 3rd edition Page 240


- The ECG does not only diagnose MI but also localizes its site
(according to which lead develops ECG changes). For example:
 ST elevation in II, III and aVF indicates inferior MI
 ST elevation in V1, V2, V3 and V4 indicates anteroseptal MI
 ST elevation in V4, V5, V6, I and aVL indicates anterolateral MI
 Tall R wave and ST depression in V1 and V2 indicates
posterior MI
Remember that:
 Acute MI is a life threatening problem. It may cause immediate
death due to:
- Arrhythmia (e.g. ventricular fibrillation)
- Severe heart failure
- Rupture of the infarcted area.
 The risk of these complications is highest during the first week
following the infarction. That’s why these patients should be
treated appropriately (e.g. by thrombolytic therapy) in special
wards (Coronary Care Units “CCU”) for close follow up and
monitoring during the acute phase.

The core of medical physiology (1) – 3rd edition Page 241


ECG changes during electrolyte disturbances
 Hyperkalemia
o Tall, slender peaked T wave
o Absent P wave
o Wide, slurred QRS
Fig 6.26

 Hypokalemia
o ST segment depression
o Appearance of U wave
o Prolonged PR interval
o T wave inversion.
Fig 6.27

The core of medical physiology (1) – 3rd edition Page 242


 Hypercalcemia:
o Short QT interval
 Hypocalcemia:
o Long QT interval
Fig 6.28

Important notes
 Hyponatremia causes low voltage ECG.
 Hyperkalemia decreases the resting membrane potential (RMP)
towards the threshold, making the cells more excitable (causing
arrhythmias) and eventually the RMP reaches the threshold,
making the cells non excitable. Here the heart stops in diastole.
 Hypercalcemia increases contractility in the heart because ECF
calcium enters the cells to cause contraction.
 Severe hypercalcemia stops the heart in systole. For this reason
IV injection of calcium in patients with hypocalcemia should be
slow to avoid injection of large amount of calcium.

The core of medical physiology (1) – 3rd edition Page 243


THE CARDIAC CYCLE
Duration

• The normal heart rate is about 75 beat/min (i.e. 75 beats in 60


seconds); therefore, each beat takes 60/75 = 0.8 second.

• Each beat is regarded as one cardiac cycle that involves phases of


contraction & relaxation for both atria and ventricles.

• Therefore, each cardiac cycle (0.8s) can be studied in 3 phases:


o Atrial systole (takes 0.1 s)
o Ventricular systole (takes 0.3 s)
o Atrial diastole (takes 0.7 s) & ventricular diastole (takes 0.5 s)
Atrial Systole

• During the phase of atrial systole, the ventricles are in state of


diastole. Therefore, atrial contraction ejects blood to fill the
ventricles through the atrio-ventricular (AV) valves.

• Ventricular filling occurs by two ways: Passive filling (blood flows


passively from atria to ventricles) & active filling (BY atrial systole).

• About 70% of ventricular filling occurs passively. Then atrial


systole completes ventricular filling (provides the 30%).

• Atrial contraction increases atrial pressure and causes reflux of


blood through the superior vena cava, causing a characteristic “a”
wave (“a” for atrial systole) in both atrial pressure curve and
internal jugular venous pressure “JVP” curve (see figure 6.29).
Ventricular Systole

• Occurs immediately after atrial systole because of the delay in the


AV node (i.e. ventricles contract while atria relax).

The core of medical physiology (1) – 3rd edition Page 244


• When the ventricles start to contract, the pressures inside them
start to rise, then contraction continues until ventricular pressure
exceeds atrial pressure, here the AV valves close to prevent reflux
of blood back to the atria.

• Closure of the AV valves results in the first heart sound “S1” (the
sound is caused by turbulence of blood due to closure of valves).

• Then the ventricles continue to contract & their pressures continue


to rise, while all valves are closed. This is the isovolumetric
contraction phase. It occurs immediately after S1.

• When pressure in the left ventricle reaches 80 mmHg and in the


right ventricle reaches 10 mmHg, the semilunar valves open &
blood starts to be ejected into the aorta and pulmonary artery
(remember that inspite of the difference in pressure, the two valves
open almost in the same time).

• The ventricles continue to contract & the pressures continue to rise


to maintain ejection which starts rapidly at first (rapid ejection
phase) and then slows down (slow ejection phase).

• The maximum pressure reached in the left ventricle is 120 mmHg


& in the right ventricle = 25 mmHg (= systolic pressure).

• About 70 ml of blood is ejected from each ventricle. It is known as


stroke volume “SV” and it is defined as volume of blood ejected
from each ventricle per beat.

• About 50 ml of blood remains in each ventricle at the end of


systole. It is known as end systolic volume “ESV”.

• About 120 ml of blood is found in each ventricle before ejection.

The core of medical physiology (1) – 3rd edition Page 245


• This 120 ml is known as the end diastolic volume “EDV”. It is
defined as volume of blood that remains in each ventricle at the
end of diastole.

• Remember that: contraction of ventricles pushes cusps of the AV


valves backwards towards the atria. This causes a characteristic
wave in atrial pressure curve & JVP curve known as “C” wave (“C”
for cusp shooting). Look at the following figure.
Fig 6.29: The cardiac cycle

Atrial & Ventricular Diastole


Atrial diastole

• Precedes ventricular diastole and continues throughout most of


the cycle (because it takes 0.7 s and the whole cycle is 0.8 s).

• During this phase blood enters the atria (venous return).

The core of medical physiology (1) – 3rd edition Page 246


• Accumulation of blood in atria raises atrial & jugular venous
pressures and produces “V” wave in atrial pressure curve and JVP
curve (“V” for venous return).
Ventricular diastole:

• Ventricular diastole starts during atrial diastole and filling of atria


with blood.

• When the ventricles start to relax, the pressures inside them start
to drop. When the ventricular pressures become lower than the
pressures in aortic & pulmonary arteries, the semilunar valves are
closed to prevent return of blood back to the ventricles.

• Closure of the semilunar valves results in the second heart sound


“S2” (the sound is caused by turbulence of blood due to closure of
the valves).

• Then the ventricles continue to relax & the pressures inside them
continue to drop, while all valves are closed. This is the
isovolumetric relaxation phase. It occurs immediately after S2.

• When the pressures inside the ventricles become lower than the
pressures in atria, the AV valves open. This allows passive filling
of ventricles with blood that already filled atria during this period.

• Then the atria contract to complete ventricular filling and to start a


new cycle.

The core of medical physiology (1) – 3rd edition Page 247


THE HEART SOUNDS
- Heart sounds are 4 sounds that result from turbulence of blood flow
within the heart during closure of the valves or rapid filling of the
ventricles.
- The first two sounds are normal sounds, the third is normal in
children and young adults and abnormal in older ages (indicates
heart failure). The fourth sound is always abnormal (hypertension).
The first heart sound (S1)
- Caused by closure of the AV valves.
- Occurs at the beginning of ventricular systole (i.e. at start of the
isovolumetric contraction phase).
- It is low pitched and more prolonged than S2.
The second heart sound (S2)
- Caused by closure of the semilunar valves
- Occurs at the start of ventricular diastole (i.e. at the start of the
isovolumetric relaxation phase).
- It is high pitched and takes less time than S1.
- It may split into two components (aortic S2 and pulmonary S2).
- In normal states, closure of aortic valve precedes closure of the
pulmonary valve (i.e. aortic S2 precedes pulmonary S2).
- Wide splitting of S2 occurs during deep inspiration, when closure of
the pulmonary valve is delayed because the stroke volume is
increased by the extra volume of blood delivered to the heart during
deep inspiration (inspiration causes chest expansion, this decreases
intra-thoracic pressure and increases venous return and therefore
stroke volume).
- Splitting of S2 decreases in expiration and pulmonary hypertension.

The core of medical physiology (1) – 3rd edition Page 248


The third heart sound (S3)
- Caused by rapid ventricular filling (turbulence of blood during
passive filling of ventricles).
- It is normal in children and young adults (filling causes turbulence
because the size of ventricles is small).
- It indicates heart failure in adults because blood volume of passive
filling is added to high end systolic volume in patients with heart
failure (the end systolic volume is high in heart failure because of the
weak ventricular contractions).
- On auscultation, presence of S3 plus S1 & S2 gives a characteristic
sound described as “Galloping rhythm”.
- Galloping rhythm is an important sign of heart failure in adults.
The fourth heart sound (S4)
- S4 is an abnormal sound; it is also known as atrial sound.
- It is caused by atrial systole in patients suffering from hypertension.
- In normal states, atrial systole does not cause a sound because
ventricles expand to receive blood ejected by atrial systole; however,
in a hypertensive patient, ventricular wall becomes stiff due to
hypertrophy (because they act against resistance). That’s why atrial
systole results in turbulence of blood and generation of S4.
Abnormal sounds
- Abnormal musical sounds heard over the heart are known as
murmurs. They are generated by turbulence of blood flow within the
heart due to valvular lesions.
- Abnormal sound heard over arteries is known as bruit (e.g. renal
bruit due to renal artery stenosis (narrowing).
- Abnormal sound heard over veins is known as venous hum.

The core of medical physiology (1) – 3rd edition Page 249


- Murmurs are either systolic or diastolic.
- Systolic murmurs are caused by:
 Stenosis (narrowing) of semilunar valves
 Regurgitation (reflux or dilatation) of atrio-ventricular valves
- Diastolic murmurs are caused by:
 Stenosis (narrowing) of atrio-ventricular valves
 Regurgitation (reflux or dilatation) of semilunar valves
- For example mitral (or tricuspid) stenosis causes mid diastolic
murmur and aortic (or pulmonary) regurgitation causes early diastolic
murmur whereas mitral (or tricuspid) regurgitation causes pan-
systolic murmur and aortic (or pulmonary) stenosis causes ejection
systolic murmur.
Pressure volume curves of the ventricles
- This curve summarizes various events in the cardiac cycle. The Y
axis represents pressure whereas X axis represents volume. The
following figure describes the curve of the left ventricle.
Fig 6.30: The pressure volume curve of the left ventricle

The core of medical physiology (1) – 3rd edition Page 250


- In the above curve:
 A represents: passive ventricular filling
 B represents: atrial systole
 C represents: isovolumetric contraction
 D represents: rapid ejection phase
 E represents: isovolumetric relaxation
 Op1 represents: opening of AV valves
 Op2 represents: opening of semilunar valves
 S1-4 represents: sites of heart sounds
 50 ml = End systolic volume
 120 ml = End diastolic volume

THE CARDIAC OUTPUT


Definition
- Volume of blood ejected by each ventricle each minute (not both
ventricles). It is about 5 L/min in an average adult male, at rest (less
in females and more during exercise).
- The cardiac index (CI) is sometimes calculated to compare between
subjects. It equals (COP/surface area) e.g. CI in adults = 5 /1.6 = 3.1
L/m2. CI in children is higher because surface area is smaller.
Measurement
- Many methods can be used for measurement of the cardiac output.
- These include:
 Indirect methods (like indicator dilution technique, Fick
principle and thermodilution method)
 Direct methods (using electromagnetic or ultrasonic flow-
meter devices).

The core of medical physiology (1) – 3rd edition Page 251


Indicator dilution technique or Hamilton dye method
- In this method, Q amount of a dye is injected in an arm vein; then
serial arterial samples are obtained from a peripheral artery to
measure the concentration of the dye as it is ejected from the heart
to the systemic arteries.
- The concentrations are plotted in a graph paper against time.
- The resulting curve shows that the concentrations increase to a
maximum, and then they start to decrease and then increase again
because of recirculation of the dye (look at figure 6.31).
- The descending limb at point (x) is extrapolated to the X axis to
obtain the time of one circulation (t). The area under the curve is
used to calculate the average concentration (C).
- Then the COP can be obtained in (L/min) as follows:
COP = (Q x 60) / (C x t)

Fig 6.31: Hamilton dye dilution method

The core of medical physiology (1) – 3rd edition Page 252


Fick principle
- Fick principle is commonly used for measurement of blood flow to
organs (e.g. renal blood flow, cerebral blood flow, liver blood flow …).
- It states that “the amount of a substance consumed (or added) by
an organ in a given time equals the arterial-venous difference in
concentration times the blood flow to the organ.
- In summary: Q = [A] - [V] x Blood flow to the organ (where Q= the
quantity of a substance consumed by the organ, [A] and [V] are the
concentrations of the substance in the artery supplying and the vein
draining the organ respectively and [A] - [V] is the arterial-venous
difference in concentration.
- From the above principle the blood flow can be calculated as
follows: Blood flow = Q / [A] – [V]
- If the lungs are taken as an organ, their blood flow = the cardiac
output from the right ventricle = the cardiac output (revise definition of
COP). Therefore COP = Q / [A] – [V]
- The substance consumed (actually added) by the lungs is oxygen.
- Oxygen added by the lungs = 250 ml oxygen/min, measured by
spirometer.
- Oxygen concentration in the pulmonary artery (carrying
deoxygenated blood) = 150 ml oxygen/L blood (measured using a
catheter introduced into the pulmonary artery).
- Oxygen concentration in the pulmonary vein (carrying oxygenated
blood) = 200 ml oxygen/L blood (measured from any peripheral
artery because all arteries have the same concentration of oxygen).
- Remember that oxygen concentration in the deoxygenated blood of
the pulmonary artery is not taken from peripheral veins.

The core of medical physiology (1) – 3rd edition Page 253


- By applying the above values to the formula:
COP = Q / [A] – [V]
COP = 250 ml oxygen/min / [50 ml oxygen/L]
(Where 50 is the arterial-venous difference between 150 and 200 ml
oxygen/L); therefore the COP = 5 L/min.
Thermo-dilution technique
- This is similar to the dilution method.
- A cold saline is injected as an indicator in the right atrium and the
change in blood temperature within the pulmonary artery is used for
calculation of the COP.
Direct methods
- Used mainly in experimental animals.
- Here surgery is needed to enter the heart through great arteries or
veins and measure the cardiac output directly using a flow-meter
device (electromagnetic or ultrasonic flow-meter device).

Control of the COP


- The COP = heart rate x stroke volume.
- 5 L/min = 72 beat/min x 70 ml/beat.
- Therefore, the two factors that control the COP are the heart rate &
stroke volume.
Control of the heart rate (HR)
- The heart rate is the number of cardiac beats per minute.
- It ranges between 60 & 100 beats per min; the average is about 72
beat/min at rest; however, normal heart rate differs according to age
(in neonates>infants>children>adults).

The core of medical physiology (1) – 3rd edition Page 254


- In normal adults, a value less than 60 beat/min is described as
bradycardia whereas a value more than 100 beat/min is described as
tachycardia.
- The term positive chronotropic effect is used to describe increase in
heart rate whereas negative chronotropic effect describes the
reverse.
- Remember that: when the heart rate is increased, the time for
ventricular filling is decreased and therefore the stroke volume is
decreased, this occurs if contractility of the heart is not affected;
however, when contractility is also increased, the stroke volume is
increased (i.e. factors that increase the heart rate & contractility:
increase the stroke volume; whereas factors that increase the heart
rate alone: decrease the stroke volume).
 Factors affecting the HR are:
Neural factors
o Sympathetic neurons
- Release noradrenaline to act on beta 1 receptors
- Activated by stress (fear, pain, emotions …)
- Increase heart rate (positive chronotropic effect)
o Chemoreceptors
- Chemical receptors found in aortic bodies (in aortic arch) & carotid
bodies (in carotid bifurcation).
- Stimulated by hypoxia, hypercapnia and acidosis
- Increase heart rate (positive chronotropic effect)
o Bainbridge effect or reflex
- Increase in heart rate (positive chronotropic effect) due to
increase in venous return that initiates a reflex or effect as follows:

The core of medical physiology (1) – 3rd edition Page 255


- The reflex is initiated in receptors in the right atrium and integrated
in the medulla. It results in stimulation of the SA node to increase
its rate of discharge (Bainbridge reflex)
- The effect is a direct mechanical effect of the blood on the SA
node to increase its rate of discharge (Bainbridge effect)
o Parasympathetic neurons
- Release acetylcholine to act on muscarinic receptors.
- Activated by rest and sleep
- Decrease heart rate (negative chronotropic effect)
o Baroreceptor reflexes
- Stretch receptors found in aortic sinus (in aortic arch) & carotid
sinus (in carotid bifurcation).
- Stimulated by elevation in blood pressure.
- Decrease heart rate (negative chronotropic effect).
Hormonal factors:
- Many hormones have positive chronotropic effect.
- Examples include catecholamines & thyroid hormones.
- Thyroid hormones act indirectly by increasing number and affinity
of beta1 receptors in the heart to catecholamines.
Physiological factors
- HR is increased by exercise (which is associated with sympathetic
stimulation due to physical stress or excitement “emotions”).
- It low during sleep (which is associated with parasympathetic
stimulation).
Physical factors
- Changes in body temperature (due to hot or cold environment or
due to fever or hypothermia) affect heart rate.

The core of medical physiology (1) – 3rd edition Page 256


- Elevation in body temperature by one degree centigrade
increases heart rate by about 15 beat/min whereas reduction in
body temperature does the reverse.
Drugs
o Atropine (muscarinic blocker):
- Blocks the parasympathetic; that’s why it increases the heart rate.
- It is used in cases of cardiac asystole.
o Atenolol & propranolol (beta blockers):
- Both drugs decrease HR by blocking beta 1 receptors in SA node.
- Atenolol is selective beta blocker (blocks beta1) whereas
propranolol is non-selective beta blocker (blocks both beta 1&2).
- Salbutamol is a commonly used drug for treatment of asthma. It
is a beta 2 agonist. Therefore, it should not increase the heart rate;
however, the drug has some effect on beta1 receptors, that’s why it
increases the heart rate as a side effect.
o Digitalis (anti arrhythmic drug that slows conduction):
- Decreases the heart rate because it slows the conductive system.
- It increases entry of calcium in cardiac muscle (through a
mechanism that involves inhibition of the Na+-K+ pump). That’s why it
increases contractility of the heart.

Control of the stroke volume (SV)


- Defined as the volume of blood ejected by each ventricle each beat.
- Normal value is about 70 ml (in an average adult male, at rest);
however, it differs according to age, sex and physiological state.
- It is smaller in neonates < infants < older children < adults due to
gradual increase in the size of the heart with normal development.

The core of medical physiology (1) – 3rd edition Page 257


- The stroke volume is larger in males compared to equivalent female
of the same age and body size because the heart is adapted to eject
more blood to more active tissues in males (muscles) compared to
more inactive tissues in females (fats).
- During stress (emotions, pain, hypoglycemia & exercise), the
sympathetic nervous system is activated resulting in increased
contractility of the heart, this increases stroke volume. At rest, the
stroke volume returns to its normal value “70 ml”.
- The term positive inotropic effect is used to describe increase in
contractility of the heart whereas negative inotropic effect describes
the reverse.
- When contractility of the heart is increased, subjects become aware
of the heart beating, this is described as “palpitation”.
- Remember that: when contractility of the heart is increased, the
stroke volume is increased whereas the end systolic volume is
decreased.
Factors affecting SV are:
Neural factors
o Sympathetic neurons
- Increase the stroke volume (positive inotropic effect).
o Chemoreceptors
- Increase the stroke volume (positive inotropic effect).
o Parasympathetic neurons
- Do not supply ventricles of the heart. Therefore they have no
direct effect on contractility of the heart or stroke volume.
o Baroreceptor reflexes
- Decrease the stroke volume (negative inotropic effect).

The core of medical physiology (1) – 3rd edition Page 258


Hormonal factors:
- Many hormones have positive inotropic effect.
- Examples include catecholamines & thyroid hormones.
- Thyroid hormones act indirectly by increasing number and affinity
of beta1 receptors in the ventricles to catecholamines.
Physiological factors
- SV is increased by exercise (positive inotropic effect).
Physical factors
- Changes in body temperature (due to hot or cold environment or
due to fever or hypothermia) have direct effect on contractility.
- High temperature increases the stroke volume (positive inotropic
effect); whereas low temperature does the reverse.
Drugs
o Atropine (muscarinic blocker):
- Blocks the parasympathetic; that’s why it has no direct effect on
contractility and stroke volume.
o Atenolol & propranolol (beta blockers):
- Both drugs decrease SV by blocking beta 1 receptors in
ventricles. They are commonly prescribed for patients with IHD
because they decrease cardiac contractility and therefore decrease
the need for oxygen by cardiac muscle.
o Digitalis (anti arrhythmic drug that slows conduction):
- Increases entry of calcium in cardiac muscle (it inhibits the Na/K
ATPase pump resulting in accumulation of sodium in cells. This
inactivates a sodium-calcium antiport pump which takes calcium to
outside in exchange to sodium. Calcium increases contractility and
therefore SV.

The core of medical physiology (1) – 3rd edition Page 259


- Digitalis also decreases heart rate and gives ventricles extra time
for filling. This increases the end diastolic volume and therefore the
SV is increased (by Frank-Starling law described below).
Calcium ions
- SV is increased by calcium ions (positive inotropic effect)
-Severe hypercalcemia is fatal. It stops the heart in systole.
Preload & after load
Preload
- It is the load on a ventricle before contraction (or the degree of
stretch before contraction).
- It is equivalent to the end diastolic volume (EDV), which depends
on venous return. Therefore, when the preload is increased, the SV
is increased (direct relationship). The increase in SV occurs
according to the Frank-starling Law (see below).
After load
- It is the resistance in arteries against ejection.
- When increased, the stroke volume is decreased (inverse
relationship).
- After load is increased by: aortic stenosis, pulmonary stenosis,
systemic hypertension, pulmonary hypertension or vasoconstriction.
Frank-Starling Law
- Described first by Frank and confirmed experimentally later by
Starling. It states that: “Within certain limits, the energy of contraction
is directly proportional to the initial length of muscle fibers”.
- The length of cardiac muscle fibers depends on the EDV, which
also depends on venous return. Therefore, higher venous return
increases EDV, this increases contractility & so increases the SV.

The core of medical physiology (1) – 3rd edition Page 260


THE VENOUS RETURN
- It is the blood that returns back to the heart through veins.
- It equals the cardiac output (= 5 L/min in a resting adult male.
- It is affected by the following factors:
Pressure gradient
- Pressure in arteries is higher than vein, higher than right atrium.
- Therefore, blood passes down the pressure gradient from arteries
to veins to right atrium in the heart.
Respiratory pump
- Inspiration increases venous return (inspiration increases chest
volume and therefore decreases intra-thoracic pressure; that’s why
venous return is increased). Expiration decreases venous return
especially expiration against closed glottis (valsalva maneuvre) which
is done during defecation, labor or lifting heavy weights.
Muscle pump
- Contraction of the lower limb muscles squeezes blood in the veins
upwards towards the heart (the venous blood does not pass down
during muscle contraction because of the one way valves in veins.
Gravity
- Standing decreases venous return due to gravity whereas lying
down increases venous return.
Blood volume
- Bleeding decreases venous return by decreasing the total blood
volume whereas fluid overload increases venous return.
Venodilation & venoconstriction
- Venodilation decreases venous return because it increases
diameters of these capacitance vessels to store blood.

The core of medical physiology (1) – 3rd edition Page 261


- Venoconstriction increases venous return (by adding some of the
stored blood to the circulation). This is very important in shock.
State of heart valves
- Stenosis or regurgitation of any of the heart valves decreases the
venous return and results in heart failure.
Pericardial pressure
- Pericardial effusion increases pressure within the pericardial sac
and therefore decreases venous return.
- It is caused by inflammation (pericarditis) due to viral or bacterial
infection or due to hypothyroidism or connective tissue diseases.

CONTROL OF ARTERIAL BLOOD PRESSURE


Definitions
Blood pressure
- The pressure of blood on walls of blood vessels.
- Remember that: hydrostatic pressure is the pressure of water (i.e.
plasma) on walls of blood vessels.
Systolic pressure
- The maximum pressure in a vessel during systole of the heart.
- Normal value is less than 120 mmHg in adults. Values of 120
mmHg or more and less than 140 have been regarded as normal;
however, according to recent studies, these values indicate pre-
hypertension state. Systolic blood pressure of 140 mmHg or more is
hypertension.
Diastolic pressure
- The minimum pressure in a vessel during diastole of the heart.

The core of medical physiology (1) – 3rd edition Page 262


- Normal value is less than 80 mmHg in adults. Values of 80 mmHg
or more and less than 90 have been regarded as normal; however,
according to recent studies, these values indicate pre-hypertension
state. Diastolic blood pressure of 90 mmHg or more is hypertension.
Pulse pressure
- The difference between the systolic and the diastolic pressures.
- Normal pulse pressure is less than 60 mmHg.
- When the systolic pressure is increased and the diastolic is
decreased, the pulse pressure becomes “wide”.
- Causes of wide pulse pressure include:
o Hyperthyroidism
o Aortic regurgitation
o High body temperature
o Hard exercise
o Severe anemia
o Pregnancy
o Arterio-venous fistula
Mean arterial pressure
- The mean pressure during the cardiac cycle.
- It is calculated as follows:
The mean arterial pressure= The diastolic + 1/3 pulse pressure
- Its value is more near to the diastolic than the systolic because the
diastolic phase takes longer time than the systolic in the cardiac
cycle).
- Example: If a blood pressure of a subject = 120/80
o The systolic = 120 mmHg
o The diastolic = 80 mmHg

The core of medical physiology (1) – 3rd edition Page 263


o The pulse pressure = 120 – 80 = 40 mmHg
o The mean arterial pressure = 80 + (1/3 x 40) = 93 mmHg

Control of the arterial blood pressure


- Blood pressure = Cardiac output  Peripheral resistance.
- Therefore the blood pressure is controlled by controlling the cardiac
output and the peripheral resistance.
- The cardiac output is controlled by controlling both the heart rate
and the stroke volume (see above). It determines the systolic
pressure (i.e. the increase in COP increases systolic pressure).
- The peripheral resistance is determined by blood viscosity, length of
arteries and radius of arteries as follows:
PR = 8VL/ r4
- Where V is the blood viscosity, L is the length of arteries and r is the
radius of blood vessels.
- The peripheral resistance determines the diastolic blood pressure.
Viscosity of the blood
- Depends on:
o The packed cell volume (PCV)
o Plasma proteins especially globulins and fibrinogen (because
of the large size of their particles)
o Body temperature (this is quite constant in the normal
physiological conditions)
Length of blood vessels
- Constant in adults.
Radius of blood vessels
- The most important factor in determining the peripheral resistance.

The core of medical physiology (1) – 3rd edition Page 264


- Inversely proportional with the peripheral resistance. For example:
vasoconstriction decreases radius of arteries & increases the
peripheral resistance and therefore increases the blood pressure
whereas vasodilatation increases the radius & decreases the
peripheral resistance and therefore decreases the blood pressure.
Remember that:
 Blood pressure in children is less than that in adults.
 The systolic is less because the COP in children is less (child’s
heart is small= stroke volume is smaller= COP is lower; in spite of
the faster heart rate).
 The diastolic is less because the length of arteries is less;
therefore, peripheral resistance is less (in spite of other factors).

The mechanisms that control the blood pressure


 Very rapid mechanism (act in seconds)
• Baroreceptors
• Chemoreceptors
• CNS ischemic response
 Less rapid mechanisms (act in minutes to hours)
• Hormonal vasoconstriction by
o Renin- Angiotensin system
o Antidiuretic hormone (ADH)
o Catecholamines
• Stress relaxation and inverse stress relaxation
• Capillary fluid shift
 Long term mechanisms (act in days)
• Renin-Angiotensin-Aldosterone system

The core of medical physiology (1) – 3rd edition Page 265


• ADH
• Atrial natriuretic peptide (ANP)
• Thirst mechanism
•Other hormones that may affect the blood pressure
 Local regulation of blood pressure

VERY RAPID MECHANISMS


Baroreceptors
- Stretch receptors found in high pressure areas of the circulation
(aortic sinus in aortic arch and carotid sinus in carotid bifurcation).
- They are stimulated by stretch caused by high blood pressure.
- They send impulses through the vagus & glossopharyngeal nerves
to the medulla oblongata.
- The impulses reach the nucleus of tractus solitarius (NTS) and from
there inhibitory impulses pass to the vaso-motor center (VMC) &
cardiac center (CC) in the medulla causing reduction in sympathetic
discharge from these centers to the heart and blood vessels.
- Excitatory impulses pass to the cardio-inhibitory center in the
medulla causing increased parasympathetic discharge from this
center to the heart.
- The decreased sympathetic & increased parasympathetic result in:
o Reduction in heart rate & stroke volume (i.e. lower COP and
therefore lower systolic blood pressure)
o Vasodilatation (i.e. decreased peripheral resistance and therefore
lower diastolic blood pressure)
- Consequently the B.P is reduced back to normal within a second.
The core of medical physiology (1) – 3rd edition Page 266
- The opposite occurs when there is reduction in B.P: hypotension=
less stretch= less stimulation of baroreceptors= less inhibitory
discharge to the medulla= more sympathetic and less
parasympathetic= increased HR & SV= higher COP= Increased
blood pressure back to normal within a second (systolic & diastolic).
Important notes:
 When the blood pressure is normal, the baroreceptors send tonic
discharge through the buffer nerves (9 and 10) causing
continuous reduction in activity of the VMC and the CC.
 Denervation of the baroreceptors (cutting the buffer nerves)
causes elevation in blood pressure.
 Baroreceptors adapt to new pressures (this is known as
resetting). For example when the BP is chronically elevated, the
baroreceptors increase the rate of their tonic discharge to correct
the pressure; however, if the BP remains elevated, the rate of
discharge will decrease gradually over time until it reaches the
earlier rate. The resetting phenomenon is reversible.
- There are other types of baroreceptors found at the venous side
(low pressure areas). These include:
o Type A atrial baroreceptors: discharge during systole
o Type B atrial baroreceptors: discharge during diastole
- Stimulation of these venous barorecptors results in reflex
vasodilatation like the arterial baroreceptors; however, the heart rate
increases.

The core of medical physiology (1) – 3rd edition Page 267


Chemoreceptors
- The peripheral chemoreceptors are found in the carotid body in
carotid bifurcation and aortic body in aortic arch.
- They are stimulated by hypoxia (low O2 in tissues), hypercapnia
(high CO2) and acidosis (high H+). These stimuli are associated with
hypotension because of the low tissue perfusion.
- They send excitatory impulses through the vagus and
glossopharyngeal nerves to the respiratory center to increase
respiration.
- The impulses also stimulate the vasomotor and the cardiac centers
(due to radiation of impulses in the medulla).
- This increases the blood pressure by increasing sympathetic
discharge from these centers to the heart and blood vessels causing
increased heart rate, higher stroke volume and vasoconstriction.
CNS ischemic response
- Reduction of the blood supply to the brain (ischemia), is caused by
hypotension. This results in accumulation of CO2.
- The vasomotor center is very sensitive to CO2. It is stimulated to
release its sympathetic discharge to the blood vessels. This
increases the blood pressure and therefore improves the blood
supply to the brain.
- A high intracranial pressure (e.g. caused by a brain tumor) impairs
cerebral circulation and results in brain ischemia. This induces similar
CNS ischemic response resulting in elevation of the blood pressure
to high levels (hypertension).
- The high blood pressure stimulates the baroreceptors to inhibit the
heart through the cardiac center and the cardioinhibitory center, but

The core of medical physiology (1) – 3rd edition Page 268


they can not inhibit the vasomotor center which is stimulated by the
CO2; this results in bradycardia.
- This sign (hypertension with bradycardia) is an important sign of
raised intracranial pressure (known as Cushing's sign).
LESS RAPID MECHANISMS
Hormonal vasoconstriction
ADH
- Release of this nona-peptide hormone from the posterior pituitary
gland is stimulated, among other stimuli, by hypotension.
- It acts on V1 receptors in the blood vessels causing
vasoconstriction. This elevates the blood pressure.
- It also elevates the B.P. by acing on V2 receptors in the kidney;
however, this action is grouped with the long term mechanisms.
Catecholamines
- Adrenaline and noradrenaline are released in response to
hypotension (which is a form of stress).
- They act on  receptors in blood vessels causing vasoconstriction.
This elevates the blood pressure.
- Remember that: noradrenaline acts on alpha receptors better than
beta whereas adrenaline acts on beta receptors better than alpha.
- That’s why noradrenaline increases the diastolic (by increasing the
peripheral resistance) whereas adrenaline increases the systolic (by
increasing the cardiac output).
Renin-Angiotensin II
- Renin enzyme is released by the Juxtaglomerular cells in the
kidney. It is stimulated by renal ischemia, hyponatremia &
sympathetic stimulation (all these are associated with hypotension).

The core of medical physiology (1) – 3rd edition Page 269


- It acts on angiotensinogen (plasma protein from the liver)
converting it to angiotensin I.
- Angiotensin I is converted to angiotensin II by angiotensin
converting enzyme (ACE) which is poduced by endothelial cells
(especially of pulmonary capillaries).
- Angiotensin II causes vasoconstriction.
- It also activates the sympathetic to release renin, then renin
converts angiotensinogen to angiotensin I and the cycle repeats itself
(positive feedback mechanism).
- Other actions of angiotensin II are long term effects (see below).
Stress relaxation and inverse stress relaxation
- The stress on walls of blood vessels, caused by high blood
pressure, leads to relaxation of smooth muscles in the walls and
reduction in the blood pressure in minutes (stress relaxation).
- Conversely, reduction in the blood pressure leads to less stress on
the walls of blood vessels and therefore constriction of the blood
vessels leading to elevation in the blood pressure (inverse stress
relaxation).
Capillary fluid shift
- Occurs due to hypotension because there is reduction in the
hydrostatic pressure (which is responsible for filtration).
- It becomes lower than the oncotic pressure in capillaries (which is
responsible for absorption).
- This shifts fluid from the interstitium to the intravascular space. The
blood volume is increased and therefore the blood pressure is slightly
increased.

The core of medical physiology (1) – 3rd edition Page 270


LONG TERM MECHANISMS
Renin-Angiotensin Aldosterone system
- As mentioned above, angiotensin II rapidly elevates the blood
pressure by causing vasoconstriction.
- In addition it has other long term effects. These include stimulation
of aldosterone, stimulation of ADH and stimulation of the thirst
center.
- There are many stimuli of aldosterone. However its stimulation by
angiotensin II completes activation of the renin angiotensin
aldosterone system.
- Aldosterone Acts on the distal convoluted tubules and collecting
ducts in kidney causing reabsorption of sodium and secretion of
potassium. Water follows sodium to the intravascular space.
- This increases blood volume & therefore the blood pressure.
ADH
- As mentioned above, in addition to vasoconstriction, ADH has
another long term effect to elevate the blood pressure. It acts on V2
receptors in the collecting ducts of the kidney causing reabsorption of
water to the intravascular space.
- This increases blood volume & therefore the blood pressure.
Thirst
- Controlled by a thirst center in the hypothalamus.
- It is stimulated by hypotension and other stimuli (hypovolemia,
hyperosmolarity & angiotensin II). The subject drinks water in
response to thirst.
- This increases the blood volume & therefore the blood pressure.

The core of medical physiology (1) – 3rd edition Page 271


Atrial natriuretic peptide (ANP)
- Atrial natriuretic peptide is a hormone released by atria in response
to stretch by hypervolemia (hypervolemia is associated with higher
blood pressure).
- ANP acts in the kidney to decrease sodium reabsorption in the
proximal convoluted tubules and antagonizes aldosterone in the
distal convoluted tubules and collecting ducts of the kidney. Sodium
is lost in urine followed by water.
- This decreases the blood volume & therefore the blood pressure.
Other hormones that may affect the blood pressure:
o Kinins: (vasodilators- decreases the B.P.)
o Adrenomedullin & pro-adrenomedullin (vasodilators- decrease it)
o VIP (vasodilator- decreases the B.P.)
o Urotensin II (vasoconstrictor- increases the B.P.)

LOCAL REGULATION
- Aim of Local regulation of blood pressure:
o To maintain adequate perfusion and therefore adequate supply
of O2 and nutrients to the tissues.
o To adjust the perfusion to tissues according to their needs;
which vary from time to time according to variation in activities.
- Local regulation occurs by the following:
1- Autoregulation:
- It is the intrinsic capacity of tissues to regulate their own blood flow.
- Found in many tissues like skeletal muscles, cardiac muscle, brain,
liver,...

The core of medical physiology (1) – 3rd edition Page 272


- Can be explained by:
 The myogenic theory of autoregulation
- High perfusion pressure results in local vasoconstriction (in organs)
to prevent their rupture (according to Law of Laplace, see below) and
to adjust blood flow to organs. This mechanism is not expected to
occur in response to very low or very high perfusion pressures.
 Metabolic theory of autoregulation
- Active tissues produce metabolites that can cause vasodilatation.
When the blood flow decreases, they accumulate and when the
blood flow increases, they are washed away.
- Metabolites that can cause vasodilatation include: CO2, H+ and K+,
ATP, ADP, adenosine, pyruvate, lactate …
- Localized vasoconstriction can be caused by serotonin and cold.
2- Substances secreted by the endothelium:
• Prostacyclin:
- Cause vasodilatation & inhibits platelet aggregation
- Prostacyclin is balanced by throboxane A2 (TXA2) which is
released from platelets (TXA2 causes vasoconstriction and
stimulates platelet aggregation). The balance is shifted towards
prostacyclin by low dose aspirin; that’s why low dose of aspirin is
commonly prescribed to patients at risk of thrombosis (e.g. diabetic
and hypertensive patients).

• Nitric oxide (NO):


- It is the endothelial derived relaxation factor (EDRF).
- It is synthesized from arginine by the enzyme “NO synthase”
- It is an important vasodilator in many organs (e.g. causes erection
in males); however, it has many other functions in other organs.

The core of medical physiology (1) – 3rd edition Page 273


• Endothelins
- Polypeptides (21 aa)
- They are potent vasoconstrictors
- They have many other functions.

IMPORTANT NOTES ABOUT BLOOD VESSELS


Arteries and arterioles
- The walls of large arteries contain high amount of elastic fibers
whereas the walls of the arterioles contain less amount of elastic
fibers, but more smooth muscle.
- The arterioles are the major site of resistance to blood flow
(resistance vessels).
Capillaries
- These are the major site of fluid exchange between plasma and
interstitium (exchange vessels).
- The junctions between endothelial cells in capillaries may be very
tight (e.g. in brain capillaries and to a lesser extent in muscles); or
fenestrated (e.g. in liver sinusoids and to a lesser extent in
glomerular capillaries).
- Blood capillaries have the highest surface area (surface area
exceeds 6300 m2 in adults) and the lowest velocity of blood flow.
- In spite of the thin wall of capillaries, they do not rupture because of
their small diameter. Similarly, arterioles constrict to resist rupture
when the perfusion pressure is high (explained by “Law of Laplace”).

The core of medical physiology (1) – 3rd edition Page 274


- The law states that: the tension (T) in the wall of a cylinder is equal
to the product of the transmural pressure (P) and the radius (r),
divided by the wall thickness (w).
Fig 6.32

- As mentioned above: T = Pr/w


- The transmural pressure (P) = pressure inside the cylinder –
pressure in tissue which is negligible. Therefore (P) = the pressure
inside the cylinder (= the distending pressure)
- The wall thickness of capillaries (w) is negligible
- Therefore the equation becomes: T = Pr
P = T/r (in blood vessels)
or P = 2T/r (in spheres)
- From the above law, the smaller the radius of a blood vessel, the
lower the tension that’s needed to balance the distending pressure;
that’s why vessels with smaller radius resist rupture.
- On the other hand, the pressure needed to inflate a sphere should
be higher than the pressure inside (i.e. higher than P). Since P is
inversely related to the radius, the smaller the radius, the higher the
distending pressure needed for inflation.

The core of medical physiology (1) – 3rd edition Page 275


Venules and veins
- Their walls are relatively thin and easily distended. Therefore they
can accommodate large volume of blood (capacitance vessels); for
example they accommodate 50% of circulating blood at rest.
- Venoconstriction occurring during hypotension (e.g. shock) adds
large amount of this stored blood into the circulation.
- The inner layer (intima) of the limb veins is folded into valves.
These are absent from the veins in the abdomen, chest, head and
neck.
Lymphatics
- The excess interstitial fluid is collected by diffusion into lymphatic
capillaries.
- These start in the interstitium of the whole body, joined
progressively together and traverse lympn nodes to drain eventually
in the right and left subclavian veins, at the neck.
- Once within the lymphatics, the fluid is called lymph. The lymph is
similar to interstitial fluid; however, it circulates lymphocytes and
transports lipids (chylomicrons) absorbed in the small intestine. It
also carries small amount of protein filtered from capillaries.
- Lymphatic vessels contain valves, and there are no visible
fenestrations in their walls.
The pulse wave
- Is a pressure wave that travels along the blood vessels and
expands them on its way. The expansion is palpable as pulse.
- In the arteries it is known as the arterial pulse. The pulse wave
moves faster than the blood flow, especially with increased age (i.e.
when the arteries lose their elastic fibers and become rigid).

The core of medical physiology (1) – 3rd edition Page 276


The blood flow
- The blood flow through a blood vessel = pressure/ resistance.
- It is calculated directly by the Fick principle using special
substances consumed by the organs to which the blood flow is being
measured (revise measurement of the cardiac output). Examples
include: or example:
o N2O for cerebral blood flow (N2O = nitrous oxide)
o PAH for renal blood flow (PAH= Para-amino-hippuric acid)
o Oxygen for blood flow to the lung (i.e. the cardiac output)
- The aorta has the highest velocity of blood flow, then the velocity
decreases gradually to be lowest in the capillaries and then it
increases slightly in veins.
- The blood flow through straight blood vessels is laminar. However,
there is certain critical velocity, above which the blood flow becomes
turbulent.
- This critical velocity may be exceeded at sites of constriction within
a blood vessel, producing turbulence beyond the constriction, or in
the heart due to rapid blood flow or closure of the valves.
- Turbulent blood flow produces noise that’s heard as bruit over
arteries or a murmur over the heart whereas laminar blood flow is
silent.

CIRCULATION THROUGH SPECIAL ORGANS


THE CORONARY CIRCULATION
- The cardiac muscle is supplied by the right and left coronary
arteries that originate from coronary sinuses at the root of the aorta,
behind two cusps of the aortic valves.

The core of medical physiology (1) – 3rd edition Page 277


- The venous blood is drained to the right atrium through the
coronary sinus and anterior cardiac veins; however, small cardiac
veins (e.g. thebesian veins) drain into other chambers, resulting in
reduction of the PO2 in arterial blood (physiological shunt).
- Coronary blood flow to the left ventricle occurs during diastole and
to the atria and the right ventricle occurs during both systole and
diastole. This is explained by the pressure gradient between the
aorta and theses chambers during the cardiac cycle (see above).
- The coronary blood flow = 250 mL/min (= 5% of the cardiac output).
It is controlled by:
- Metabolites (high CO2, H+, lactate, K+, prostaglandins and
adenosine): These are the primary dilators of the coronary artery.
Among these metabolites adenosine is the most important dilator.
- Sympathetic stimulation: Both alpha and beta 2 receptors are found
in the coronary artery but the direct sympathetic effect is
vasoconstriction through alpha receptors; however, the sympathetic
increases contractility and therefore increases metabolites. Through
metabolites it causes vasodilatation and increases the coronary
blood flow (i.e. the indirect effect is more dominant).
- Parasympathetic stimulation: causes dilatation of the coronaries.

•Factors affecting the coronary blood flow (CBF) include:


- Tachycardia (decreases diastole and therefore decreases coronary
blood flow “CBF”).
- Aortic stenosis (associated with severe compression of the
coronary artery during systole of the left ventricle= decreases CBF).
- Hypotension (= decreases pressure in aorta and therefore
decreases CBF).

The core of medical physiology (1) – 3rd edition Page 278


- Reduction in the coronary blood flow (ischemia) decreases oxygen
supply to the cardiac muscle. This produces severe chest pain that’s
aggravated by exercise and relieved by rest (angina pectoris).
- Prolonged or severe ischemia (due to complete obstruction of the
coronary artery) results in necrosis and death of the cardiac muscle
(Myocardial infarction). Here the pain is not relieved by rest.

CEREBRAL CIRCULATION
= 0.75 L/min (measured by the Fick principle using inhaled N2O).
- In addition to the sympathetic and parasympathetic innervation,
cerebral blood vessels are supplied by sensory nerves. These nerves
carry pain sensation triggered by traction or injury of these vessels.
- The brain is very sensitive to hypoxia; occlusion of its blood supply
causes unconsciousness in about 10 seconds.
- Chronic hypoxia produces intellectual deficits and affects the basal
ganglia, the thalamus and the inferior colliculus; however, the
vegetative structures in the brain stem are more resistant to hypoxia.

SPLANCHNIC CIRCULATION
= The liver receives 1.5 L/min of blood (0.5L /min by the hepatic
artery and 1.0 L/min by the portal vein).
- Other abdominal viscera receive about 1.0 L of blood per minute.
- This indicates that the liver and viscera receive about 1.5 L of
arterial blood per minute (= about 30% of the cardiac output).
- However, the blood flow to the small intestine increases after a
meal for up to 3 hours and therefore shifts some of the blood supply
to the brain. That’s why subjects become sleepy after a meal.

The core of medical physiology (1) – 3rd edition Page 279


- The pressure in the portal vein has a clinical importance (= 10
mmHg). It is high in patients with portal hypertension (e.g. due to
bilharziasis).

CUTANEOUS CIRCULATION
= less than 0.5 L/min but varies with environmental temperature. It is
controlled by sympathetic neurons (no parasympathetic supply).
- Skin blood vessels develop the following reactions:
White reaction
- Follows light skin trauma by a pointed object. The reaction is
caused by reduction in capillary blood flow following contraction of
the precapillary sphincter.
Triple response
- Follows firm skin trauma by a pointed object. It involves three
changes in the skin:
o Red reaction: (due to capillary dilatation by the pressure)
o Local swelling (wheal): (due to increased capillary permeability)
o Diffuse reddening (flare): (due to arteriolar dilatation)
- The capillary changes appear to be due to local release of
substance P.
- The triple response is an example of axon reflex (in which afferent
impulses travel antidromically through axons of sensory nerves).
Reactive hyperemia
- Increased blood flow to an area after a transient occlusion of its
blood supply; due to vasodilatation caused by hypoxia and
metabolites.

The core of medical physiology (1) – 3rd edition Page 280


CARDIOVASCULAR RESPONSES TO EXERCISE
- The cardiovascular responses to exercise differ according to the
type and level of exercise and according to the age and degree of
training. Most of the responses involve changes in the cardiovascular
and reparatory systems. The following is a brief account about some
of the cardiovascular responses:
Increased heart rate due to:
o Increased sympathetic stimulation (due to stress)
o Withdrawal of the parasympathetic effect on the heart (more
important than the increased sympathetic)
o Circulating catecholamines (from adrenal medulla & sympathetic)
o Proprioceptors (send excitatory impulses to the medullary
centers)
o Temperature (stimulates the SA node directly)
o Bainbridge effect (due to increased venous return by the muscle
pump & respiratory pump)
o Stimulation of chemoreceptors (due to hypoxia and acidosis)
o Stretch receptors in the lung (when the lung is inflated, impulses
pass through the vagus to inhibit the cardio-inhibitory center)
- The maximum heart rate (MHR) achieved during exercise
decreases with age.
- The easiest method to calculate it is by the formula:
MHR = 220 - age
However, there are many alternative formulas suggested by
researchers (e.g. MHR= 206.9 - (0.67 x age)).
- Trained athletes start with lower heart rate because their resting
heart rate is low (they have physiological bradycardia).
The core of medical physiology (1) – 3rd edition Page 281
Increased stroke volume due to:
o Increased sympathetic stimulation (due to stress)
o Circulating catecholamines (from adrenal medulla & sympathetic)
o Temperature (stimulates the SA node directly)
o Stimulation of chemoreceptors (due to hypoxia and acidosis)
o Increased venous return = increases EDV (Frank-Starling’s law)
- Exercise with isotonic contractions produce marked increase in
stroke volume whereas exercise with isometric contraction produces
little change in it.
Increased venous return due to:
o Muscle pump
o Respiratory pump (hyperventilation)
o Venoconstriction (by the sympathetic and catecholamines)
Increased cardiac output due to:
o Increased HR & SV
o Depends on degree of training and level of exercise
o May reach more than 35L/min in athletes
Increased or decreased peripheral resistance (PR) due to:
o Compression of blood vessels during isometric contractions (=
increased peripheral resistance “PR”)
o Dilatation of skeletal muscle arterioles during isotonic contractions
(= decreased peripheral resistance). This dilatation occurs due to:
- Sympathetic cholinergic discharge acting on M receptors
- Circulating catecholamines acting on B2 receptors
- Metabolites
- Remember that: the level of exercise determines the net effect on
peripheral resistance. Hard exercise decreases PR.

The core of medical physiology (1) – 3rd edition Page 282


Blood pressure changes:
- Changes in systolic blood pressure are determined by the cardiac
output whereas changes in the diastolic blood pressure are
determined by the peripheral resistance.
- Generally, isometric contraction increases both systolic and
diastolic pressures whereas isotonic contraction increases the
systolic and decreases the diastolic; however, these effects depend,
to a large extent, on the level of exercise.
- In strenuous exercise the systolic is increased, the diastolic is
decreased and therefore the pulse pressure is increased.

HEART FAILURE & SHOCK


HEART FAILURE
- Defined as failure of the heart to meet the metabolic demands of
tissues.
- Causes include: Severe anemia, severe hypertension, arrhythmia,
myocardial infarction, valvular disease and thyrotoxicosis.
- Heart failure can be classified as:
o Left sided HF (LHF): failure of the left side of the heart
o Right sided HF (RHF): failure of the right side of the heart
o Congestive HF (CHF): failure of the left & right sides of the heart
- Other terms may be used: for example “high output heart failure”
Here the cardiac output is increased; however, the high COP is still
not enough to meet the metabolic demands of tissues which are
highly increased (e.g. in thyrotoxicosis).

The core of medical physiology (1) – 3rd edition Page 283


Physiological mechanisms in heart failure
= All the mechanisms in heart failure are stimulated by hypotension
and tissue hypoxia.
- These include activation of the renin-angiotensin-aldosteone
system because of the low blood supply to the kidney. This results in
retention of sodium and water and therefore contributes to edema
formation (see control of blood pressure).
- When the left ventricle fails to eject blood (LHF), blood accumulates
in the lung causing pulmonary edema whereas when the right
ventricle fails (RHF), blood accumulates in the venous side causing
raised JVP, hepatomegaly, ascites and lower limb edema.
- When the two ventricles fail (CHF), blood accumulates in the lung
and the venous side; therefore, all symptoms and signs of LHF and
RHF are found in CHF.
Symptoms of heart failure include:
- Due to blood accumulation in the lung & pulmonary edema:
o Breathlessness: occurs in LHF & CHF
o Cough: occurs in LHF & CHF
o Sputum with blood: occurs in LHF & CHF
- Due to blood accumulation in venous side:
o LL swelling: occurs in RHF & CHF
o Venous pulsations in the neck: occur in RHF & CHF
- Due to low blood supply to muscles:
o Fatigability & weakness: may occur in all types of HF
- Other symptoms:
o Palpitation: may occur in all types of HF
o Cyanosis: may occur in all types of HF (especially LHF & CHF)

The core of medical physiology (1) – 3rd edition Page 284


Signs of LHF include:
o Third heart sound (galloping rhythm)
o Signs of pulmonary edema (basal crepitations)
Signs of RHF include:
o Third heart sound (galloping rhythm)
o Generalized edema
o Engorged neck veins (raised JVP)
o Hepatomegaly and sometimes splenomegaly
o Ascites
Signs of CHF include:
o Signs of both LHF & RHF
Signs that may give clue to the possible cause of HF include:
o Displaced apex beat: Lt ventricular hypertrophy (hypertension)
o Pale mucus membranes: Anemia (or vasoconstriction)
o Murmur: Valvular lesion
o Irregular pulse: Arrhythmia (atrial fibrillation)
o Goiter (thyroid enlargement): Thyrotoxicosis
Investigations:
o To assess the heart: ECG, chest X ray and echocardiography.
o For a possible cause: Hb and PCV, level of thyroid hormones
in the plasma …
Treatment
o Correction of the cause (e.g. blood transfusion for anemia)
o Diuretics like “Lasix” (for the fluid overload)
o Angiotensin converting enzyme inhibitors (ACEI) (to inhibit
formation of angiotensin II and to decrease aldosterone)
o Others (e.g. oxygen, morphine …)

The core of medical physiology (1) – 3rd edition Page 285


SHOCK
- Defined as state of the circulation in which there is inadequate
tissue perfusion resulting in disturbance of functions.
Types
 Hypovolemic shock (due to decreased blood volume)
 Distributive or low resistance shock (due to vasodilatation)
 Cardiogenic shock (due to cardiac lesion causing low COP)
 Obstructive shock (due to obstruction of blood flow in the chest)
Symptoms
o Irritability (due to low blood supply to the brain)
o Thirst (due to hypovolemia)
o Palpitation (due to increased contractility of the heart for
compensation)
Signs (Not in all types of shock)
o Pallor and cold clammy skin (due to vasoconstriction)
o Sweating (due to sympathetic activation)
o Tachycardia (due to sympathetic activation)
o Hypotension (due to hypovolemia, vasodilatation, cardiac
lesion or kinking of the aorta …)
o Low urine output (due to low renal blood flow and high
release of ADH)
o Hyperventilation (due to chemoreceptor stimulation by hypoxia
or acidosis)
Body responses to shock
o Revise the mechanisms that control the blood pressure (very
rapid, less rapid and long term mechanisms).

The core of medical physiology (1) – 3rd edition Page 286


Complications of shock
- Failure to treat shock leads to refractory or irreversible state. Here a
positive feedback mechanism is initiated (low COP= low blood
pressure= low venous return= low COP= low blood pressure and so
on) which leads eventually to death.

Hypovolemic shock
- Subdivided according to the cause of hypovolemia into:
o Hemorrhagic shock
o Surgical shock
o Traumatic shock
o Burn shock
o Shock of fluid loss (e.g. by vomiting or diarrhea)
- The compensatory reactions to shock are rapid and long term (see
control of blood pressure).
- In hemorrhagic shock, plasma is restored in about 3 days whereas
red blood cells are restored in 4-8 weeks.

Distributive shock
- Subdivided according to the cause of vasodilatation into:
o Anaphylactic shock (caused by histamine released by allergic
reactions)
o Septic shock (caused by endotoxins released by bacteria)
o Neurogenic shock (caused by vasovagal attacks caused by
severe traumatic pain in certain sites; e,g. testicular trauma)
- Due to the vasodilatation, the skin is usually warm (this
differentiates hypovolemic shock, in which the skin is cold, from
distributive shock (like septic shock), in which the skin is warm.

The core of medical physiology (1) – 3rd edition Page 287


Cardiogenic shock
- Caused by diseases of the heart which impair the cardiac output.
- Examples include:
o Myocardial infarction
o Arrhythmias
o Congestive heart failure
- In addition to the symptoms and signs of shock, there are usually
symptoms and signs of pulmonary congestion.

Obstructive shock
- Occurs due to obstruction of blood flow within the chest (kinking of
the aorta or interruption of pulmonary blood flow).
- Examples include:
o Tension pneumothorax
o Cardiac tamponade
o Cardiac tumor
o Pulmonary embolism

The core of medical physiology (1) – 3rd edition Page 288


QUESTIONS FOR SELF ASSESSMENT-9 (BEST OF FIVE)
1. The cardiac muscle differs from skeletal muscle because it is:
a. Striated muscle
b. Supplied by the autonomic nervous system
c. Inactive in the absence of external stimulation
d. Tetanized after repeated stimulation
e. Active in the absence of oxygen
2. The stroke volume is directly proportional to:
a. Parasympathetic simulation
b. The chronotropic effect of sympathetic stimulation
c. Starling’s forces
d. The initial length of muscle fibers
e. All of the above
3. During the cardiac cycle, closure of the aortic valve occurs at the:
a. End of isometric contraction
b. Beginning of rapid ejection phase
c. Beginning of ventricular relaxation
d. End of ventricular contraction
e. End of rapid filling phase
4. Normal P wave in the ECG indicates:
a. Increased size of atria
b. Normal atrial contraction
c. Ventricular depolarization
d. Normal cardiac pacemaker
e. Papillary muscle repolarization
5. Which of the following about the ECG is not true:
a. Can detect the size of the heart
b. Is useful in detecting reduction in the coronary blood flow
c. It is recorded from limb leads and chest leads
d. It can detect some electrolyte disturbances
e. Is used for measurement of the cardiac output
6. Measurement of cardiac output using Fick principle depends on:
a. Measurement of circulation time
b. Frequent arterial blood sampling
c. Use of a dye injected into an arm vein
d. Calculation of blood flow using the formula (blood flow= Q x 60/Ct)
e. None of the above
7. Left ventricular heart failure leads to:
a. Increased pulmonary capillary hydrostatic pressure
b. Engorged neck veins
c. Bradycardia
d. Increased myocardial contractility
e. Ascites

The core of medical physiology (1) – 3rd edition Page 289


8. Coronary blood flow:
a. Is mainly regulated by sympathetic supply
b. Increases due to beta blockers
c. Is highest during systole because of myocardial activity
d. Increases when myocardial metabolism increases
e. Is regulated by myogenic autoregulation
9. If the radius of an artery is halved (divided by 2) its resistance will
increase:
a. 2 times
b. 4 times
c. 8 times
d. 16 times
e. 32 times
10. End diastolic volume may not increase in which of the following:
a. An increase in preload
b. Sympathetic stimulation
c. Reduction in the ejection fraction
d. An increase in venous return
e. An increase in afterload
11. The pressure in the left ventricle:
a. Varies between 0 am 120 mmhg
b. Rises rapidly during ventricular filling
c. Shows a maximum of 80 mmHg
d. Falls rapidly during the initial phase of diastole
e. Remains constant during the rapid ejection phase
12. Venous return:
a. Is increased on standing
b. Increases during inspiration
c. Is decreased by venoconstriction
d. When decreased, it activates bainbridge reflex
e. When increased, it increases both heart rate & stroke volume
13. Which of the following is a vasodilator:
a. Serotonin
b. Endothelin
c. Nitric oxide (NO)
d. Thromboxane A2
e. Norepinephrine
14. When viscosity of blood is increased, which of the following is
increased:
a. Systolic blood pressure
b. Diastolic blood pressure
c. Central venous pressure
d. Venous return
e. PCV

The core of medical physiology (1) – 3rd edition Page 290


15. Activity in the carotid sinus results in:
a. Tachycardia
b. Hyperventilation
c. Increased peripheral resistance
d. Edema
e. Sweating
16. “Lead II” in the ECG is:
a. Unipolar lead
b. Normally characterized by absent Q wave
c. Characterized by tall peaked T wave in hypokalemia
d. Characterized by absent P wave in fast atrial fibrillation
e. Not characterized by ST elevation in anterior myocardial infarction
17. The heart rate is increased by:
a- Parasympathetic stimulation
b- Beta blockers
c- Cervical sympathectomy
d- Denervation of the SA node
18. Concerning the ECG:
a- P wave follows atrial systole
b- ORS complex is due to ventricular repolarization
c- ST segment represents conduction in all the conductive system
d- the first heart sound follows the P wave
e- the third heart sound occurs at the same time as the P wave
19. The cardiac muscle proper:
a- Consumes protein as the main source of energy
b- Depends on anaerobic metabolism for generation of energy
c- Characterized by prepotential phase on its action potential
d- Undergoes rhythmic contractions
e- Is not protected from tetanus
20. The ECG is most important in detecting a major reduction in:
a- Ventricular contractility
b- Mean blood pressure
c- Total peripheral resistance
d- Cardiac output
e- Coronary blood flow
21. Propagation of Action Potential through the heart is fastest in:
a- SA node
b- AV node
c- Atrial muscle
d- Purkinge fibres
e- Ventricular muscle
22. The cardiac output:
a. Is about 2 L/min at rest in adults
b. Is always increased by increasing the heart rate

The core of medical physiology (1) – 3rd edition Page 291


c. Is decreased by increasing the stroke volume
d. Is not changed during exercise
e. Is higher in males than females
23. If a blood pressure is 130/70, which of the following is not true:
a. Systolic pressure equals 130 mmHg
b. Diastolic pressure equals 70 mmHg
c. Pulse pressure equals 60 mmHg
d. Mean arterial pressure equals 120 mmHg
e. The blood pressure is normal
24. The sino-atrial node (SA node):
a. Is an un-myelinated neural structure
b. Is found in the left atrium
c. Is supplied by the sympathetic but not the parasympathetic
d. Delays conduction from atria to ventricles
e. Responsible for the normal cardiac rhythm
25. The stroke volume is decreased by:
a. Stimulation of the parasympathetic
b. Calcium ions
c. Stimulation of the baroreceptors
d. Exercise
e. Digitalis
26. A drug that has a positive inotropic effect on the heart will:
a. Increase the heart rate
b. Increase the force of contraction
c. Decrease the force of contraction
d. Decrease the heart rate
e. Increase the end diastolic volume
27. The peripheral resistance in adults is mainly determined by:
a. Radius of arterioles
b. Radius of great arteries
c. Viscosity of the blood
d. Length of blood vessels
e. Radius of veins
28. Baroreceptors:
a. Are located in the carotid and aortic bodies
b. Their activity is increased on standing
c. Result in tachycardia when stimulated
d. Afferents from them terminate directly on the vasomotor centre
e. Send tonic discharge in response to normal blood pressure
29. Hypovolemic shock is characterized by all the following except:
a. Tachycardia
b. Sweating
c. Thirst
d. Vasodilatation

The core of medical physiology (1) – 3rd edition Page 292


e. Low urine output
30. Bradycardia may be produced by stimulation of
a.  1 adrenergic receptors
b.  2 adrenergic receptors
c.  adrenergic receptors
d. Muscarinic receptors
e. Nicotinic receptors
31. The stroke volume is proportional to:
a. Parasympathetic simulation
b. The chronotropic effect of sympathetic stimulation
c. Starling’s hypothesis
d. The initial length of muscle fibers
e. End systolic volume
32. The pacemaker prepotential:
a. Is a slow depolarization due to opening of Ca++ channels
b. Is a slow inecrease in the RMP
c. Maitained by opening of long acting Ca channels
d. Is due to a decrease in K+ efflux
e. Occurs only in the SA node
33. The following is true regarding action potential of cardiac muscle:
a. Increased extracellular K+ causes immediate depolarization
b. Repolaristion is due to Na+ current
c. Exrtacellular Na+ affects the pacemaker potential
d. Plateau phase of action potential is due to Ca++ influx
e. Repolarisation is due to delayed K+ efflux
34. Clinical examination of a patient shows pale mucous membranes,
a pulse rate of 110/min and blood pressure of 150/ 40 mmHg.
Which of the following is a possible diagnosis:
a. Heart failure
b. Hypertension
c. Aortic regurgitation
d. Hypothyroidism
e. High grade fever

Question 1. 2. 3. 4. 5. 6. 7. 8. 9. 10. 11. 12.


Answer b d c d e e a d d b a e
Question 13. 14. 15. 16. 17. 18. 19. 20. 21. 22. 23. 24.
Answer c b c d d d d e d e d e
Question 25. 26. 27. 28. 29. 30. 31. 32. 33. 34.
Answer c b a e d d d d d c

The core of medical physiology (1) – 3rd edition Page 293


CHAPTER 7
THE RESPIRATORY SYSTEM
STRUCTURE AND FUNCTION
Divisions
- The respiratory system is divided anatomically into:
Upper respiratory tract (URT)
- includes all structures outside the thoracic cavity (the chest). These
are the nasal cavity, pharynx, larynx and upper part of the trachea.
Lower respiratory tract (LRT)
- includes all structures inside the thoracic cavity. These are the
lower part of the trachea, bronchi, bronchioles, alveolar sacs
(including alveolar ducts and alveoli).

Fig 7.1: The respiratory system

The core of medical physiology (1) – 3rd edition Page 294


- The URT is characterized by presence of hair (within the nose),
ciliated epithelium, mucus secreting cells and rich blood supply.
- Evaporation of water from surface of the respiratory mucosa
moistens and equilibrates temperature of inspired air with that of the
body; thus making the inspired air suitable for gas exchange in the
distal parts of the lung.

- The respiratory system is divided according to major function:


Conducting zone:
- For conduction of air to the lower zone. Includes the nasal cavity,
pharynx, larynx, trachea, bronchi and bronchioles (up to the terminal
bronchioles).
Respiratory zone:
- For gas exchange. Includes the respiratory bronchioles and alveolar
sacs (alveolar ducts and alveoli).
Fig 7.2: Physiological divisions of the respiratory system

The core of medical physiology (1) – 3rd edition Page 295


Important anatomical points
The tracheo-bronchial tree
- Is formed by about 23 divisions.
- The first 16 divisions (starting from the trachea to the terminal
bronchioles) form the conducting zone whereas the other divisions
(starting from the respiratory bronchioles to the alveoli) form the
respiratory zone.
- The trachea and bronchi have cartilage in their walls but little
smooth muscle while the bronchioles have smooth muscle in their
walls but no cartilage.
- The cartilage supports walls of the trachea and large bronchi and
prevents their collapse when the pressure inside them is decreased
(as occurs during inspiration).
- This support is lost in cases of tracheomalacia; that’s why patients
suffer from an inspiratory sound due to URT obstruction (stridor).
The lungs
- The lungs (the right and the left lungs) are found within the thoracic
cavity, protected by the rib cage.
- Each lung consists of parenchymal tissue supporting airways, blood
vessels, nerves and lymphatics.
- Each lung is divided, by horizontal and oblique fissures, into lobes.
The right lung (consisting of 3 lobes) is larger than the left lung
(consisting of 2 lobes due to presence of the heart).
- The lungs provide a surface for gas exchange.
- LRT diseases mainly affect the lungs. They include: inflammation of
the lung (pneumonia), acute airway obstruction (asthma), chronic
airway obstruction “COPD” (emphysema), lung fibrosis ...

The core of medical physiology (1) – 3rd edition Page 296


The Pleural cavity
- Each lung is covered by a membrane that’s attached tightly to its
outer surface (the visceral pleura). The membrane continues to line
the inner surface of the chest wall (the parietal pleura).
- The potential space which is formed between the visceral and
parietal pleurae is called the pleural cavity.
- The pleural cavity contains a few millimeters of fluid that acts as a
lubricant. It allows easy expansion of the lungs and resists separation
of the two membranes, therefore normally, no cavity is actually
present.

Fig 7.3: The pleural cavity

FUNCTIONS OF THE RESPIRATORY SYSTEM


 Provides oxygen to the tissues
 Eliminates carbon dioxide from the tissues
 Participates in regulation of pH of the blood

The core of medical physiology (1) – 3rd edition Page 297


- These are the major functions of the respiratory system; however,
participation of other systems is essential to perform these functions;
especially the blood, cardiovascular system and the renal system.

Other functions of the respiratory system:


 Participates in regulation of body temperature. Hyperventilation
increases heat loss by evaporation from the mucus membranes.
This is especially important in animals like dogs “panting”.
 Has some important metabolic functions. Theseinclude:
- Conversion of angiotensin I to angiotensin II (by angiotensin
converting enzyme (ACE) which is produced by the pulmonary
endothelium).
- Inactivation of certain vasoactive substances like bradykinin
(also by ACE).
- Breakdown of arachidonic acid metabolites like prostaglandins
and leukotriens.
- Synthesis of surfactant (see below).
 Has many important defense mechanisms:
- Hairs within the nose for filtration of air (removes particles > 10
µm in diameter).
- Mucus on the surface of respiratory epithelium for trapping of
smaller particles (2-10 µm in diameter).
- Cilia on cells for transporting the trapped particles upwards
towards the nasopharynx to be swallowed or coughed out (= This
is known as muco-ciliary clearance).
- Cough or sneezing reflexes for ejection of unwanted substances
through the mouth to the outside.

The core of medical physiology (1) – 3rd edition Page 298


- IgA antibodies and complement proteins within the respiratory
secretions for certain antigens.
- Antiproteases (e.g. alpha 1-antitrypsin) for proteases released
from dead bacteria or WBCs.
- Alveolar macrophages for ingestion of bacteria, debris and
foreign particles that reach the alveoli (usually less than 2 µm in
diameter).
Remember these clinical notes:
 ACE inhibitors are used for treatment of hypertension. They block
the conversion of angiotensin 1 to angiotensin II; however, they
also block the breakdown of bradykinin by the angiotensin
converting enzyme. This results in many side effects caused by
the bradykinin, including angioedema and irritating cough. For this
reason, the angiotensin receptor blockers are very important for
patients who suffer from these side effects. The receptor blockers
block effects of angiotensin II without affecting the converting
enzyme; therefore the enzyme inactivates bradykinin.
 Congenital or acquired deficiency of one or more of the above
defense mechanisms results in certain respiratory problems, for
example: deficiency of IgA antibodies causes repeated respiratory
infections, deficiency of alpha 1-antitrypsin causes destruction of the
lung tissue by proteases (as occurs in emphysema) and impaired
muco-ciliary clearance due to immotile cilia causes repeated
respiratory infections (as occurs in primary ciliary dyskinesia, also
known as Kartagener's syndrome).

The core of medical physiology (1) – 3rd edition Page 299


VENTILATION
Definitions
- Ventilation is the process of getting air into and out of the lungs
during breathing. It is always adjusted to meet the metabolic
demands of the body (i.e. to provide sufficient oxygen and to
eliminate excess carbon dioxide).
- Hyperventilation refers to ventilation in excess of the metabolic
demands of the tissues. It usually results in hypocapnia (low Pco2).
- Hypoventilation refers to ventilation less than the metabolic
demands of the tissues. It results in hypercapnia (high Pco 2).
Mechanism of ventilation
- Air moves into or out of the lungs due to pressure gradient (when
the atmospheric pressure is higher than the intrapulmonary pressure
it gets into the lungs (= inspiration); and when the atmospheric
pressure is lower than the intrapulmonary pressure it gets out of the
lungs (= expiration)).
Fig 7.4: Ventilation

The core of medical physiology (1) – 3rd edition Page 300


- According to Boyle’s law: there is an inverse relationship between
pressure of gases and volume. Therefore, there is an inverse
relationship between pressure within the lungs and their size.
- Inspiration results in inflation of the lungs and therefore reduction in
intrapulmonary pressure (IPP) whereas expiration does the reverse.
- The following table shows variations in IPP during respiration:
Table 7.1: The intrapulmonary pressure
Phase of respiration (with closed glottis) IPP
End of normal inspiration -1 mmHg
End of normal expiration +1 mmHg
End of maximum inspiration -30 mmHg
End of max expiration (valsalva maneuver) > +50 mmHg
Remember that:
 The atmospheric pressure is 760 mmHg. Values of the IPP
indicate difference from the atmospheric pressure. A value of -1
mmHg indicates a pressure less than the atmospheric by 1 (i.e.
759 mmHg) whereas +1 indicates a higher pressure (761 mmHg).
 Closure of the glottis at the upper part of the larynx isolates the
respiratory system from the atmosphere. Therefore, there is no
equilibration between IPP and atmospheric pressure.
 Opening of glottis allows equilibration of IPP with the atmospheric
pressure. That’s why the IPP at the end of all respiratory phases
equals zero (i.e.=760 mmHg similar to the atmospheric pressure).
Intrapleural pressure (IPLP)
- The intra pleural pressure (IPLP) undergoes similar changes;
however, opening and closure of the glottis do not change its values.
- The following table shows variations in IPLP during respiration:

The core of medical physiology (1) – 3rd edition Page 301


Table 7.2: The intrapleural pressure
Phase of respiration IPLP
End of normal inspiration -6 mmHg
End of normal expiration -2.5 mmHg
End of maximum inspiration -30 mmHg
End of max expiration (valsalva maneuver) > +50 mmHg
Remember that:
 The intra-pleural pressure is subatmospheric (-ve) during normal
inspiration and expiration whereas the IPP is –ve during
inspiration and +ve during expiration.
- Negativity of intra-pleural pressure is explained by the tendency of
the lung and the chest wall to recoil into opposite directions (the lung
tends to recoil inwards, due to its elastic properties, whereas the
chest tends to recoil outwards, due to position of the ribs).
- The point when recoil of the lung equals recoil of the chest is the
end of normal expiration. Here the volume of air within the lungs
equals the functional residual capacity (see below).
- In the upright position, there is difference in intrapleural pressure
between apex and base of the lung, because of gravity. The pressure
at the apex is lower than that at the base. That’s why there is higher
tendency of bullae at the apex of an emphysemayous lung to rupture
more than bullae at the base.
Mechanism of Inspiration:
 Contraction of inspiratory muscles
 Expansion of the chest
 Reduction of intra-pleural pressure

The core of medical physiology (1) – 3rd edition Page 302


 Expansion of the lungs
 Reduction of intra-pulmonary pressure
 Air moves into the lungs
- Inspiration is an active process because it involves contraction of
inspiratory muscles. The inspiratory muscles can be grouped into:
Inspiratory muscles working at rest (and during exercise):
 The diaphragm:
- Responsible for about 75% of inspiration
- Descends down during contraction (about 1.5 up to 7cm). This
increases the vertical diameter of the chest
- Supplied by the phrenic nerve (C4)
- Consists of central tendon, costal fibers and crural fibers
 The external intercostals muscle
- Contraction causes expansion of the chest by increasing the
antero-posterior and transverse diameters of the chest
Accessory inspiratory muscles working during forced inspiration only:
 Scalene
 Sternocleidomastoid
 Serratus anterior
 Trapezius
Mechanism of Expiration:
 Relaxation of inspiratory muscles
 Increased intrapleural pressure
 Recoil of the lungs to the expiratory position
 Increased intra-alveolar pressure
 Air moves out of the lungs

The core of medical physiology (1) – 3rd edition Page 303


- Expiration at rest is a passive process since it does not involve
contraction of any expiratory muscle.
- However forced expiration requires the action of the following
expiratory muscles that decrease the size of the chest:
 Internal intercostals muscle
 Abdominal muscles

Measurement of ventilation
- Volumes of air that enter or leave the lungs can be measured by
special devices (e.g. the Benedict Roth Spirometer).
Fig 7.5: The Benedict Roth Spirometer

The core of medical physiology (1) – 3rd edition Page 304


Volumes and capacities measured by the Spirometer:
 Tidal volume (TV)
- Volume of air inspired or expired each breath. = 0.5L in adult
males and females at rest.
 Inspiratory reserve volume (IRV)
- Volume of air inspired by maximum inspiratory effort following
tidal inspiration. = 3L in adult males and 1.9L in adult females.
 Inspiratory capacity (IC)
- Volume of air inspired by maximum inspiratory effort following
tidal expiration. = TV + IRV
 Expiratory reserve volume(ERV)
- Volume of air expired by maximum expiratory effort following tidal
expiration. = 1L in adult males and 0.7L in adult females.
 Vital capacity (VC)
- Volume of air expired by maximum expiration following maximum
inspiration. (= IRV + TV + ERV or = IC + ERV).
- Normal values: About 5L in adult males, 4L in adult females.
- Like other lung volumes, it differs according to age, gender, body
size (height or weight), position (higher during standing) and ethnic
background (higher in Western population than African ones).
- It is an important index of disease. For this reason it is usually
measured to diagnose certain respiratory problems.
- During measurement, the subject is asked to inspire air maximally
and then to expire maximally into the mouth piece of the Spirometer.
- When the subject is asked to expire forcefully and as quickly as
possible, the measured volume is called forced vital capacity (FVC).
- The FVC is an important parameter in chest medicine (see below).

The core of medical physiology (1) – 3rd edition Page 305


The forced vital capacity (FVC)
- Volume of air expired forcefully by maximum expiration following
maximum inspiration.
- Expiration normally takes about 4 s up to 6 s. Forced expiratory
time that takes longer than 6 seconds indicates airway obstruction.
- The volume of air expired during the first second of the FVC is
called the forced expiratory volume in the first second (FEV 1). It
equals more than 3 quarters of the FVC (i.e. more than 75%).
- For example when the FVC is 5 L, the FEV1 is about 4 L (i.e.
FEV1/FVC ratio = 80%).
- The values of the FEV1 and the FVC can be measured by a
vitalograph. The device provides FVC and FEV1 values on a graph
paper in the Y axis; whereas X axis represents time in seconds.
Fig 7.6: The vitalograph

- The FEV1/FVC ratio is usually measured to differentiate between


obstructive and restrictive lung diseases.
- Normal ratio is about 80% (see above).

The core of medical physiology (1) – 3rd edition Page 306


- In obstructive lung diseases (in which FEV1 is lower than normal)
the ratio is less than 75%, e.g. asthma, emphysema and chronic
bronchitis (see curve B in fig 7.7).
- In restrictive lung diseases (in which both FEV1 and FVC are lower
than normal) the ratio is normal or increased (up to 100%), e.g. lung
fibrosis and lung collapse (see curve C in fig 7.7).
- In combined problems (obstructive and restrictive problems), all the
parameters are lower than normal (i.e. low FEV1, low FVC and low
FEV1/FVC ratio), e.g. an asthmatic patient with lung fibrosis (see
curve D in fig 7.7).
Remember that:
 When the FEV1 = 5 L and the FVC = 5 L; the FEV1/FVC ratio =
100%. This indicates that the expiratory time is only one second
(i.e. wrong maneuver); however, the values indicate normal test.
 The use of the vitalograph is replaced by the digital spirometer
which gives FEV1, FVC and the FEV1/FVC ratio directly.
Fig 7.7: The vitalograph in different cases

The core of medical physiology (1) – 3rd edition Page 307


Table 7.3: Spirometry in different cases
Lung condition FEV1 FVC FEV1/FVC ratio
Normal Normal Normal Normal
Obstructive diseases Low Normal Low
Restrictive diseases Low Low Normal
Obstructive + Restrictive Low Low Low

Volumes and capacities not measured by the Spirometer:


 Residual volume (RV)
- Volume of air that remains in the lungs after maximum expiration.
= 1.2 L in adult males and 1.1 L in adult females.
- Higher volumes are found in obstructive lung diseases (due to
difficulty in expiration) and lower volumes in restrictive lung diseases.
- The residual volume has the following functions:
o Allows easy expansion of the lungs
o Allows continuous gas exchange throughout the respiratory cycle.
o Prevents complete lung collapse
 Functional residual capacity (FRC)
- Volume of air that remains in the lungs following tidal expiration.
= ERV + RV
 Total lung capacity (TLC)
- Volume of air accommodated by the lungs at the end of max.
inspiration.
= (IRV + TV + ERV + RV) or (IC + FRC) or (VC + RV)
= 6L in adult males and 5L in adult females.
- The RV & FRC are higher in:
o Males compared to females

The core of medical physiology (1) – 3rd edition Page 308


o Adults compared to children (i.e. increase with age)
o Obstructive lung diseases (asthma, chronic bronchitis and
emphysema) compared to restrictive (lung fibrosis).
- The above volumes and capacities can be measured by the
following methods; Body plethysmography, Helium Dilution
Technique or Nitrogen Washout Technique.
- Revise the last topic in this chapter (Lung function tests).
Pulmonary ventilation (or respiratory minute volume)
- The volume of air inspired or expired per minute
=TV X RR (where TV= tidal volume & RR= respiratory rate)
=500 X 12= (6L/min) at rest.
Alveolar ventilation
- The volume of air that ventilates the alveoli per minute
= (TV-dead space volume) X RR
= (500-150) X 12= (4200 ml/min) or (4.2L/min) at rest.
Fig 7.8: The spirogram

The core of medical physiology (1) – 3rd edition Page 309


Dead space volume (DS)
- Defined as volume of air that does not participate in gas exchange.
- Includes two types:
o Anatomical DS: volume of air that occupies the conducting zone.
o Physiological DS: volume of air that occupies the conducting zone
(anatomical DS) plus volume of air in the respiratory zone but not
participating in gas exchange (e.g. air within the upper alveoli that
receive low blood supply because of the gravity).
- The anatomical DS is about 150 ml in an average adult male (or
roughly it equals the weight of the body in pounds).
- The physiological DS volume = the anatomical dead space volume
+ any additional alveolar air not participating in gas exchange.
- Normally, the physiological is almost equal to the anatomical dead
space. The difference being less than 5 ml of air.
- The anatomical dead space volume can be measured by the single
breath nitrogen test (see lung function tests).
- The physiological dead space volume can be measured by the Bohr
equation: VD/VT = PACO2 - PECO2)/PACO2
Where: VD = Volume of dead space, VT = Tidal volume, PACO2=
Partial pressure of carbon dioxide in alveolar air and PECO2= Partial
pressure of carbon dioxide in expired air.
- Normally PACO2 is the same as PaCO2 (= Partial pressure of carbon
dioxide in arterial blood). Therefore measurement requires an arterial
blood sample to measure CO2 in arterial blood and a gas analyzer to
measure CO2 in expired air.
- Notice that CO2 expired from the alveoli that contain DS air is zero
(similar to the atmosphere) because there is no gas exchange.

The core of medical physiology (1) – 3rd edition Page 310


ELASTIC PROPERTIES: SURFACTANT AND COMPLIANCE
- The elastic properties of the lung are caused by:
o Elastic fibers in the lung tissue
o Surface tension of fluid in the alveoli (see below)
- These act against inflation of the lung.
- For ventilation to take place, it should overcome these two causes.
However, the two causes are affected by many factors. These are
discussed below:

SURFACTANT
 A phospholipid produced by type II alveolar cells.
 Acts to reduce surface tension of fluid in the alveoli. This is
achieved by covering the surface of water, separating it from air
(because the surface tension occurs at the water/air interface).
 The surface tension is a physical property of liquids. It arises
because the cohesive forces between water molecules attract
each other, tending to contract their surface and eventually cause
alveolar collapse.
 Reduction of the surface tension prevents:
o Alveolar collapse
o Development of pulmonary edema (due to negative interstitial
pressure caused by the alveolar collapse)
 The presence of surface tension in the lung was first noticed
when air and saline were compared during inflation of excised
lungs. Inflating lungs with saline was found to be easier than
inflating them with air. This is because there is no surface tension
acting against inflation when saline was used.

The core of medical physiology (1) – 3rd edition Page 311


 Surfactant effects are mainly exerted on small alveoli; especially
during expiration. This is because these small alveoli have higher
tendency to collapse.
 The higher tendency of small alveoli to collapse can be explained
by the law of Laplace:
P = 2T/r
Where: P= pressure inside the alveolus (= distending pressure), T=
tension and r= radius of an alveolus.
 This indicates that the smaller the radius the higher the distending
pressure needed to keep it patent.
 Production of surfactant starts late in pregnancy (after the 32 nd
week of pregnancy). Therefore it is deficient in pre term babies.
 These babies develop cyanosis & difficulty in breathing at birth. A
serious condition known as infant respiratory distress syndrome
(IRDS) or (hyaline membrane disease); characterized by collapse
of alveoli and retention of fluids in the interstitium and alveoli.
 Retention of fluid in the alveoli occurs because surfactant is
needed for maturation of epithelial sodium channels (ENaC)
responsible for absorption of sodium and water from the alveoli
after birth. Failure of maturation of these channels due to
deficiency of surfactant results in fluid retention.
 Treatment of IRDS requires, in addition to high oxygen supply
and fluid balance, inhalation of phospholipid or synthetic
surfactant.
 Surfactant production is increased by: Glucocorticoids (that’s why
pregnant women who develop premature labor contractions are
given injections of hydrocortisone, to increase its production).

The core of medical physiology (1) – 3rd edition Page 312


 Surfactant production is decreased by:
o Occlusion of the pulmonary artery
o Occlusion of a main broncus
o Chronic inhalation of (100%) oxygen
o Cigarette smoking

LUNG COMPLIANCE
 Compliance is defined as change in volume per unit change in
pressure.
 Lung compliance is described as the distensibility or stretchibility
of the lungs (i.e. the capacity of the lungs to expand or stretch).
 It differs from elasticity which is the resistance to that stretch (i.e.
compliance = 1/elasticity). Therefore when elasticity is decreased,
compliance is increased.
 It is measured in terms of change in volume per unit change in
pressure (i.e. C =  V/  P).
 Here a pressure volume curve is used for its measurement. For
example the relaxation pressure curve.
 To obtain the relaxation pressure curve, a subject breathing
through Spirometer is asked to inhale a given amount of air and
then to relax his respiratory muscles while his mouth and nose
are shut. During the process, the intrapulmonary pressure is
measured by a device in his mouth.
 Then the amount of inhaled air is increased and the procedure is
repeated until he takes max. inspiration, folled by max. expiration.
 The intrapulmonary pressure is plotted against volume as
appears in the following figure.

The core of medical physiology (1) – 3rd edition Page 313


 Notice that the relaxation pressure equals zero at the end of quiet
expiration (i.e. when the lung volume equals the functional
residual capacity, the point of equilibrium between the inward
recoil of the lungs and the outward recoil of the chest).

Fig 7.9: The relaxation pressure curve

 The slope of the curve equals compliance. However, it differs in


different lung volumes. For this reason the term specific lung
compliance is sometimes used. It equals the value of the lung
compliance divided by the lung volume
Specific compliance = Lung compliance / Lung volume
 The compliance obtained in this way is known as static lung
compliance.

The core of medical physiology (1) – 3rd edition Page 314


 It is important to notice that static lung compliance during inflation
is slightly lower than static lung compliance during deflation.
Therefore the relaxation pressure curves during inflation and
deflation are not the same. This is known as “hysteresis of the
lung”.
 When the change in lung volume is measured during breathing;
the resistance within the airways affects the value of compliance;
here the measured compliance is known as the dynamic lung
compliance.
 Normal values:
o Compliance of the lung (CL) = 0.2 L/cm H2O
o Compliance of the chest (CC) = 0.2 L/cm H2O
o Compliance of both (CL&C) = 0.1 L/cm H2O
- Notice that: 1/CL + 1/CC = 1/CL&C (like resistances connected in
parallel).
 Factors that increase lung compliance:
o Emphysema & old age (due to loss of elastic fibers in the lung)
 Factors that decrease lung compliance:
o Lung fibrosis, pulmonary edema, high surface tension of fluid
in alveoli (surfactant deficiency) and small lung size (children).
 Factors that decrease lchest compliance:
o Stiffness of joints, obesity and deformity of the chest wall (e.g.
kyphosis, scoliosis).
Remember that:
 High lung compliance decreases work of breathing whereas low
lung compliance increases work of breathing.
 Surfactant decreases work of breathing.

The core of medical physiology (1) – 3rd edition Page 315


Bronchial tone
- The smooth muscle in the bronchial wall is controlled by the
autonomic nervous system. The sympathetic dilates it (e.g. during
inspiration) and the parasympathetic constricts it (e.g. during
expiration); however, there are multiple irritants, chemicals and
hormones that may affect the normal tone of the bronchial tree; these
include:
Factors causing bronchoconstriction:
o Irritants & chemicals: e.g. sulfur dioxide
o Cool air
o Exercise (possibly by the cool air during hyperventilation)
o Substance P
o Adenosine
o Many inflammatory modulators & cytokines involved in the
pathogenesis of asthma (e.g. leukotriens); that’s why anti-
leukotriens are added for treatment of asthma.
Factors causing bronchodilation:
o Catecholamines
o VIP (vasoactive intestinal polypeptide)
- There is circadian rhythm in bronchial tone throughout the day, with
maximal constriction early in the morning. That’s why asthmatic
patients usually suffer from symptoms of airway obstruction early in
the morning.

The core of medical physiology (1) – 3rd edition Page 316


Pulmonary circulation
- Each lung receives its blood supply from two sources: the
pulmonary artery (which supplies the alveoli) and bronchial arteries
(which supply the airways and the pleurae).
- The pulmonary artery carries deoxygenated blood from the right
ventricle. It forms an extensive network of capillaries that surround
the alveoli to allow gas exchange. Oxygen is taken up into the blood
while carbon dioxide diffuses into the alveoli. The oxygenated blood
returns through pulmonary veins to the left atrium.
- The bronchial arteries carry oxygenated blood from the aorta. They
supply the lung parenchyma, airways and pleurae with oxygen and
nutrients. They also equilibrate temperature of inspired air with that of
the body. Deoxygenated blood is drained through bronchial veins to
the azygos vein and therefore to the right atrium via the inferior vena
cava; however, there are anastomoses between some bronchial
capillaries and pulmonary capillaries. This allows some
deoxygenated blood in bronchial capillaries to drain into the
pulmonary capillaries and then into the pulmonary vein, resulting in
the physiological shunt (mixture of oxygenated blood with some
deoxygenated blood).
- In addition, there is another source for the physiological shunt. It is
from small cardiac veins (thebesian veins) that drain deoxygenated
blood into the left side of the heart, which contains oxygenated blood.
- The physiological shunt results in reduction of arterial PO2 by 2
mmHg and reduction of arterial oxygen saturation by 0.5% compared
to oxygenated blood coming from alveolar capillaries.

The core of medical physiology (1) – 3rd edition Page 317


GAS EXCHANGE IN THE LUNGS

- There are two sites of gas exchange in the body:


o Between alveoli & pulmonary capillaries (in the lungs)
o Between tissue cells & systemic capillaries (in the tissues).
- Gas exchange in the lungs depends on the following factors:
o Pressure gradient of the gas
o Surface area of the respiratory membrane
o Thickness of the respiratory membrane
o Physical properties of the gas

1) Pressure gradient
- Gases move passively from an area of high pressure to an area of
low pressure.
- The pressure of a single gas in a container containing mixture of
gases is called its partial pressure.
- The partial pressure of a gas is calculated by multiplying its
fractional concentration times the total pressure of all gases. The
following table explains calculation of partial pressures of gases in
the atmosphere (dry air):
Table 7.4: Calculation of partial pressures of gases
Gas Percentage (%) Partial pressure
Nitrogen 78.06 78.06 x 760 = 593.3 mmHg
Oxygen 20.98 20.98 x 760 = 159.4 mmHg
Carbon dioxide 0.04 0.04 x 760 = 0.3 mmHg
Inert gases 0.92 0.92 x 760 = 7 mmHg
Total 100 760 mmHg

The core of medical physiology (1) – 3rd edition Page 318


Partial pressure of oxygen (Po2)
o In dry air = 159 mmHg
o In inspired air (after humidification in the airways) = 149 mmHg
20.98% x [760 - 47]; PH2O = 47 mmHg at body temperature
o In alveolar air = 100 mmHg (due to rapid diffusion of oxygen
into pulmonary capillaries & diffusion of CO2 into alveoli)
o In venous blood (coming to pulmonary capillaries) = 40 mmHg
(Prior to gas exchange)
o Po2 in arterial blood (leaving pulmonary capillaries)= 100 mmHg
(After the gas exchange; however, this value is decreased by the
physiological shunt).

Fig 7.10

The core of medical physiology (1) – 3rd edition Page 319


Partial pressure of carbon dioxide [Pco2 ]
o In dry air = 0.3 mmHg
o In inspired air = 0.29 mmHg (0.04% x [760 - 47]
o In alveolar air = 40 mmHg (due to rapid diffusion of CO2 from
pulmonary capillaries to alveoli)
o In venous blood (coming to pulmonary capillaries) = 45 mmHg
(Prior to gas exchange)
o Pco2 in arterial blood (leaving the pulmonary capillaries)= 40 mmHg
(After the gas exchange)

Fig 7.11

The core of medical physiology (1) – 3rd edition Page 320


2) Thickness
- The respiratory membrane consists of the following layers (fig 7.11):
o Fluid in the alveoli
o Alveolar wall (basement membrane + epithelium)
o Interstitial fluid
o Capillary wall (basement membrane + endothelium)
- Normal thickness = 0.5 micrometer
- Gas exchange is inversely proportional to thickness of the
respiratory membrane. For example when the thickness is
decreased (as occurs during exercise), gas exchange is increased.
- It is impaired when the thickness is increased (e.g. due to lung
fibrosis or pulmonary edema). This causes hypoxemia (low oxygen in
blood); however, thickness of the respiratory membrane is a less
common cause of hypoxemia than ventilation: perfusion mismatching
(see below).
Fig 7.12; The

The core of medical physiology (1) – 3rd edition Page 321


3) Surface Area
- The available area for gas exchange is called the effective surface
area. It indicates well ventilated alveoli in contact with well perfused
capillaries.
- Gas exchange is directly proportional to the effective surface area.
- For example when the surface area is increased (as occurs during
exercise), gas exchange is increased.
- The effective surface area is increased during exercise because:
o More alveoli are ventilated (due to increased ventilation)
o More capillaries are perfused (due to increased perfusion)
- Total surface area equals about 70 m 2 (normal range: 50-100 m2).
4) Diffusion Coefficient
- Defined as the amount of gas that diffuses across the respiratory
membrane per unit pressure difference per unit surface area per unit
time. It depends on:
o Solubility of the gas (direct relation)
o Molecular weight of the gas (inverse relation)
- Although molecular weight of CO2 is larger than O2, its diffusion
coefficient is higher than O2. This is due to the high solubility of CO2.
Diffusion capacity of the respiratory membrane:
- The volume of gas that crosses the respiratory membrane per unit
partial pressure difference per unit time. It is affected by:
o Thickness of the membrane (inverse relationship)
o Surface area of the membrane (direct relationship)
- It is measured by using carbon monoxide which is highly soluble in
blood (unlike other gases it is “diffusion limited”)
- Normal diffusion capacity equals 25 ml/min/mmHg.

The core of medical physiology (1) – 3rd edition Page 322


The Ventilation : Perfusion Ratio (V/Q ratio)
- The ratio of alveolar ventilation to pulmonary blood flow (perfusion).
- Alveolar ventilation is about 4 L/min whereas pulmonary blood flow
is about 5 L/min; therefore V/Q ratio = 0.8 (≈ 1.0).
- It is affected by: Gravity and lung diseases
Effect of gravity on V/Q ratio:
- In the upright position, the V/Q ratio differs in different parts of the
lung due to the effect of gravity.
At the apex
- Blood flow (Q) is decreased and ventilation (V) is also decreased
but to a lesser extent. Therefore the ratio is increased.
- When perfusion is decreased to zero, the ratio is increased to
infinity (V/Q =V/0 = Infinity).
- Since ventilation > perfusion, the extra air ≡ wasted ventilation (or
dead space ventilation).
At the base
- Blood flow (Q) is increased and ventilation (V) is also increased but
to a lesser extent. Therefore the ratio is decreased.
- Since ventilation < perfusion, the extra blood ≡ wasted perfusion (or
shunt flow).
Effect of lung diseases on V/Q ratio:
- Many lung diseases are characterized by V/Q inequality.
- These may result in either: wasted ventilation (e.g. pulmonary
embolism) or wasted perfusion (e.g. Lung collapse); the ratio is
changed accordingly.
- Remember that, V/Q inequality is the most common cause of
hypoxemia.

The core of medical physiology (1) – 3rd edition Page 323


Effect of V/Q mismatching on PO2 and PCO2 of alveolar air
 If ventilation to an alveolus is reduced relative to its perfusion (i.e.
less O2 supply from environment and less CO2 removal):
o PO2 in alveoli (PAO2) decreases
o PCO2 in alveoli (PACO2) increases
- This normally occurs in some alveoli at the base of the lung.
 If perfusion to an alveolus is reduced relative to its ventilation (i.e.
less carbon dioxide reaches the alveoli from blood):
o PO2 in alveoli (PAO2) increases
o PCO2 in alveoli (PACO2) decreases
- This normally occurs in some alveoli at apex of the lung.
- The lung apex is the most favorable site of infection for the tubercle
bacilli (because of the high PAO2).

GAS TRANSPORT IN THE BLOOD


TRANSPORT OF OXYGEN
- Oxygen is transported in the blood in two forms:
o Dissolved in plasma ……... 2%
o Bound to hemoglobin..…....98%
Dissolved oxygen:
= Solubility of oxygen x PO2
• Solubility of oxygen = 0.003 ml/100ml blood/mmHg
• Po2 = 100 mmHg (in arterial blood) and 40 mmHg (in venous blood)
- Therefore dissolved oxygen in 100 ml arterial blood=
100 x 0.003 = 0.3 ml oxygen/ 100 ml blood
- Dissolved oxygen in 100 ml venous blood=
40 x 0.003 =0.12 ml oxygen/ 100 ml blood

The core of medical physiology (1) – 3rd edition Page 324


Remember that PO2 in arteries (PaO2) is actually less than 100
mmHg (= 95 mmHg) because of the physiological shunt.
Oxygen bound to hemoglobin:
- Oxygen binds to hemoglobin in a rapid reversible oxygenation
reaction (iron remains in the ferrous state)
- The reaction takes less than 0.01 s.
- Each gram of Hb can carry up to 1.34 ml oxygen (If 100%
saturated; as in arteries).
- Oxygen bound to Hb in arterial blood can be calculated as follows:
o O2 = [Hb] x 1.34 x (% saturation of Hb with oxygen)
o [Hb] = 15 g/100 ml blood
o There fore O2 = 15 x 1.34 x 100% = 20 ml O2 /100 ml blood
- Oxygen bound to Hb in venous blood can be calculated as follows:
o O2 = [Hb] x 1.34 x (% saturation of Hb with oxygen)
o [Hb] = 15 g/dL
o Therefore O2 = 15 x 1.34 x 75% = 15 ml O2 /100 ml blood
- The relation between Po2 & percent saturation of Hb with oxygen is
explained by the oxygen hemoglobin dissociation curve:

The Oxygen-Hemoglobin Dissociation Curve


- Indicates direct relation between PO2 and % saturation of Hb with
oxygen.
- Sigmoid shaped. (starts slowly, becomes steep in the middle and
then reaches a maximum).
- The sigmoid shape can be explained as follows:
o Oxygen binds to the subunits of Hb successively (not all of them
at the same time).

The core of medical physiology (1) – 3rd edition Page 325


o The binding with the first subunit facilitates binding with the
second subunit and this facilitates binding with the third subunit &
so on.
o The facilitation occurs due to changes in the configuration of Hb
from the tense (T) form to the relaxed (R) form.
- Therefore, binding:
o Starts slowly: indicating low affinity of Hb to oxygen (occurs when
oxygen is binding to the first subunits).
o Becomes steep in the middle: indicating high affinity of Hb to
oxygen (occurs when oxygen is binding to the other subunits).
The affinity is increased up to 500 folds.
o Reaches maximum at the end (indicating full saturation).

Fig 7.13: The oxygen hemoglobin dissociation curve

The core of medical physiology (1) – 3rd edition Page 326


Notes to remember from the curve:
 PO2 of 40 mmHg gives oxygen saturation of 75% (as in veins)
 PO2 of 95 mmHg gives oxygen saturation of 97% (as in arteries)
 PO2 of 97 mmHg gives oxygen saturation of 97.5% (as in
oxygenated blood in pulmonary capillaries). Less saturation in
arteries (0.5%) is due to the physiological shunt.
 PO2 of 26 mmHg gives oxygen saturation of 50%. This is known
as the P50.
 The P50 is defined as the PO2 when Hb is 50% saturated with O2.
It is used to describe the affinity of Hb to O 2 (e.g. high P50
indicates low affinity of Hb to O2 whereas low P50 indicates the
reverse.

Affinity of hemoglobin to oxygen:


- The affinity of Hb to oxygen is affected by certain factors. These
factors can shift the curve to the right or to the left.
- The P50 gives information about affinity of hemoglobin to oxygen.
Normal value = 26 mmHg; higher values indicate shift to the right
(low affinity) & lower values indicate shift to the left (high affinity).
Shift to the right:
o Indicates lower affinity of Hb to oxygen (high P50)
o Indicates increased release of oxygen to tissues
o Caused by:
• High carbon dioxide,
• High Hydrogen ions (low pH)
• High 2,3 DPG
• High temperature

The core of medical physiology (1) – 3rd edition Page 327


- The 2-3 diphosphoglycerate (2,3 DPG) is a product of glycolysis. It
is highly present in RBCs when metabolism is increased. It is also
increased in exercise, high altitude and by some hormones like
growth hormone, thyroid hormones and androgens. Its binding to the
beta chain of Hb decreases the binding of Hb to oxygen).
Shift to the left:
o Indicates increased affinity of Hb to oxygen (low P50)
o Indicates decreased release of oxygen to tissues
o Caused by:
• Low carbon dioxide,
• Low hydrogen ions,
• Low 2,3 DPG (e.g. due to acidosis or in stored blood)
• Low temperature
• Myoglobin
• Hemoglobin F
Fig 7.14: Shifts of the oxygen-hemoglobin dissociation curve

The core of medical physiology (1) – 3rd edition Page 328


Remember that:
- Shift to the right occurs in tissues, where CO2, H+ & temperature are
high whereas shift to the left occurs in the lung where these factors
are decreased.
- Hb F & Myoglobin have very high affinity to oxygen. They shift the
curve to the left.
- Hb F binds less avidly to 2,3 DPG; this increases its affinity to O2.
- Anemia does not affect the shape of the curve. That’s because PO2
is normal and therefore the % saturation of Hb is normal.
Bohr effect
- The affinity of Hb to oxygen is decreased when the pH of the blood
falls.
- That’s why increase in CO2 content of the blood decreases the
affinity of Hb to O2 & causes shift of the curve to the right.

TRANSPORT OF CARBON DIOXIDE


- Carbon dioxide is transported in 3 forms:
o As Bicarbonate (the main form of transport)
o Bound to proteins (carbamino compounds)
o Dissolved
Dissolved CO2
- The solubility of CO2 is higher than O2 (up to 20 times).
- The dissolved CO2 constitutes about 5% of total CO2 in arterial
blood and 6% of total CO2 in venous blood.
- Generally there is no reaction between CO2 & water in the plasma
(due to absence of carbonic anhydrase enzyme in the plasma).

The core of medical physiology (1) – 3rd edition Page 329


Transport as Bicarbonate
- This is the main form of CO2 transport in the blood.
- CO2 diffuses inside RBCs and reacts with water in the presence of
carbonic anhydrase enzyme to produce carbonic acid & then
bicarbonate & hydrogen ion.
CO2 + H2O = H2CO3 = HCO3- + H+
- Hydrogen ions are buffered by hemoglobin.
- About 70% of bicarbonate diffuses to the plasma in exchange to
chloride (= Chloride shift).

Fig 7.15: Chloride shift

Remember that:
- Due to diffusion of CO2 into RBCs, the number of active osmotic
particles in RBCs is increased (by either HCO3- or Cl-)
- So, in venous blood water enters RBCs by osmosis, increasing the
size of RBCs. Then it passes out in the lung; when chloride leaves
out and the RBCs return to their normal size in arteries.
- That’s why PCV of venous blood is higher than arterial blood by
about 3%.

The core of medical physiology (1) – 3rd edition Page 330


Bound to proteins
- CO2 forms carbamino-compounds by binding to proteins (plasma
proteins in plasma and hemoglobin in RBCs).
- About 11% of CO2 in the blood is carried to the lungs as carbamino-
CO2.
Haldane effect
- Deoxy Hb in venous blood binds CO2 more readily than oxy Hb in
arterial blood.
- Therefore binding of oxygen to Hb in the lungs facilitates release of
CO2 from Hb; this is known as the Haldane effect.
- For this reason, arteries (containing oxygenated blood) carry less
carbon dioxide than veins (containing deoxygenated blood).

Summary
- CO2 is transported in plasma as:
o Dissolved
o Carbamino-CO2
o HCO3
- CO2 is transported RBCs as:
o Dissolved
o Carbamino Hb
o HCO3
- About 70% of the HCO3 enters the venous blood in exchange to
chloride (chloride shift).
- PCV of venous blood is higher than arterial blood by about 3%.

The core of medical physiology (1) – 3rd edition Page 331


CONTROL OF RESPIRATION
- Can be studied as: Neural control and Chemical control
Neural Control
- Two types:
o Involuntary control
- By the respiratory center
o Voluntary control
- By the cerebral cortex

The respiratory center


- Collection of neurons in the medulla & pons
- Arranged into 4 groups:
o Dorsal group
o Ventral group
o Apneustic center
o Pneumotaxic center
The dorsal group
o Found at the dorsal aspect of the medulla.
o It contains inspiratory neurons (it is responsible for inspiration).
o It is called the rhythmicity center because it can discharge
impulses rhythmically.
o The rhythmic discharge is initiated in the pre-Botzinger complex in
the medulla.
The ventral group
o Found in the ventral aspect of the medulla.
o Contains expiratory neurons + some inspiratory neurons.
o Inactive at rest (that’s why expiration occurs passively).

The core of medical physiology (1) – 3rd edition Page 332


o Inactivated when the dorsal group is stimulated & vice versa
(reciprocal innervation).
o Responsible for forced expiration.
Fig 7.16: The respiratory center

The Apneustic center


o Found in the lower part of the pons.
o Stimulates the dorsal group to increase depth of inspiration.
o Inhibited by: The vagus and the pneumotaxic center.
The Pneumotaxic center
o Found in the upper part of the pons.
o Its function is unknown, may be switching between inspiration &
expiration.

- Functions of the different groups of the respiratory center are


studied in animals by performing sections at various levels in the
brain stem.

The core of medical physiology (1) – 3rd edition Page 333


Section 1 [below the medulla]
o Results in death if the lesion is above C4. That’s because the
descending impulses from the respiratory center fail to reach
the phrenic nerve which supplies the diaphragm.
Section 2 [between medulla & pons]
o Results in gasping (shallow) respiration. That’s because the
apneustic center fails to increase depth of inspiration.
Section 3 [mid pontine section, + cutting the vagal supply]
o Results in apneustic breathing (deep inspiration). That’s
because the pneumotaxic center fails to inhibit the apneustic
center.
Section 4 [Above pons]
o No effect on involuntary respiration; however, it impairs the
voluntary control of respiration.

Fig 7.17: Sections below and above the respiratory center

The core of medical physiology (1) – 3rd edition Page 334


Factors affecting the respiratory center
- The respiratory center is affected by impulses coming from:
 Higher centers
• Cerebral cortex
• Hypothalamus
• Limbic system
• Other brain stem centers
 Baroreceptors
 Chemoreceptors
 Lung stretch receptors
 Proprioceptors
 Other receptors
Higher centers
- The cerebral cortex:
o For voluntary modification of respiration
o E.g. voluntary hyperventilation or voluntary apnea
- The hypothalamus (temperature center):
o Stimulates heat loss by increasing respiration
o E.g. panting in dogs
- The limbic system:
o Emotions may affect respiration (e.g. fear)
- Other brain stem centers:
o Stimulation or inhibition of the cardiac or the vasomotor centers
in the medulla, results also in stimulation or inhibition of the
respiratory center (impulses radiate between the centers).
That’s why hyperventilation is associated with tachycardia and
hypoventilation is associated with bradycardia.

The core of medical physiology (1) – 3rd edition Page 335


The Baroreceptors
- Stretch receptors found in the aortic & the carotid sinuses;
connected to the cardiac & vasomotor centers in the medulla by the
cranial nerves 9 & 10; send inhibitory impulses to these centers when
stimulated by stretch caused by high blood pressure.
- The inhibitory impulses decrease the sympathetic discharge from
these centers to the heart & blood vessels. This lowers the blood
pressure by:
o Decreasing the heart rate
o Decreasing contractility
o Vasodilatation
- The inhibitory impulses also inhibit the respiratory center. For this
reason, hypertension is associated with hypoventilation &
hypotension is associated with hyperventilation but mainly due to
activity of the chemoreceptors.
The Chemoreceptors
- See chemical control of respiration below.
The Proprioceptors
- Found in the joints, ligaments & tendons of muscles.
- Stimulated by movement (even passive movement).
- Send impulses directly to the respiratory center to increase
respiration.
The Lung stretch receptors
- Stretch receptors.
- Found in the smooth muscles of bronchioles.
- Stimulated by stretch during inflation of the lung.
- Send inhibitory impulses through the vagi to stop further inspiration.

The core of medical physiology (1) – 3rd edition Page 336


- This protective reflex is called Hering Breuer inflation reflex.
- It is not active in humans (except with very high tidal volume).
- There is also Hering Breuer deflation reflex stimulated by deflation
of the lung.
- Here excitatory impulses are carried also through the vagi to restart
inspiration.
Other receptors
(I) receptors:
o Found in upper respiratory tract
o Stimulated by irritants (dust, smoke, ...)
o Mediate coughing reflex, sneezing reflex...
(J) receptors:
o Found in juxtaposition to pulmonary capillaries
o Stimulated when the capillaries become distended with blood
o Function unknown, may be mediation of the sense of dyspnea

Chemical Control
- By chemoreceptors that detect chemical changes in blood or CSF
- There are two types of chemoreceptors:
o Peripheral chemoreceptors
o Central chemoreceptors
The peripheral chemoreceptors
- Special receptors found in: Aortic bodies and Carotid bodies
- Aortic bodies are found in aortic arch and carotid bodies are found
in carotid bifurcation.
- They are stimulated by:
o Hypoxia (the main stimulus)

The core of medical physiology (1) – 3rd edition Page 337


o Hypercapnia
o Acidosis
- They send excitatory impulses through the cranial nerves 9 & 10 to
stimulate the respiratory center in the medulla.
- This results in hyperventilation to correct the stimulus.
- The blood supply per gram tissue to these structures is very high,
e.g. the carotid bodies receive 2000 ml/100 gram tissue whereas 100
gram tissue in the kidneys receives 420 ml and in the brain receives
54ml. That’s why they can detect minor changes in the chemical
composition of the blood.

The central chemoreceptors


- Found on the anterolateral surface of the medulla; in contact with
the CSF.

Fig 7.18: The central chemoreceptors

The core of medical physiology (1) – 3rd edition Page 338


- The central chemoreceptors are stimulated by:
o High carbon dioxide (in blood and therefore in CSF)
o High hydrogen ions (in the CSF not that in blood)
- Hydrogen ions in the blood cannot cross the blood brain barrier
“BBB” easily. Hydrogen ions in the CSF are formed from carbon
dioxide as follows: Carbon dioxide crosses the BBB & reacts with
water in the CSF to produce carbonic acid & then dissociates into
bicarbonate & hydrogen ions. Then hydrogen ions in the CSF
stimulate the central chemoreceptors.
- In other words, carbon dioxide stimulates the central
chemoreceptors directly (by CO2 itself) and indirectly (by H+).
Remember that:
 In patients with chronic hypercapnia (e.g. patients with COPD),
respiration is stimulated by hypoxia not hypercapnia (because the
receptors become used to the high CO2).
 Treatment with high pressure O2 in these patients corrects
hypoxia and stops respiration. That’s why they should be treated
with low pressure O2 (e.g. 24% or 28%; not 80% or 100% O2).
HYPOXIA AND CYANOSIS
Hypoxia
 Defined as oxygen deficiency at the level of tissues or low tissue
oxygenation (Notice that hypoxemia is reduction of oxygen in the
blood).
 Classified into 4 types:
–Hypoxic hypoxia
–Anemic hypoxia
–Stagnant hypoxia
The core of medical physiology (1) – 3rd edition Page 339
–Histotoxic hypoxia
Hypoxic hypoxia
 Oxygen delivery to tissues is reduced because of low oxygenation
of blood.
 This is caused by:
–High altitude,
–Hypoventilation due to:
–Lung diseases
–Respiratory center depression
–Paralysis of respiratory muscles
–Venoarterial shunts
 It is characterized by:
–Low PO2 (in arterial blood)
–Low % saturation of Hb
–Low total oxygen content of the blood
 All these parameters are also low in venous blood.
 Oxygen therapy is useful in this type of hypoxia; however, when
the cause of hypoxia is veoarterial shunt, oxygen therapy fails to
increase oxygen content of the blood. This differentiates
venoarterial shunt from other types of hypoxia.
Anemic hypoxia
 Decreased oxygen carrying capacity of the blood as a result of:
–Decreased Hb (e.g. iron deficiency anemia).
–Abnormal Hb (e.g. methemoglobinaemia).
–Unavailable Hb (e.g. carbon monoxide poisoning).
 Characterized by:
–Normal PO2 in arterial blood

The core of medical physiology (1) – 3rd edition Page 340


–Normal % saturation of Hb
–Low total oxygen content of the blood
 These parameters are lower than normal in venous blood.
 Oxygen therapy is slightly useful in this type of hypoxia because it
only increases dissolved oxygen.
 Heavy smoking in a closed room may cause coma due to carbon
monoxide poisoning.
 Treatment of carbon monoxide poisoning requires administration
of oxygen within a container under high pressure “hyperbaric
oxygen”. That’s because the affinity of hemoglobin to CO is
higher than oxygen by 250 times.
Histotoxic hypoxia
 Inability of the tissues to take oxygen as a result of:
–Poisoning of the oxidative enzymes (cyanide poisoning)
–Long distance between blood & cells (edema)
 Characterized by:
–Normal PO2 (in arterial blood)
–Normal % saturation of Hb
–Normal total oxygen content of the blood
 All these parameters are higher than normal in venous blood.
 Oxygen therapy is not useful in this type of hypoxia.
Stagnant hypoxia
 Reduced blood flow to tissues as a result of:
–Obstruction of a supplying vessel (localized hypoxia)
–Heart failure (generalized hypoxia)
 Characterized by:
–Normal or low PO2 (in arterial blood)

The core of medical physiology (1) – 3rd edition Page 341


–Normal or low % saturation of Hb
–Normal or low total oxygen of blood
 When heart failure is mild, these parameters are low in venous
blood because of high uptake of oxygen in tissues; and then
corrected in the lungs to return to normal.
 In moderate-severe heart failure, all these parameters are lower
than normal in both venous and arterial blood; because the lungs
fail to oxygenate blood back to normal.
 Oxygen therapy is useful in this type of hypoxia.
CYANOSIS
 Bluish coloration of the skin & mucus membranes
o Appears when the concentration of reduced hemoglobin is
more than 5g/dL. For this reason it is rarely seen in anemia
and commonly seen in polycythemia.
 Types of cyanosis:
–Peripheral
o Appears at the tips of the fingers
o Due to peripheral vasoconstriction in response to cold
–Central
o Appears in the skin & mucus membranes
o Due to causes of hypoxic hypoxia & stagnant hypoxia
 Cyanosis is extremely rare in anemic hypoxia (anemic patients
may die when > 5g/dL of Hb become deoxygenated). It is also
extremely rare in CO poisoning (also anemic hypoxia) because it
gives reddish color. No cyanosis occurs in histotoxic hypoxia.
 Patients with methemoglobinemia may acquire a dark color that
resembles cyanosis.

The core of medical physiology (1) – 3rd edition Page 342


LUNG FUNCTION TESTS
Importance of lung function tests
- Depending on history and examination to diagnose a respiratory
problem, without objective measurement of lung function, may result
in wrong diagnosis.
- This was confirmed by many studies that recommend the use of
lung function tests as objective tools for:
o Diagnosis of respiratory diseases
o Follow up of respiratory problems
o Assessment of fitness for anesthesia
o Insurance purposes
o Clinical research
Types of lung function tests:
o Tests of mechanical properties
o Gas diffusion
o Blood gas interpretation
o Exercise testing
 Tests of mechanical properties
o Lung volumes and capacities
o Spirometry & the flow volume loops
o The peak expiratory flow
o Static compliance
o Airway resistance
o Respiratory muscle power

- This is a brief account about some of the devices and techniques


used in lung function tests:

The core of medical physiology (1) – 3rd edition Page 343


MEASUREMENT OF LUNG VOLUMES AND CAPACITIES
Benedict Roth spirometer
- Measures the tidal volume, inspiratory reserve volume, expiratory
reserve volume and the vital capacity (see above).
The helium dilution technique
- For measurement of many volumes and capacities especially total
lung capacity, residual volume and functional residual capacity.
- This method depends on the fact that helium does not diffuses
through the respiratory membrane into the blood.
- The subject is connected to a helium container at the end of:
o Maximum expiration (when measuring the residual volume)
o Tidal expiration (when measuring the functional residual capacity)
o Maximum inspiration (when measuring the total lung capacity)
- The subject breathes into a bag containing a known volume and
concentration of Helium (V1 & C1). This changes the concentration
of helium to C2 and expands the volume to V1 + the volume being
measured in the lung.
- Since C1 x V1= C2 x V2; V2 can be obtained by calculation.
- V2 = V1 + the required volume in the lung.
- Therefore the required volume can be obtained.
Body plethysmography (or body box)
- For measurement of all types of lung volumes and capacities
including the total lung capacity, residual volume and functional
residual capacity.
- Depends on the equation of gases: P1V1 = PV
- The subject sits within an airtight box. He breathes through a
mouthpiece that can be closed electrically (For example when it is

The core of medical physiology (1) – 3rd edition Page 344


closed at the end of normal expiration, the functional residual
capacity (FRC) can be measured). Then the subject inhales against
the closed shutter; his chest volume is increased while the volume
within the box is decreased. The pressure within the box can be
measured by a device connected to the box. The pressure within the
lung can be measured by a device within the mouth.
- At first the change in volume within the box is calculated using the
formula (P1V1 = PV). P1 is the pressure within the box (known), V1 is
the volume within the box (known), P is the new volume within the
box after inspiration (known), and V is the new volume within the box
after inspiration (unknown). The change in volume within the box =
V1 – V = X; this equals the change in volume within the lung.
- Then the formula (P1V1 = PV) is used again to calculate the lung
volume before inspiration (the FRC). P1 is the intrapulmonary
pressure before inspiration (known), V1 is the FRC (unknown), P is
the new intrapulmonary pressure at the end of inspiration (known),
and V is the new volume within the lung at the end of inspiration (=
FRC + X). X is known from the previous step; therefore the formula is
solved to obtain the FRC.
Single breath nitrogen washout test
- For measurement of the, residual volume, functional residual
capacity, uneven ventilation, anatomical dead space and closing
volume.
- The subject takes full inspiration of pure oxygen (100% O 2), then he
exhales slowly (not more than 0.5 L/S).
- A nitrogen analyzer near his mouth detects nitrogen concentration
in expired air.

The core of medical physiology (1) – 3rd edition Page 345


- Four phases appear when plotting nitrogen% against lung volume
as follows:
Fig 7.19

 Phase I = pure oxygen (from the dead space)


 Phase II = nitrogen + oxygen (mixture of dead space air +
alveolar air)
 Phase III = plateau of nitrogen (alveolar air); flat in normal people
& has increased slope in uneven ventilation
 Phase IV = rapid rise in [N2] from upper alveoli (following closure
of the distal airways).
- Dead space volume = volume of phase I + mid portion of phase II
- Phase III terminates at the closing volume
- The closing volume is the volume within the lung when the distal
airways begin to close because of less intra-mural pressure.
- The rise in the slope of phase IV and to some extent phase III is
due to expiration of air in the upper portions of the lung which contain
higher proportion of nitrogen (i.e. less diluted by the inspired oxygen).

The core of medical physiology (1) – 3rd edition Page 346


Spirometry (by the vitalograph or the digital spirometer)
- For measurement of the forced vital capacity (FVC), the forced
expiratory volume in the first second (FEV1) and the FEV1/FVC ratio
(read the section of volumes and capacities above).
- Other parameters and tests that can be measured by spirometry
include:
The forced expiratory volume 25-75% (FEF 25-75)
- The forced expiratory flow at the middle of the FVC
- Normally equals or exceeds 50% of the predicted FVC
- Has good correlation with the FEV1 in obstructive lung diseases.
- Has the advantage that it avoids the effort dependent first quarter of
the FVC.
- May diagnose mild airway obstruction while other spirometric
parameters are normal.
Bronchodilator studies
- See reversibility below
Bronchial Provocation Testing (challenge tests)
- These tests measure the response of the airways to chemical
substances known to cause broncho-constriction.
- This determines the extent of airway hyper-responsiveness which is
a characteristic of asthma.
- They are used to support the diagnosis of asthma when the results
of spirometry are not conclusive.
- The subject inhales increasing concentrations of a provocative
substance, followed each time by spirometry.
- A 20% fall in FEV1 is considered as a positive test.
- Examples of provocative substances: histamine & methacholine.

The core of medical physiology (1) – 3rd edition Page 347


- The provocative dose needed to decrease FEV1 by 20% is called
PD20%
- PD20% is lower in patients with bronchial hyper-responsiveness
than normal individuals.
- Other indirect provocative tests include exercise, adenosine and
non isotonic saline.
The spirometric flow volume loops
- The flow volume loops are a plot of inspiratory and expiratory flow
against volume. Flow is recorded on the Y-axis while volume is
recorded on the X-axis.
- When a subject completes his forced expiration during the FVC
maneuver, he immediately takes a deep inspiration to obtain a loop
as appears in the following figure:

Fig 7.20: the flow volume loop

The core of medical physiology (1) – 3rd edition Page 348


- The normal expiratory portion of the flow-volume curve is
characterized by a rapid rise to the peak flow rate, followed by a
nearly linear fall in flow as the patient exhales towards the residual
volume.
- Changes in the contour of the loop can aid in the diagnosis of
obstructive and restrictive lung diseases. However, it is not the
primary diagnostic tool of these disorders.

The peak expiratory flow (by the Peak Flow Meter)


- The PEF is the highest velocity of air flow achieved transiently
during forced expiration from the total lung capacity (i.e. after max
inspiration).
- The device used (the peak flow meter) is small, portable &
inexpensive device. It is manufactured in many designs: Wright peak
flow meter, Mini Wright peak flow meter and other designs
Fig 7.21: The peak flow meter

The core of medical physiology (1) – 3rd edition Page 349


- For interpretation of PEF readings (and other lung function test
parameters), it should be compared to predicted values (reference
values) that vary according to age, sex and body size (height).
- Normal value in an average adult male is about 600 L/min (or 10
L/s). Therefore a value of 300 L/min is low since it is 50% of
expected. Values more than or equal 80% of expected are normal.
- The PEF readings are used for:
1- Diagnosis of asthma (by variability and/or reversibility)
Variability:
- The PEF readings are normally variable. Lowest values are
obtained early in the morning (see the bronchial tone).
- Variability more than 20% indicates asthma. It is detected by
measuring and recording the PEF twice daily (morning and evening)
for two weeks. Variability= Highest PEF – Lowest PEF/Highest PEF
Fig 7.22: PEF variability

The core of medical physiology (1) – 3rd edition Page 350


Reversibility:
- The PEF is measured before and then 15 minutes after
administration of salbutamol (bronchodilator).
- Improvement of the PEF reading by more than 15% indicates
reversible airways, which is an important characteristic of asthma.
- The FEV1 is used more than the PEF for detection of reversibility,
because it is more reproducible. Here improvement of the FEV1 by
more than 12% (or 200 ml) indicates reversibility.
2- Assessment of severity of asthma
- Reduction of PEF to less than 50% of expected indicates acute
severe asthma whereas reduction to less than 33% of expected
indicates life threatening asthma.
3- Other uses of the PEF
- The PEF is used to guide therapeutic decisions in asthmatic
patients. It is also used to measure the degree of response to
therapy (i.e. follow up).

Lung compliance
- Read about compliance above

Airway resistance
- The force needed to inflate the lungs is greater than that needed to
overcome the elastic recoil alone.
- The additional impedance is the airway resistance.
- The airway resistance is defined as the pressure difference
between the alveoli and the mouth per unit of air flow. It is generated
from friction between air and mucosa.

The core of medical physiology (1) – 3rd edition Page 351


- Air way resistance = Mouth pressure –alveolar pressure/flow of air;
(Alveolar pressure is measured by the body plethysmograph).
- Airway resistance is higher when the airflow is turbulent than when
it is laminar. It is also increased by many other factors, including the
factors that increase the bronchial tone.

Respiratory muscle power


- Investigated by many tests to exclude respiratory muscle
weakness. These include the following tests:
Vital capacity
- It is a useful screening test to exclude respiratory muscle
weakness. When measured in an erect and then supine posture, the
difference is within 5%. This is because the abdominal viscera in
supine posture exert more loads on the respiratory muscles.
- A difference more than 25% indicates respiratory muscle weakness.
Maximum inspiratory pressure
- Measured during maximum inspiratory effort against an occluded
airway at RV or FRC. Normal value varies with age and sex. Values
above 80 cm H2O excludes respiratory muscle weakness.
Maximum expiratory pressure
- Measures the power of expiratory muscles at TLC.
- It is less useful in predicting the ability to breathe.
Transdiaphragmatic pressure
- Measures diaphragmatic function.
- Estimated from difference between pleural and abdominal
pressures (i.e. esophageal and gastric pressures).

The core of medical physiology (1) – 3rd edition Page 352


- Balloon catheters are used for measurement during spontaneous
breathing and during phrenic nerve stimulation.

Gas Diffusion studies


The transfer factor:
- Measures the transfer of carbon monoxide (CO) across the
respiratory membrane, from the alveoli to the RBCs.
- It is measured by many techniques. However, the single breath
technique is the best. Here the subject exhales maximally and then
inspires the test gas (containing 0.3% CO, 10% Helium in addition to
oxygen and nitrogen) to the total lung capacity.
- After 10 s he exhales into a bag, after discarding the first 0.5 L of
expired air.
- The sample is analyzed for CO and Helium. The change in
concentration of CO and Helium is used to calculate the transfer
factor.
- The transfer factor obtained by this method is a measure of the
diffusing capacity of the lung for carbon monoxide (D LCO). It is also
known as (TLCO), the transfer factor of the lung for CO.
- It is affected by diseases that affect the thickness of the respiratory
membrane and the V/Q ratio.
- For example it is reduced in lung fibrosis, emphysema, pulmonary
embolism …
- When the DLCO is corrected for differences in lung volume, it is
known as the transfer coefficient (KCO); (i.e. KCO = DLCO /alveolar
volume).

The core of medical physiology (1) – 3rd edition Page 353


Blood gas interpretation
- In addition to data obtained by the tests that assess mechanical
properties of the lung, diagnosis of respiratory problems may need
blood gas interpretation.
- This requires information about:
o Blood oxygenation (obtained by arterial blood gas analysis )
o Oxygen saturation of hemoglobin (see below)
o V/Q distribution (read about V/Q ratio above)
o Acid base status (read volume 2)

Pulse oximetry:
- The pulse oximeter probes, placed on fingers or earlobes are used
widely to assess oxygen saturation of Hb (SpO2).
- Light originating from the oximeter probe is absorbed by Hb at a
degree proportional to its level of oxygenation. This is calculated by a
processor to obtain the degree of saturation.
- Normal readings are higher than 95%. Readings lower than 88%
raises the suspicion of hypoxemia and indicates the need for arterial
blood gas analysis (ABG).
- There are certain conditions that give false estimation of the oxygen
saturation when using the pulse oximeter. For example
Carboxyhemoglobin and Methemoglobin give overestimation of O 2
saturation (higher readings) while fingernail polish and motion
artifacts underestimate the O2 saturation (lower readings).

The core of medical physiology (1) – 3rd edition Page 354


Exercise testing
- The cardiopulmonary exercise test is used to assess dyspnea that’s
not explained by other lung function tests. It is also used to assess
fitness for anesthesia, disability and response to therapy.
- The test assesses the responses of the respiratory, cardiovascular
and muscular systems to increasing workload offered by a bicycle
ergometer or treadmill.

Fig 7.23: Body plethysmography

The core of medical physiology (1) – 3rd edition Page 355


QUESTIONS FOR SELF ASSESSMENT-10 (BEST OF FIVE)
1. At the end of normal inspiration:
a. Intra-alveolar pressure equals -6 mmHg
b. Intra-pleural pressure equals zero when the glottis is open
c. The transverse diameter of the chest is increased by about 3 cm
d. The accessory muscles of inspiration are contracted
e. The abdominal muscles are relaxed
2. Surfactant:
a. Is a glycoprotein
b. Is produced in the fetus early in pregnancy
c. Prevents development of pulmonary edema
d. Acts to increase surface tension of fluid in the alveoli
e. Deficiency causes hypoventilation
3. Carbon dioxide (CO2):
a. Partial pressure in venous blood is about 40 mmHg
b. Partial pressure in arterial blood is about 100 mmHg
c. Is less soluble than oxygen in body fluids
d. Is not carried in hemoglobin
e. Is not the primary stimulus for hyperventilation in metabolic acidosis
4. Hypoxic hypoxia is caused by:
a. Heart failure
b. Cyanide poisoning
c. Carbon monoxide poisoning
d. Lung fibrosis
e. Anemia
5. Bronchodilatation is caused by:
a. ß blockers
b. Acetylcholine
c. Leukotriens
d. VIP
e. Substance P
6. Oxygen - Hemoglobin Dissociation curve is shifted to the right by:
a. Low CO2
b. Low 2,3 DPG
c. Low temperature
d. Low pH
e. Low metabolism
7. Acute hypoventilation increases:
a. PO2
b. PCO2
c. pH
d. Hemoglobin concentration
e. Blood viscosity

The core of medical physiology (1) – 3rd edition Page 356


8. The main form of carbon dioxide transport in the blood is:
a. Bound to hemoglobin
b. Bound to plasma protein
c. Dissolved in plasma
d. Bicarbonate
e. As carbonic acid
9. Volume of air that can be expired by maximum expiration
following maximum inspiration is the:
a. Tidal volume
b. Inspiratory reserve volume
c. Vital capacity
d. Expiratory reserve volume
e. Residual volume
10. Gas exchange in the lungs is increased by:
a. increasing thickness of the respiratory membrane
b. decreasing surface area of the respiratory membrane
c. increasing the pressure gradient of a gas between the two sides of
the respiratory membrane
d. decreasing the diffusion coefficient of the gas
e. hypoventilation
11. Gas exchange in the lungs is increased by:
a. Increasing thickness of the respiratory membrane
b. Decreasing surface area of the respiratory membrane
c. Increasing the pressure gradient across the respiratory membrane
d. Decreasing the diffusion coefficient of the gas
e. Hypoventilation
12. The diffusing capacity of the respiratory membrane:
a. Is not affected by thickness of the respiratory membrane
b. Increases in exercise
c. Is normal in anemia
d. Is best measured using helium
e. Is best measured using carbon dioxide
13. The partial pressure of oxygen in venous blood is:
a. About 60 mmHg
b. Normal in stagnant hypoxia
c. Higher than normal in histotoxic hypoxia
d. The same in all veins in the body
e. Gives accurate information about gas exchange in the lung
14. The functional residual capacity is best measured by:
a. Helium dilution method
b. Plethysmography
c. Challenge test
d. Spirometry
e. Nitrogen washout method

The core of medical physiology (1) – 3rd edition Page 357


15. Which of the following does not occur as blood passes through
systemic capillaries:
a. Hematocrit increases
b. Hemoglobin affinity to oxygen decreases
c. Viscosity decreases
d. pH decreases
e. red blood cell size increases
16. Histotoxic hypoxia is caused by:
a. Heart failure
b. Cyanide poisoning
c. Carbon monoxide poisoning
d. Ascending to high altitude
e. Anemia
17. Normal quiet expiration is associated with contraction of:
a. The diaphragm
b. Internal intercostal muscles
c. External intercostal muscles
d. Abdominal muscles
e. None of the above
18. Reduced affinity of Hemoglobin to O2:
a. Is known as the haldane’s effect
b. Favors uptake of O2 at the lung
c. Is caused by low (H+)
d. Indicated by high P50
e. Does not occur in Bohr’s effect
19. When TV is 500 ml, DS volume is 150 ml, RR is 10 breath/min and
COP is 5 L/min. All the following are correct except:
a. V/Q is 1
b. Pulmonary ventilation is 5 L/min
c. Vital capacity cannot be calculated
d. Dead space ventilation is 1.5 L/min
e. Alveolar ventilation is 4.2 L/min
20. Regarding activity of carotid bodies, which of these is not correct:
a. They are highly stimulated during hyperpnoea of high altitudes
b. Glomus cells release dopamine in response to hypoxia
c. Peripheral chemoreceptors are activated by low hemoglobin
d. They are not the major site of CO2 action on ventilation
e. Hypoxia causes closure of special type of potassium channels
21. High in 2-3 diphosphoglycerate is found in all the following except:
a. Exercise
b. High basal metabolic rate
c. Stored blood
d. High altitudes
e. Hyperthyroidism

The core of medical physiology (1) – 3rd edition Page 358


22. The physiologic dead space:
a. Is exactly equal to the anatomical dead space in normal people
b. Decreases during exercise
c. Is primarily due to the presence of the physiological shunt
d. Is normally about 50% of pulmonary ventilation
e. Is measured by the single breath nitrogen analysis
23. Cyanosis is:
a. Always present in fingers during cold exposure
b. Usually caused by carboxy-hemoglobin
c. Commonly seen in severe stagnant hypoxia
d. Rarely occurs in severe histotoxic hypoxia
e. Never expected in anemic hypoxia
24. The rhythemicity center:
a. Is located in the pons
b. Its damage causes apnoea
c. Activity causes expiration
d. Activity is increased by pneumotaxic center
e. Activity is increased by a rise in blood pressure
25. The functional residual capacity of an average adult male is about:
a. 1 liter
b. 2 liters
c. 3 liters
d. 3.5 liters
e. 4 liters
26. Which of the following is expected to be lower than normal in
obstructive lung disease:
a. TV
b. FVC
c. RV
d. FEF25-75
e. FRC
27. When total lung capacity 6 liters, vital capacity 4 liters, tidal
volume 0.5 liter, the residual volume is:
a. 1.5 liter
b. 2 liters
c. 2.5 liters
d. 3.5 liters
e. 1 liter
28. During normal quiet breathing:
a. The intrapelural pressure is always negative
b. Expiration is an active process
c. The antero-posterior diameter of the chest changes by 4 cm
d. The apex and base of the lung are equally ventilated
e. The apex and base of the lung are equally perfused

The core of medical physiology (1) – 3rd edition Page 359


29. Which of the following is not expected in a patient with anemia:
a. Normal PAO2
b. Normal PaO2
c. Normal % of Hb saturation with oxygen
d. Low oxygen carrying capacity of blood
e. High P50 of O2-Hb dissociation curve
30. At an altitude where the atmospheric pressure 600 mm Hg, the
partial pressure of O2 in dry air is approximately:
a. 120 mmHg
b. 125 mmHg
c. 130 mmHg
d. 135 mmHg
e. 150 mmHg
31. Carbon dioxide (CO2):
a. Partial pressure in venous blood is about 40 mmhg
b. Partial pressure in arterial blood is about 100 mmhg
c. Is more soluble than oxygen in body fluids
d. Is carried mainly in hemoglobin
e. Diffuses through biological membranes less easily than oxygen
32. ventilation is most sensitive to changes in:
a. pH of arterial blood
b. PO2 in arterial blood
c. Oxygen concentration in arterial blood
d. PCO2 in arterial blood
e. Noreadrenaline
33. Regarding the normal oxygen hemoglobin dissociation curve,
which of the following relations is not correct:
a. PO2 40 - percent saturation 75
b. PO2 60 - percent saturation 90
c. PO2 95 - percent saturation 98
d. PO2 50 - percent saturation 50
e. PO2 500 - percent saturation 100

Question 1. 2. 3. 4. 5. 6. 7. 8. 9. 10. 11. 12.


Answer e c e d d d b d c c c b
Question 13. 14. 15. 16. 17. 18. 19. 20. 21. 22. 23. 24.
Answer c b c b e d e c c b c b
Question 25. 26. 27. 28. 29. 30. 31. 32. 33.
Answer b d b a e b c d d

The core of medical physiology (1) – 3rd edition Page 360


FURTHER READING
1. Alderman MH, Cohen HW, Sealey JE, Laragh JH. Pressor responses to
antihypertensive drug types. Am J Hypertens 2010; 23:1031–1037.
2. Anderson PA, Bohlmann HH: Anterior decompression and arthrodesis
of the cervical spine: Long-term motor improvement: Improvement in
complete traumatic quadriplegia. J Bone Joint Surg 1992; 74:671-682.
3. Basran S, Frumento RJ, Cohen A, Lee S, Du Y, Nishanian E, Kaplan
HS, Stafford-Smith M, Bennett-Guerrero E. The association between
duration of storage of transfused red blood cells and morbidity and
mortality after reoperative cardiac surgery. Anesth Analg 2006;103:15-
20.
4. Benevento BT, Sipski ML. Spinal cord injury series. Phys Ther 2002;
82(6):601-612.
5. Besarab A, Levin A. Defining a renal anemia management period. Am J
Kidney Dis. Dec 2000;36(6 suppl 3):S13-23.
6. Branson RD, Johannigman JA. What is the evidence base for the newer
ventilation modes?. Respir Care. Jul 2004;49(7):742-60.
7. Brenda R. The Handbook of Cognitive Neuropsychology: What Deficits
Reveal about the Human Mind. Psychology Press; 2001.
8. Briel M, Meade M, Mercat A, et al. Higher vs lower positive end-
expiratory pressure in patients with acute lung injury and acute
respiratory distress syndrome: systematic review and meta-
analysis. JAMA. Mar 3 2010;303(9):865-73.
9. Chandramouli R. Textbook of physiology. 2nd ed. New Delhi: Jaypee
brothers; 2003.
10. Cullen B, O'Neill B, Evans JJ, Coen RF, Lawlor BA. A review of
screening tests for cognitive impairment. J Neurol Neurosurg Psychiatry
2007;78(8):790-9.
11. Dakin J, Kourteli E, Winter R. Making sense of lung function tests.
London: Arnold; 2003.
12. Deicher R, Horl WH. New insights into the regulation of iron
homeostasis. Eur J Clin Invest. May 2006;36(5):301-9.
13. Despopoulos A, Silbernagl S. Color atlas of physiology. 4th ed. New
York: Thieme; 1990.
14. Dudai Y. The Neurobiology of Memory: Concepts, Findings, Trends.
Oxford: Oxford University Press; 1989.
15. Easterbrook PJ. Basic medical sciences for MRCP part 1. London:
Churchil Livingstone; 1999.
16. Eiken O, Convertino VA, Doerr DF, Dudley GA, Morariu G, Mekjavic IB.
Characteristics of the carotid baroreflex in man during normal and flow-
restricted exercise. Acta Physiol Scand 144: 325–331, 1992.[Web of
Science][Medline]

The core of medical physiology (1) – 3rd edition Page 361


17. Fine EJ, Ionita CC, Lohr L. The history of the development of the
cerebellar examination. Semin Neurol 2002; 22(4):375-84.
18. Ganong WF. Review of medical physiology. 21st ed. Lebanon: McGraw-
Hill; 2003.
19. Geeta Akhwaja. Plantar reflex. Journal Indian Academy of Clinical
Medicine “JIACM” 2005; 6(3):193-197.
20. Guyton AC. Human physiology and mechanisms of disease. 4th ed.
Philadelphia: W.B. Saunders Company; 1987.
21. Hall JE. Pocket companion to Guyton and Hall textbook of medical
physiology. 2nd ed. Philadelphia: Elsevier; 2001.
22. Ichinose M, Saito M, Kondo N, Nishiyasu T. Time-dependent
modulation of arterial baroreflex control of muscle sympathetic nerve
activity during isometric exercise in humans. Am J Physiol Heart Circ
Physiol 290: H1419–H1426, 2006.
23. Isselbacher KJ, Braunwald E, WilsonJD, Martin JB, Fauci AS, Kasper
DL. Harrison's principles of internal medicine. 13th ed. New York:
McGraw-Hill; 1994.
24. Kandel ER, Schwartz JH, Jessell TM. Principles of Neural Science.
New York: McGraw-Hill; 2000.
25. Keele CA, Neil E, Joels N. Samson Wright's Applied physiology. 13th
ed. New York: Oxford university press; 1982.
26. Landrew BL. Experimental physiology. 9th ed. London: Churchill
Livingstone; 1972.
27. Luciano DS, Vander AJ, Sherman JH. Human anatomy and physiology:
structure and function. 2nd ed. New York: McGraw-Hill; 1983.
28. Marx SJ. Familial hypocalciuric hypercalcemia. In: Favus MJ ed. Primer
on the metabolic bone diseases and disorders of mineral metabolism.
4th ed. Philadelphia: Lippincott, Williams & Wilkins, 1999.
29. McGeon JG. Physiology. Singapore: Churchil Livingstone; 1996.
30. Meister M, Berry MJ. The neural code of the retina. Neuron 1999;
22(3):435-50
31. Mostafa S, Tagboto S, Robinson M, Burden A, Davies S. Over-
representation of diabetic patients with renal anaemia in the primary
care setting. Fam Pract. Jun 2009;26(3):180-2.
32. Myers A. Respiratory system: Crash course. Philadelphia: Elsevier
Mospy; 2006.
33. Navaneethan SD, Nigwekar SU, Sehgal AR, Strippoli GF. Aldosterone
antagonists for preventing the progression of chronic kidney disease: a
systematic review and meta-analysis. Clin J Am Soc
Nephrol. Mar 2009;4(3):542-51.
34. Nolte J. The Human Brain: an introduction to its functional anatomy. 5th
ed. St. Louis: Mosby; 2002.
35. Obayashi S. Possible mechanism for transfer of motor skill learning:
implication of the cerebellum. Cerebellum 2004; 3(4):204-11.

The core of medical physiology (1) – 3rd edition Page 362


36. Ogoh S, Fisher JP, Dawson EA, White MJ, Secher NH, Raven PB.
Autonomic nervous system influence on arterial baroreflex control of
heart rate during exercise in humans. J Physiol 566: 599–611, 2005.
37. Pizzi LT, Bunz TJ, Coyne DW, Goldfarb DS, Singh AK. Ferric gluconate
treatment provides cost savings in patients with high ferritin and low
transferrin saturation. Kidney Int. Dec 2008;74(12):1588-95.
38. Raven PB, Fadel PJ, Ogoh S. Arterial baroreflex resetting during
exercise: a current perspective. Exp Physiol 91: 37–49, 2006.
39. Sager M, Hermann B, La Rue A, Woodard J. Screening for dementia in
community-based memory clinics. WMJ 2006; 105(7):25-9.
40. Sala-Mercado JA, Ichinose M, Hammond RL, Coustos M, Ichinose T,
Pallante M, Iellamo F, O'Leary DS. Spontaneous baroreflex control of
heart rate versus cardiac output: altered coupling in heart failure. Am J
Physiol Heart Circ Physiol (January 11, 2008); doi:10:1152/ajpheart.
01186.2007.
41. Schulz HL, Goetz T, Kaschkoetoe J, Weber BH. The Retinome -
defining a reference transcriptome of the adult mammalian retina/retinal
pigment epithelium. BMC Genomics 2004; 5(1):50.
42. Siddall PJ, McClelland J. Non-painful sensory phenomena after spinal
cord injury. J Neurol Neurosurg Psychiatry 1999; 66:617-622.
43. Steensma DP, Hoyer JD, fairbanks VF. Hereditary red blood cell
disorders in middle eastern patients. Mayo Clin Proc 2001;76:285-293.
44. Weatheral DJ, L:edingham JG, Warrell DA. Oxford textbook of
medicine. 3rd ed, London: Oxford University Press;1996.
45. West JB. Respiratory physiology: the essentials. 4th ed. London:
Williams & Wilkins; 1993.
46. Willatts SM. Lecture notes on fluid and electrolyte balance. London:
Blackwell; 1982.
47. Williams WJ, Beutler E, Erslev AJ, Lichtman MA. Hematology. 4th ed.
New York: McGraw-Hill; 1990.
48. Ziegler R. Hypercalcemic crisis. J Am Soc Nephrol 2001; 12(suppl
17):S3-9.

The core of medical physiology (1) – 3rd edition Page 363


2010/548 :‫رقم االيداع‬

The core of medical physiology (1) – 3rd edition Page 364

You might also like